Diktat Full [PDF]

  • Author / Uploaded
  • ahmad
  • 0 0 0
  • Suka dengan makalah ini dan mengunduhnya? Anda bisa menerbitkan file PDF Anda sendiri secara online secara gratis dalam beberapa menit saja! Sign Up
File loading please wait...
Citation preview

Kata Pengantar



Puji syukur penulis panjatkan kehadirat Allah SWT Rabb semesta alam atas limpahan rahmat dan hidayah-Nya, sehingga penulis bisa menyelesaikan Diktat ini dengan judul “ Diktat Fisika Modern” tepat pada waktunya. Sholawat serta salam semoga tercurahkan kepada Rosul Allah Muhammad SAW yang telah membawa kita dari kegelapan kepada cahaya Rabbi, semoga tercurahkan juga kepada keluarga Beliau, sahabat dan semoga safa’at dapat kita terima di akhirat kelak. Amin. Penulis ingin menyampaikan terima kasih kepada Bapak Bambang Supriadi, dan teman-teman satu tim yang telah mendukung penyelesaian makalah sebagai tugas kuliah. Penulis menyadari sepenuhnya bahwa penyajian ini jauh dari tingkat kesempurnaan, maka dari itu kritik dan saran sangat kami harapkan demi perbaikan makalah ini. Mudah-mudahan bantuan dan dukungan yang diberikan Bapak atau semua pihak dapat menjadi amal jariyah yang bermanfaat. Dengan segala keterbatasan dan kelemahan yang ada pada penulis semoga Diktat ini dapat bermanfaat bagi yang membutuhkan.



November 2015 Penulis



i



DAFTAR ISI



Kata Pengantar



i



Daftar Isi



ii



BAB I Teori Relativitas Khusus



1



1.1 Teori Relativitas khusus 1.2 Postulat Einstein 1.3 Akibat Postulat Einstein 1.4 Transofrmasi Lorentz 1.5 Dinamika Relativistik 1.6 Dinamika Relativistik 1.7 Keserampakan dan Paradoks Kembar 1.8 Uji Percobaan Teori Relativitas Khusus 1.9 Latihan Soal BAB II Sifat Partikel Radiasi Elektromagnet



2 9 10 14 19 20 24 28 30 32



2.1 Tinjauan Ulang Gelombang Elektromagnetik 2.2 Radiasi Benda Hitam 2.3 Efek Fotolistrik 2.4 Efek Compton 2.5 Proses Foton Lainnya 2.6 Apakah Foton itu 2.7 Latihan Soal



BAB III Sifat Gelombang dari Partikel



33 43 50 57 60 61 62



64



3.1 Hipotesis de Broglie 3.2 Hukum Difraksi Bragg 3.3 Hubungan Ketidakpastian Bagi Gelombang Klasik 3.4 Hubungan Ketidakpastian Heisenberg 3.5 Paket Gelombang 3.6 Probabilitas dan Keacakan 3.7 Amplitudo Probabilitas 3.8 Latihan Soal BAB IV Persamaan Schrodinger



65 67 69 74 79 83 84 86 87



4.1 Pembenaran Persamaan Schodinger 4.2 Probabilitas dan Normalisasi 4.3 Beberapa Penerapan 4.4 Latihan Soal



ii



88 92 95 106



BAB V Model Atom Rutherford-Bohr



108



5.1 Sifat-Sifat Dasar Atom 5.2 Model Atom Thomson 5.3 Inti Atom Rutherford 5.4 Spektrum Garis 5.5 Model Atom Bohr 5.6 Percobaan Frank-Hertz 5.7 Asas Persesuaian 5.8 Pertanyaan



110 111 118 127 130 138 140 144



Daftar Pustaka



149



iii



1 Teori Relativitas Khusus



Relativitas klasik (yang diperkenalkan pertama kali oleh Galileo Galilei dan didefinisikan ulang oleh Sir Isaac Newton)



sederhana



mencakup



diantara



transformasi



benda



yang



bergerak dan seorang pengamat pada kerangka (inersia).



1



acuan



lain



yang



diam



TEORI RELATIVITAS KHUSUS



1.1 Kegagalan Relativitas Klasik Relativitas klasik (yang diperkenalkan pertama kali oleh Galileo Galilei dan didefinisikan ulang oleh Sir Isaac Newton) mencakup transformasi sederhana diantara benda yang bergerak dan seorang pengamat pada kerangka acuan lain yang diam (inersia). Jika kamu berjalan di dalam sebuah kereta yang bergerak, dan seseorang yang diam diatas tanah (di luar kereta) memperhatikanmu, kecepatanmu relatif terhadap pengamat adalah total dari kecepatanmu bergerak relatif terhadap kereta dengan kecepatan kereta relatif terhadap pengamat. Jika kamu berada dalam kerangka acuan diam, dan kereta (dan seseorang yang duduk dalam kereta) berada dalam kerangka acuan lain, maka pengamat adalah orang yang duduk dalam kereta tersebut. Pandangan paham Newton tentang alam memberi suatu kerangka nalar dasar yang membantu kita memahami sejumlah besar gejala alam. Pandangan tentanng alam ini, yang sebenarnya berasal dari Galileo, mengatakan bahwa ruang dan waktu adalah mutlak. Juga dikemukakan bahwa setiap percobaan yang dilakukan dalam kerangka acuan (pengamatan) kita barulah bermakna fisika apabila dapat dikaitkan dengan percobaan serupa yang dilakukan dalam kerangka acuan mutlak, yaitu sistem koordinat kartesius semesta yang padanya tercantelkan jam-jam mutlak. Sebagai contoh, pernyataan yang lazim dikenal sebagai asas kelembaman (inersia) Galileo, mengatakan bahwa sebuah benda yang diam cenderung diam kecuali jika padanya dikenakan gaya luar. Jika anda mencoba menguji asas ini dalam sebuah kerangka acuan yang mengalami percepatan, seperti sebuah mobil yang berhenti secara mendadak, atau sebuah komidi putar yang sangat cepat perputarannya, akan anda dapati bahwa asas ini tidak berlaku (dilanggar). Jadi, hukum-hukum Newton (termasuk asas kelembaman) tidak berlaku dalam kerangka acuan yang mengalami percepatan, kecuali dalam kerangka acuan yang bergerak dengan kecepatan tetap. Kerangka acuan (yang bergerak dengan kecepatan tetap) ini, disebut kerangka lembam (inersial). Peristiwa-peristiwa yang diamati dari berbagai kerangka lembam dapat tampak berbeda bagi masing-masing pengamat dalam tiap kerangka itu. Tetapi, mereka semua akan sependapat bahwa hukum-hukum Newton, kekekalan energi, dan seterusnya, tetap berlaku dalam kerangka acuan mereka. Pembandingan pengamatan-pengamatan yang dilakukan dalam berbagai 2



kerangka lembam, memerlukan transformasi Galileo, yang mengatakan bahwa kecepatan (relative terhadap tiap kerangka lembam) mematuhi aturan jumlah yangpaling sederhana. Transformasi Galileo. Relativitas berhubungan dengan dua kerangka acuan yang saling bergerak dengan kecepatan konstan. Pada Gambar 2.1.2a diilustrasikan kerangka acuan “diam”, yaitu pengamat yang diam di tepi rel dan kerangka acuan “bergerak”, yaitu pengamat yang berada dalam kereta. Kita dapat menjelaskan situasi ini dengan menggunakan kerangka acuan inersial. Pada Gambar 2.1.2 dilukiskan dua buah kerangka acuan inersial. Kerangka acuan S yang berhubungan dengan pengamat diam di tepi rel, memiliki system koordinasi XYZ dengan titik dasar O. Kerangka acuan S’ yang berhubungan dengan pengamat dalam kereta, memiliki koordinat X’Y’Z’ relatif terhadap kerangka acuan S. Mula-mula (saat t =t’= 0), titik asal kedua acuan adalah berimpit. Dalam transformasi Galileo yang akan kita turunkan ini, selang waktu yang dicatat oleh pengamat di S di anggap sama dengan yang dicatat oleh yang dicatat oleh pengamat di S’. Jadi, t’=t.



Gambar 2.12 (a) S, memiliki sistem kordinat XYZ dan S’, memiliki sistem kordinat X’Y’Z’ (b) Setelah selang waktu t, titik asal koordinat S’ berada sejauh v.t dari titik asal koordinat S . Setelah selang waktu t, koordinat setiap benda (missal titik P) pada kerangka acuan S’ kita nyatakan dengan koordinat pada kerangka acuan S. dari gambar 2.1.2b tampak bahwa O’P = OP – OO’ O’P adalah koordinat x’, OP adalah koordinat x, dan OO’ = v t, sehingga persamaan di atas menjadi x' = x – v t Koordinat y dan z dari benda tidak berubah karena kerangka acuan S’ dibatasi hanya bergerak sepanjang sumbu X, dan tidak pada sumbu Y dan Z. oleh karena itu



3



y' = y, z' = z Jadi, transformasi Galileo untuk koordinat dan waktu adalah



.....................................................(2.1-1) Transformasi kebalikannya adalah



....................................................(2.1-2) Untuk memperoleh transformasi Galileo untuk kecepatan, persamaan (2.1-1) kita diferensialkan terhadap waktu. x' = x – v t



𝑑𝑥′ 𝑑𝑥 𝑑 (𝑣𝑡) = 𝑣 = 𝑢𝑥′ , = 𝑢𝑥 𝑑𝑎𝑛 𝑑𝑡 𝑑𝑡 𝑑𝑡 sehingga kita peroleh transformasi Galileo untuk kecepatan adalah:



.....................................................(2.1-3)Transformasi kebalikannya adalah: ux = ux' + v uy = uy' uz = uz'



(2.1-4)



Di sini, ux' adalah komponen kecepatan benda sejajar sumbu X', uy' adalah komponen kecepatan benda sejajar sumbu Y', uz' adalah komponen kecepatan benda sejajar sumbu Z'. Transformasi Galileo untuk percepatan kita peroleh dengan mendeferensialkan Persamaan (2.1-3) terhadap waktu. ux' = ux – v



4



dux'/dt = ax', dux/dt = ax, dan dv/dt = 0 sebab v konstan, sehingga kita peroleh: ax' = ax dengan cara yang sama, kita peroleh:



ax' = ay, az' = az



Jadi, transformasi Galileo untuk percepatan adalah: ax' = ax ay' = ay .....................................................(2.1-5) az' = az Dari persamaan (2.1-5) dapat kita simpulkan bahwa F' = ma' sama dengan F = ma, sebab



a'



= a. sekali lagi tampak bahwa hukum-hukum mekanika berlaku sama, baik pada kerangka acuan S' ataupun kerangka acuan S. ini adalah sesuai dengan prinsip relativitas Newton yang telah ditanyakan sebelumnya. Contoh Soal : Dua mobil melaju dengan laju tetap di sepanjang sebuah jalan lurus dalam arah yang sama. Mobil A bergerak dengan laju 60 km/jam, sedangkan mobil B 10 km/jam. Masing-masing laju ini diukur relatif terhadap seorang pengamat di tanah. Berapakah laju mobil A terhadap mobil B? Jawab : Misalkan O adalah pengamat di tanah yang mengamati mobil A bergerak dengan laju v = 60 km/jam. Anggaplah O’ bergerak dengan mobil B dengan laju u = 10 km/jam. Maka v’ = v – u = 60 km/jam – 10 km/jam = 50 km/jam



5



Contoh soal Sebuah kereta api bergerak dengan kelajuan 70 km/jam. Seorang penumpang melemparkan benda dengan kelajuan 15 km/jam. Tentukan kelajuan benda terhadap orang yang diam di tepi rel kereta, jika arah lemparan: a. searah gerak kereta b. berlawanan dengan arah gerak penyelesaian Orang yang diam di tepi rel kereta kita pilih sebagai kerangka acuan s dan kereta api sebagai kerangka acuan s’ yang bergerak dengan kelajuan v=70 km/jam relatif terhadap s. a. Gerak benda searah gerak kereta api ux’ = 15 km/jam kelajuan benda relatif terhadap orang yang diam di tepi rel kereta: ux = ux’ + v = 15 + 70 = 85 km/jam b. Gerak berlawanan arah dengan gerak kereta api ux’ = -15 km/jam kelajuan benda relatif terhadap orang yang diam di tepi rel kereta: ux = ux’ + v = -15 + 70 = 55 km/jam



Permasalahan dengan relatifitas ini terjadi ketika diaplikasikan pada cahaya, pada akhir 1800-an, untuk merambatkan gelombang melalui alam semesta terdapat substansi yang dikenal dengan eter, yang mempunyai kerangka acuan (sama seperti pada kereta pada contoh di atas). Eksperimen Michelson-Morley, telah gagal mendeteksi gerak bumi relatif terhadap eter, dan tak ada seorangpun yang bisa menjelaskan fenomena ini. Ada sesuatu yang salah dalam interpretasi klasik dari relatifitas jika diaplikasikan pada cahaya dan kemudian munculah pemahaman baru yang lebih matang setelah Einstein datang untuk menjelaskan fenomena ini. Gejala gelombang secara umum dapat kita definisikan sebagai rambatan gangguan periodik melalui suatu zat perantara. Dengan cara apakah perambatan gelombang ini berlangsung, bergantung pada gaya-gaya yang bekerja antar partikel zat perantaranya. Setelah Maxwell memperlihatkan bahwa kehadiran gelombang elektromagnet diramalkan berdasarkan persamaan-persamaan elektromagnet klasik, para fisikawan segera melakukan berbagai upaya untuk mempelajari sifat zat perantara yang berperan bagi perembatan gelombang elektromagnet. Zat perantara ini disebut eter. Namun, karena zat ini belum pernah teramati dalam percobaan, maka dipostulatkan bahwa ia tidak bermassa dan tidak tampak, tetapi mengisi seluruh ruangan fungsi dan fungsi satu-satunya hanyalah untuk merambatkan gelombang elektromagnet. Konsep eter ini sangat menarik perhatian karena sekurang-kurangnya dua 6



alasan berikut. Pertama, sulit untuk membayangkan bagaimana sebuah gelombang dapat merambat tanpa memerlukan zat perantara (bayangkan gelombang air tanpa air). Kedua, pengertian dasar eter ini berkaitan erat dengan gagasan Newton tentang ruang mutlak, eter dikaitkan dengan Sistem Koordinat Semesta Agung. Dengan demikian, keuntungan sampingan yang bakal diperoleh dari penyelidikan terhadap eter ini adalah bahwa dengan mengamati gerak bumi mengarungi eter, akan terungkap pula gerak bumi relative terhadap “ruang mutlak”. Percobaan awal yang paling saksama untuk mendapatkan bukti kehadiran eter dilakukan pada tahun 1887 oleh fisikawan Amerika, Albert A. Michelson dan rekannya E.W. Morley. Percobaan mereka pada dasarnya mempergunakan interferometer Michelson yang dirancang khusus bagi maksud ini. Dalam percobaan ini, seberkas cahaya monokromatik (satu warna) dipisahkan menjadi dua berkas yang dibuat melewati dua lintasan berbeda dan kemudian diperpadukan kembali. Karena adanya perbedaan panjang lintasan yang ditempuh kedua berkas, maka akan dihasilkan suatu pola interferensiseperti tmpak pada gambar (2.3.)



Gambar 2.2 diagram skematis interferometer michelson. Seberkas cahaya dari sumber cahaya S dipisahkan menjadi dua berkas titik A. Berkas yang satu dipantulkan oleh cermin di B, sedangkan yang lainnya di C. Kedua berkas kemudian diperpadukan kembali untuk diamati interferensinya. Separuh gambar sebelah kanan memperlihatkan gambar sketsa dari peralatan Michelson ini. Untuk meningkatkan kepekaannya, cermin-cermin disusun sedemikian rua sehingga kedua berkas melintasi masing-masing lengan peralatan ini sebanyakdelapan kali, ketimbang hanya dua kali. Untuk mengurangi pengaruh getaran dari lingkungan disekitarnya, interferometer ini didudukkan di atas sebongkah batu datar persegi dengan luas permukaan 1,5 m yang mengapung pada genangan air raksa.



Gambar 2.3 beberapa frinji interferensi sebagaimana diamanati dengan interferometer Michelson dari gambar 2.2. Apabila panjang gelombang lintasan ACA berubah sebanyak setengah panjang gelombang relatif terhadap ABA , maka semua daerah terang akan berubah menjadi gelap dan yang gelap mejadi terang.



7



Gambar 2.3



Untuk sementara, marilah kita membayangkan bahwa bumi sedang bergerak



mengarungi eter dalam arah AB. Pada pola interferensi, pita-pita gelap terjadi di tempat kedua berkas cahaya berinteferensi secara meminimumkan (destructive), sedangkan pita-pita terang di tempat interferensinya maksimum (constructive). Interferensi minimum dan maksimum brgantung pada beda fase antara kedua berkas cahaya. Ada dua saham (contribution) bagi beda fase ini. Yang pertama berasala dari beda jalan (AB-AC), karena salah satu berkas menempuh jarak yang lebih panjang, sedangkan saham kedua bagi beda fase ini ternyata akan selalu ada meskipun panjang kedua lintasan berkas tepat sama. Seberkas cahaya yang “berenang” mengarungi eter dalam arah lawan turut aliran eter akan berbeda waktu tempuhnya dengan yang melintasi dalam arah silang dan kembali. Jika kita dapat memisahkan dan mengukur saham kedua ini, maka kita dapat menarik kesimpulan tentang “laju” aliran eter, dan dari sini pula tentang gerak bumi mengarungi eter. Saham bagi beda fase yang disebabkan oleh beda jalan, tentu saja tidak berubah, karena sekarang berkas sepanjang AC yang bergerak menuruti aliran eter, sedangkan yang sepanjang AB sekarang melawan aliran eter. Adanya perubahan tanda pada saham kedua ini diperkirakan bakal teramati sebagai perubahan pola frinji (fringes, atau pita) terang dan gelap bila peralatannya diputar. Setiap perubahan terang menjadi gelap atau gelap menjadi terang menggambarkan suatu perubahan fase sebesar 180ᴼ (setengah siklus), yang setara dengan keterdahuluan atau keterlambatan waktu sebesar setengah periode. Ketika Michelson dan Morley melakukan percobaan ini, mereka tidak mengamati adanya perubahan mencolok dalam pola frinji interferensi, yang mereka simpulkan hanyalah suatu pergeseran yang lebih kecil daripada 0,01 frinji, yang berhubungan dengan laju bumi mengarungi eter, paling tinggi 5 km/detik. Sebagai upaya terakhir, Michelson dan Morley bernalar bahwa mungkin gerak orbital bumi menghapus gerak translasi mengarungi eter. Jika hal ini benar, maka enam bulan kemudian, bumi akan bergerak dalam orbitnya pada arah yang berlawanan, sehingga dengan demikian penghapusan ini tidak akan terjadi. Ketika percobaan ini mereka ulangi enam bulan kemudian, kembali diperoleh hasil nihil.



8



1.2 Postulat Einstein Albert Einstein (1879-1955, warga Jerman-Amerika Serikat). Serang filsuf dan pencinta damai yangramah. Ia adalah guru intelektual bagi dua generasi fisikawan teori yang meninggalkan sidik karyanya dalam hampir setiap bidang kajian fisika modern.



Permasalahan yang dimunculkan percobaan Michelson-Morley ini ternyata baru berhasil terpecahkan dengan teori relativitas khusus yang membentuk landasan bagi konsepkonsep baru tentang ruang dan waktu. Teori ini didasarkan pada dua postulat berikut, yang diajukan Albert Einstein pada tahun 1905. Postulat I : hukum-hukum fisika tetap sama pernyataannya dalam semua sistem lembam Postulat II : laju cahaya memiliki nilai c yang sama dalam semua sistem lembam. Postulat pertama, secara mudah mengatakan bentuk hukum-hukum fisika yang berlaku pada suatu kerangka acuan inersial akan sama dengan hukum yang berlaku pada kerangka inersial lainnya. Misalnya pada suatu kerangka acuan inersial berlaku hukum kedua newton F = m.a (untuk massa konstan), maka pada kerangka acuan inersial lainnya bentuk persamaannya akan sama hanya saja F' = m'.a', nilai F, m, atau a-nya mungkin berbeda. Postulat pertama ini menyatakan bahwa tidak ada kerangka acuan mutlak hingga gerak benda hanya bersifat relatif, sehingga tidak mungkin mengukur kecepatan mutlak suatu benda, yang ada hanya kecepatan relatif. Sebagai contoh: seseorang berada di dalam pesawat terbang yang bergerak dengan kecepatan penerbangan konstan jika penumpang tersebut melempar bola ke atas, maka bola akan bergerak parabola. Begitu pula dengan orang yang berada di bumi bila melempar ke atas gerakan bola juga parabola. Hal ini berarti bahwa bola yang dilempar di dalam pesawat terbang dan dibumi sama-sama membentuk gerakan parabola. Postulat kedua tidak lain merupakan konsekuensi dari percobaan Michelson-Morley bahwa laju cahaya dalam arah silang maupun searah sumber adalah sama. Dan postulat kedua ini menegaskan pula bahwa laju cahaya pun akan tetap sama bagi pengamat yang sedang berada dalam keadaan gerak relatif, selama pengamat tersebut merupakan sistem inersial. Postulat yang kedua menunjukan bahwa bagaimana pun cara kita mengukurnya kecepatan cahaya tidak pernah berubah. Apa pun patokan yang kita gunakan untuk mengukur kecepatan cahaya, di 9



mana pun posisi kita mengukur, dan berapa pun kecepatan kita saat mengukur, kecepatan cahaya selalu konstan. 1.3 Akibat Postulat Einstein 1. Pemuluran Waktu atau Time Dilatation Pemuluran waktu ini maksudnya bahwa jika suatu jam bergerak dengan kecepatan tertentu, waktunya akan memuai (mulur). Misalnya ada seorang astronot yang membawa jam tangannya saat menjalankan misi ke luar angkasa. Pesawat luar angkasa yang membawanya meluncur sangat cepat. Jika kita, yang berada di bumi, punya teropong yang sangat sensitif dan bisa melihat ke dalam pesawat yang sedang meluncur cepat itu, kita bisa menggunakan teropong itu untuk mengintip jam tangan si astronot. Sebelum si astronot berangkat kita sudah menyesuaikan jam tangan itu dengan jam tangan yang kita gunakan di bumi. Aneh, di jam tangan si astronot yang sedang meluncur di luar angkasa itu lebih lambat dibanding jam tangan kita di bumi. Padahal sebelum ia berangkat kedua jam sudah dicocokkan dan si astronot tidak mengubahnya sama sekali sejak keberangkatannya itu. Jarum detiknya tampak bergerak lebih lambat dibanding jarum detik di jam tangan kita. Inilah yang disebut dengan waktu yang mulur saat bergerak pada kecepatan tinggi. Semakin besar kecepatan gerak suatu benda atau partikel, waktu akan berjalan semakin lambat bagi benda atau partikel tersebut. Tentu saja hal ini tidak dirasakan oleh si astronot. Menurut si astronot, jam tangannya tidak berubah kecepatannya, yang berubah justru kecepatan jam tangan kita di bumi yang tampak bergerak lebih cepat. Hal ini disebabkan segala sesuatu di dalam pesawat astronot bergerak lambat termasuk proses metabolisma tubuh, getaran atom dan sebagainya. Tinjauan dua pengamat O dan O’. O menembakkan seberkas cahaya menuju sebuah cermin berjarak L darinya dan kemudian mengukur selang waktu 2 ∆𝑡 yang dibutuhkan berkas tersebut untuk menempuh jarak ke cermin dan kemudian dipantulkan kembali ke O. (tentu saja L = c ∆𝑡). Pengamat O’ sedang bergerak denagan laju tetap u seperti tampak pada gambar 2.4. menurut pandangan O, pengiriman dan penerimaaan berkas cahaya ini sama, dan O’ bergerak menjauhinya (O) dalam arah tegak lurus. Gambar 2.5 memperlihatkan percobaan yang sama dari sudut pandang O’, yang menurutnya O sedang bergerak dengan kecepatan –u. Menurut pandangan O’ ini,



10



Gambar 2.4 pengamat O mengirimkan dan menerima seberkas cahaya yang dipantulkan oleh sebuah cermin. Pengamat O’ sedang bergerak dengan laju u.



Gambar 2.5 percobaan yang diperlihatkan pada gambar 2.4 sebagaimana dilihat oleh pengamat O’. Pengamat O memancarkan seberkas cahaya di titik A dan menerima pantulannya di B.



Berkas cahaya dikirim dari titik A dan diterima di titik B setelah selang waktu 2 ∆𝑡′ kemudian. Jarak AB baginya adalah 2u ∆𝑡′. Menurut O, berkas cahaya menempuh jarak 2L dalam selang waktu 2 ∆𝑡 , sedangkan menurut O’, berkas cahaya itu menempuh lintasan AMB yang berjarak 2 √𝐿2 + (𝑢 ∆𝑡 ′ )2 dalam selang waktu 2 ∆𝑡′ menurut relativitas Galileo, ∆𝑡 = ∆𝑡′, dan O mengukur lajunya cahaya c sehingga laju cahaya menurut pengukuran O’ adalah √𝑐 2 + 𝑢2 . Menurut postulat kedua einstein, ini tidaklah mungkin karena baik O maupun O’ keduaduannya haruslah mengukur laju cahaya yang sama, yakni c oleh karena itu, ∆𝑡 𝑑𝑎𝑛 ∆𝑡′ haruslah berbeda. Hubungan antara ∆𝑡 𝑑𝑎𝑛 ∆𝑡 ′ dapat kita cari dengan mengambil kedua pengukuran laju cahaya sama dengan c. Menurut O, c = 2L/2 ∆𝑡, jadi L = c ∆𝑡. Menurut O’, c = 2 √𝐿2 + (𝑢 ∆𝑡 ′ )2 /2 ∆𝑡′, jadi c ∆𝑡′ = √𝐿2 + (𝑢 ∆𝑡 ′ )2 dengan menggabungkan keduanya, kita dapati c ∆𝑡′ = √(c ∆𝑡)2 + (𝑢 ∆𝑡 ′ )2 dan pemecahannya bagi ∆𝑡′ adalah ∆𝑡′ =



∆𝑡



√1− 𝑢2 /𝑐 2



11



Keterangan: ∆𝑡 ′ = selang waktu yang diamati pada kerangka diam Δt = selang waktu pada kerangka bergerak u = kecepatan relatif Contoh Soal Dua orang A dan B adalah anak kembar. Pada umur 20 tahun A pergi ke ruang angkasa dengan pesawat yang lajunya 0,8c dan kembali ke bummi pada saat B berumur 30 tahun. Berapakah umur B menurut A yang baru kembali Penyelesaian ∆𝑡 =



∆𝑡0 2



√1−𝑢2 𝑐



10 =



∆𝑡0 2



√1−(0,8𝑐) 2 𝑐



10 = 10 =



∆𝑡0 √1−0,64 ∆𝑡0 √0,36



∆𝑡0 = 6 tahun Umur A = 20 + 6 = 26 tahun



2. Penyusutan Panjang Penyusutan panjang juga berkaitan dengan perbedaan kecepatan. Misalnya si astronot agak lelah, lalu mulai berbaring di tempat tidur yang sudah disediakan di pesawat luar angkasanya. Dengan teropong yang sama, kita bisa mengintip si astronot yang tidur berbaring itu. Aneh, sewaktu berbaring si astronot tampak lebih pendek? Sewaktu ia masih di bumi dan pesawatnya belum berangkat, ia tampak tinggi. Lebih aneh lagi, sewaktu ia sudah terbangun lagi dari tidurnya dan kembali berdiri, tiba-tiba ia kelihatan tinggi seperti biasa. Tetapi ia juga kelihatan lebih kurus saat berdiri. Hal ini terjadi karena ia sedang berada dalam pesawat yang meluncur cepat, saat ia tidur kita melihat panjang tubuhnya menciut (terjadi kontraksi panjang). Saat ia berdiri, kita melihat lebar tubuhnya menciut (juga merupakan kontraksi panjang). Ia sendiri tidak merasakan perubahan apa-apa di 12



dalam pesawat. Benda yang bergerak dengan kecepatan mendekati kecepatan cahaya akan tampak lebih pendek bila diukur dari kerangka diam. L’ = selang waktu yang diamati pada kerangka diam L = selang waktu pada kerangka bergerak u = kecepatan relatif Contoh Soal Sebuah benda yang panjangnya 1 meter diamati oleh pengamat yang beergerak dengan kecepatan 0,6c. Berapa panjang benda itu menurut pengamat? Penyelesaian : 𝑢2



L’ = 𝐿√1 − 𝑐 2 L’ = 1√1 −



(0,6𝑐)2 𝑐2



L’ = √1 − 0,36 L’ = √0,64 L’ = 0,8 meter



3. Efek Doppler Efek Doppler bagi gelombang cahaya dalam fisika klasik 𝑓′ = 𝑓



𝑣 ± 𝑣0 𝑣 ∓ 𝑣0



Postulat pertama Einstein mengatakan bahwa ini tidak mungkin berlaku bagi gelombang cahaya, karena gelombang cahaya tidak memerlukan zat perantara dan tidak ada percobaan yang dapat mengungkapkan geraak mutlak. 2



√1 − 𝑢2 𝑐 𝑓′ = 𝑓 𝑢 1−𝑐



13



𝑢 1+ 𝑐 𝑓 = √ 𝑢 1− 𝑐 ′



Rumus ini adalah rumus pergeseran Doppler yang taat asas dengan keddua postulat Einstein. Rumus ini tidak membedakan antara gerak sumber dan pengamat dan hanya bergantung pada laju relatif u. 1.4 Transformasi Lorentz Transformasi Galileo, persamaan (2.1-1) sampai dengan persamaan (2.1-4) hanya berlaku jika kecepatan-kecepatan yang terlibat lebih jauh dari cepat rambat cahaya, c. sebagai contoh dapat kita lihat bahwa persamaan pertama dari persamaan (2.1-2) tidak akan berlaku untuk kecepatan cahaya. Untuk cahaya yang bergerak terhadap kerangka acuan S’ dengan kelajuan ux’ = c akan memiliki kelajuan ux = ux’ + v atau ux = c + v terhadap kerangka acuan S. Jadi, jelaslah diperlukan suatu transformasi baru agar senantiasa berlaku bahwa pada kerangka acuan apa saja, kelajuan cahaya dalam vakum adalah c. Kekeliruan transformasi Galileo untuk kelajuan-kelajuan yang mendekati kelajuan cahaya adalah anggapan bahwa selang waktu pada kerangka acuan S’ sama dengan selang waktu pada kerangka acuan S (t =t) untuk memasukkan konsep relativitas Einstein, maka selang waktu ini tidaklah sama (t ≠ t). jika kita anggap transformasi ini adalah linier maka hubungan transformasinya akan mengandung suatu pengali 𝛾, disebut tetapan transformasi. Dengan demikian transformasi baru ini akan berbentuk: x = 𝛾( x’ +v t) y = y’



(2.2-1)



z = z’ Perhatikan, kita ,menganggap persamaan y dan z tidak berubah karena tidak ada perubahaan gerak pada arah ini. Prinsip relativitas ini menyatakan bahwa S’ bergerak kekanan terhadap kerangka acuan S sama saja dengan S bergerak ke kiri terhadap kerangka acuan S’. Oleh karena itu, transformasi kebalikan persamaan pertama dari Persamaan (2.2-1) adalah: x’ = 𝛾( x– v t) Sekarang jika pulsa cahaya meninggalkan titik asal bersama S dan S’ pada saat t=t’=0, maka setelah selang waktu t,pulsa tersebut akan menempuh sepanjang sumbu X sejauh x=ct (dalam kerangka acuan S) atau x’=ct’ (dalam kerangka acuan S’). Dari persamaan transformasi untuk x dam x’, kita peroleh: x = 𝛾( x’ + v t) 14



ct= 𝛾(ct’+vt’) ct= 𝛾(c+v)t’ ............................ (2.2-2) dan x’= 𝛾(𝑥 − 𝑣𝑡) ct’= 𝛾(𝑐𝑡 − 𝑣𝑡) ct’= 𝛾(c-v)t t’=



𝛾(𝑐−𝑣) 𝑐



t .................................(2.2-3)



Substisusikan t’ dari (2.2-7) ke (2.2-6), kita peroleh : ct= 𝛾(𝑐 + 𝑣) [ ct=



𝛾(𝑐−𝑣)



𝛾2 (𝑐+𝑣)(𝑐−𝑣) 𝑐



𝑐



𝑡]



𝑡



Bagi kedua ruas persamaan dengan t, kita peroleh: c=



𝛾2 (𝑐+𝑣)(𝑐−𝑣)



𝑐2 =



𝑐



𝛾



2 (𝑐 2



+ 𝑣2) 𝑐



𝑐2



𝛾 2 = (𝑐 2 +𝑣2 ) = 𝛾=√



𝑐2 𝑣2 𝑐 2 (1− 2 ) 𝑐



=



1 𝑣2



1− 2 𝑐



1 1−



𝑣2 𝑐2



Setelah kita mengetahui tetapan transformasi 𝛾 kita akan menentukan hubungan antara t dan t. Untuk mengerjakan ini, kita gabungkan persamaan x’= 𝛾(x-vt) dengan x= 𝛾(x’+vt’) dengan cara mensubsisusikan x kedalam persamaan pertama. x’= 𝛾(x-vt) x’= 𝛾([𝛾(x’+vt’)-vt] x’= 𝛾2 ( x’+vt’)- 𝛾vt x’= 𝛾2x’+ 𝛾2vt’- 𝛾vt 𝛾= 𝛾2vt’+( 𝛾2-1)x’



: 𝛾𝑣 15



𝛾2 −1



t= 𝛾𝑡′ +



x’ ..................................(2.2-4)



𝛾𝑣



Mari kita tentukan dahulu nilai dari



𝛾2 −1



, sebagai berikut.



𝛾𝑣



𝛾2 − 1 1 =𝛾− 𝛾 𝛾



=



𝑣2



𝑣2



1 2



√1−𝑣2



− √1 − 𝑐 2 =



𝑐



𝛾2 −1 𝛾



=



1−(1− 2 ) 𝑐 2



√1−𝑣2 𝑐



𝑣²/𝑐² √1− 𝑣² 𝑐²



𝛾²−1 𝛾



𝛾2 −1 𝛾𝑣



Masukkan nilai



=



𝑣²/𝑐² 𝑣² 𝑐²



𝑣√1−



=



𝑣 𝑐²



x



1 2



√1−𝑣2 𝑐



𝑣



= 𝑐² 𝛾……………………….(2.2-5)



𝛾2 −1 𝛾𝑣



dari (2.2-5) ke dalam (2.2-4) kita peroleh: 𝑣



t = 𝛾𝑡 ′ +



𝑐² 𝑣𝑥′



t = (𝑡 ′ +



𝑐²



𝛾𝑥′ )…………………………………………. (2.2-7)



Akhirnya dapatlah kita peroleh hasil transformasi baru tersebut sebagai berikut. x=



1 2



√1−𝑣2



(x’ + vt’)



𝑐



y = y’………………………………………………………..(2.2-8) z = z’ t = 𝛾(𝑡 ′ +



𝑣𝑥′ 𝑐²



)



atau transformasi kebalikannya x’ =



1 2



√1−𝑣2



(x - vt’)



𝑐



16



y’ = y………………………………………………………(2.2-9) z’ = z 𝑣𝑥



t’ = 𝛾(𝑡 −



𝑐²



)



Transformasi pada persamaan (2.2-8) dan (2.2-9) disebut transformasi Lorentz. Persamaan-persamaan ini pertama kali diusulkan dalam dalam suatu bentuk yang sedikit berbeda oleh Lorentz, pada tahun 1904. Ia mengajukan persamaan-persamaan ini untuk menjelaskan hasil nol dalam percobaan Michelson-Morley, dan untuk membuat persamaanpersamaan Maxwell memiliki bentuk yang sama untuk semua kerangka acuan inersial. Setahun kemudian, Einstein menueunkan persamaan-persamaan ini secara independen berdasarkan teori relativitasnya. Perhatikan, bukan hanya persamaan kedudukan x yang dimodifikasi jika dibandingkan dengan transformasi Galileo, tetapi juga persamaan waktu t. Akhirnya, dapatlah kita lihat bahwa dalam relativitas khusus Einstein, ruang dan waktu adalah relatif (dalam relativitas Newton, ruang dan waktu adalah mutlak). Transformasi Lorentz Untuk Kecepatan Seperti biasanya, kecepatan dapat kita peroleh dari turunan fungsi kedudukan terhadap waktu. Ux =



𝑑𝑥 𝑑𝑡



………………………………………………(2.2-10)



x = 𝛾𝑥 ′ + 𝛾𝑣𝑡 ′ 𝑑𝑒𝑛𝑔𝑎𝑛 𝛾 𝑑𝑎𝑛 𝑣 𝑏𝑖𝑙𝑎𝑛𝑔𝑎𝑛 𝑘𝑜𝑛𝑠𝑡𝑎𝑛 dx = 𝛾𝑑𝑥 ′ + 𝛾𝑣𝑑𝑡′…………………………………(2.2-11) 𝑣𝑥′



t = 𝛾(𝑡 ′ + t = 𝛾𝑡 ′ +



𝑐² 𝛾𝑣 𝑐²



)



x’



jika variable t, t, dan x’ kita tarik diferensialnya, kita peroleh: dt = 𝛾𝑑𝑡 +



𝛾𝑣 𝑐²



dx’……………………………………..(2.2.12)



Masukkan elemen dx dari (2.2-11) dan dt (2.2-12) ke dalam (2.2-10), sehingga kita peroleh kecepatan



17



𝑑𝑥



ux=



=



𝑑𝑡



𝛾𝑑𝑥 ′ +𝛾𝑣𝑑𝑡′ 𝛾𝑣



𝛾𝑑𝑡 ′ + 2 𝑑𝑥 𝑐



1



𝑥 𝑑𝑡′ 1 𝑑𝑡′



𝑑𝑥′ + 𝛾𝑣 𝑢𝑥 = 𝑑𝑡 𝛾𝑣 𝑑𝑥′ 𝛾+ 2 𝑐 𝑑𝑡′ 𝛾



= =



𝛾𝑢𝑥′ + 𝛾𝑣 𝛾𝑣



𝛾+ 2 𝑢𝑥 ′ 𝑐



, sebab



𝑑𝑥′ 𝑑𝑡′



= 𝑢𝑥′



𝛾(𝑢′ +𝑣) 𝑣



𝛾(1+ 2 𝑢𝑥 ′ ) 𝑐



𝑢



𝑥=



′ +𝑣 𝑢𝑥 𝑣 1+ 2 𝑐 𝑢𝑥 ′



Sekarang kita akan menentukan kecepatan pada sumbu y, uy. Dari persamaan (2.2-9) y=y’ , sehingga dy=dy’ 𝑑𝑦



uy = =



=



𝑑𝑡



𝑑𝑦′



𝛾𝑣



𝛾𝑑𝑡 ′ + 2 𝑑𝑥′ 𝑐



𝑑𝑦/𝑑𝑡



𝑑𝑡′



, sebab dy’/dt’=uy’ dan dx’/dt’=ux’



𝛾𝑣 𝑑𝑥′



𝛾+ 2 ( ′ ) 𝑐 𝑑𝑡 𝑢𝑦′



1



uy=𝛾 𝛾=



1



𝑥 𝑑𝑡′ 1



𝑣𝑢



[1+ 2𝑥′ ] 𝑐 1



1



2 √1−𝑣2 𝑐



𝑣2



𝛾=√1 − 𝑐 2 ,sehingga



𝑢



uy=



𝑣2 𝑦′ √1− 2 𝑐 𝑣𝑢𝑥 1+ 2 𝑐



Dengan cara yang sama, dapat kita peroleh kecepatan pada sumbu Z, uz , yaitu:



uz =



𝑣² 𝑢𝑧 ′√1−



1+



𝑐² 𝑣𝑢𝑥 ′ 𝑐²



Akhirnya dapatlah kita peroleh hasil transformasi Lorentz untuk kecepatan, yaitu : ux =



𝑢𝑥 ′+ 𝑣 1+



𝑣𝑢𝑥 𝑐²



𝑢𝑦′



uy = uz =



√1−𝑣² 𝑐² 𝑣𝑢 1+ 𝑥′ 𝑐²



………………………………………………(2.2-13)



𝑢𝑧′ 1+



𝑣𝑢𝑥′ 𝑐²



18



atau transformasi kebalikannya ux’ =



𝑢𝑥 + 𝑣 1+



𝑣𝑢𝑥 𝑐²



𝑢



uy’ =



𝑣² 𝑦√1− 𝑐² 𝑣𝑢 1+ 𝑥′ 𝑐²



………………………………………………(2.2-14)



𝑢



uz’ =



𝑣² 𝑧√1− 𝑐² 𝑣𝑢 1+ 𝑥′ 𝑐²



Contoh soal Pengamat O mengukur laju cahaya bernilai c. buktikan bahwa tranformasi kecepatan Lorentz memberikan nilai yang sama menurut pengukuran O’ Penyelesaian Jika system yang diamati adalah sebuah berkas cahaya yang bergerak dalam arah x, maka menurut pengukuran O, vx = c, dan kita harus mencari vx’ 𝑣𝑥′ =



𝑣𝑥 − 𝑢 𝑐−𝑢 = =𝑐 2 1 − 𝑣𝑥 𝑢/𝑐 1 − 𝑐𝑢⁄ 2 𝑐



Jadi O’ juga mengukur nilai c yang bergantung pada nilai u. keistimewaan tranformasi Lorentz ini tentu saja sesuai dengan postulat Einsten tentang ketetapan laju cahaya



1.5 Dinamika Relativistik Pengertian ketidak ubaha (invariance) ini terhadap translasi dalam waktu dan ruag, dan terhadap rotasi (pemutaran) dalam ruang dapat diperlihatikan setara dengan konsep kita tentang kekekalan energy, momentum linier, dan momentum sudut. Dengan demikian, membuang konsep-konsep ini menyiratkan bahwa kita hidup dalam alam semesta yang sangat aneh. Karena itu, kita akan tetap beranggapan bahwa alam semesta kita memilia semacam struktur yang sangat serasi, dan bahwa hukum-hkum kekekalan ini tetap berlaku, namun dngan catata bahwa relativitas khusu ungkin menghendaki suatu pedefinisian ulang terhadap besaranbesaran diamika dasa. Kita sebenarya dapat dengan segera menebak bahwa ini memang merupakan sesuatu hal yang perlu dilakukan. Andaikanlah kita kenakan suatu gaya etap F pada sebuah benda yang bermassa m, yang memberikan percepatan a = F/m. jika gaya tersebut kemudian kita kenakan selama suatu selang wakut yang cukup lama, maka dinamika klasik meramalkan bahwa patikelnya akan terus bertambah lajunga ingga melampaui laju cahaya. Tetapi, kita ketahui bahwa rasformasi Lorentz 19



member hasil yang tidak bermaka fisika bila u≥c. jadi, kita memerlukan sehimpunan hukum dinamika baru yang mencegah benda mengalami percepatan sehingga melaju melampaui laju cahaya. 1.6 DINAMIKA RELATIVISTIK Kita telah melihat bagaimana kedua postulat Einstein menuntut kita kepada suatu penafsiran “relatif” baru terhadap konsep-konsep mutlak yang dianut sebelumnya seperti panjang dan waktu. Juga darinya kita berkesimpulan bahwa konsep klasik kita tentang laju relatif tidak lagi benar. Dengan demikian, cukup beralasan bagi kita untuk menanyakan sejauh manakah revolusi konsep ini mengubah tafsiran kita terhadap berbagai konsep fisika. Oleh karena itu, kita sekarang membahas ulang berbagai besaran-besaran dinamika seperti massa, energi, momentum, dan gaya, agar kita dapat mengkajinya dari sudut pandang teori relativitas khusus. Apakah hubungan yang telah lazim kita kenal, seperti 𝑝 = 𝑚𝑣 , 𝐾 = 1⁄2 𝑚𝑣 2 , 𝐹 = 𝑚𝑎, (atau lebih tepat 𝐹 =



𝑑𝑝 ), 𝑑𝑡



tetap berlaku, apalah kita harus mempunyai



konsep baru lagi bagi besaran-besaran dinamika ini? Begitu pula, bagaimana halnya dengan hukumhukum kekekalan dasar dari fisika klasik, seperti kekekalan energi dan kekekalan momentum linear? Semua konsep ini begitu pentingnya dalam fisika klasik sehingga rasanya kita enggan membuangnya. Kedua hukum kekekalan ini bersama dengan hukum kekelan momentum sudut dapat diperlihatkan merupakan akibat dari kehomogenan dari keisotropian alam semesta. Jika kita mengoreksi semua efek lokal(seperti perubahan pada atmosfer atau keadaan lingkungan), maka percobaan yang dilakukan pada suatu hari tertentu akan memberikan hasil sama seperti yang diperoleh dari percobaan serupa yang dilakukan pada hari berikutnya. Pengertian ketidakubahan (invariance) ini terhadap translasi dalam waktu dan ruang, dan terhadap rotasi (pemutaran) dalam ruang dapat diperlihatkan setara dengan konspe kita tentang kekekalan energi, momentum linear, dan momentum sudut. Dengan demikian, membuang konsepkonsep ini menyiratkan bahwa kita hidup di alam semesta yang sangat aneh. Karena itu, kita akan tetap beranggapan bahwa alam semesta ini memiliki semacam struktur yang sangat serasi, dan bahwa hukumhukum kekekalan ini tetap berlaku, namun dengan catatan bahwa relativitas khusus mungkin menghendaki suatu pendefinisian ulang terhadap besaran-besaran dinamika dasar. Kita sebenarnya dapat dengan segera menebak bahwa ini memang suatu hal yang perlu dilakukan. Andaikanlah kita kenakan suatu gaya tetap 𝐹 pada sebuah benda bermassa 𝑚, yang memberikan percepatan 𝑎 = 𝐹/𝑚. Jika gaya tersebut kemudian kita kenakan selama suatu selang waktu yang cukup lama, maka dinamika klasik meramalkan bahwa partikelnya akan terus bertambah lajunya hingga melampaui laju cahaya. Tetapi, kita ketahui bahwa transformasi Lorentz memberi hasil yang tidak bermakna fisika bila 𝑢 ≥ 𝑐. Jadi, kita ememrlukan sehimpunan hukum dinamika baru yang mencegah benda mengalami percepatan sehingga lajunya melampaui laju cahaya.



20



Andaikanlah dua massa identik saling mendekati, masing-masing dengan laju 𝑣. Setelah bertumbukan, kita eproleh sebuah massa 2𝑚 dalam keadaan diam. Ii adalah gambaran menurut pengamat 𝑂 dalam laboratorium.



Sesudah



Sebelum 1



2 v



v



V=0



Marilah kita sekarang beralih ke suatu kerangka acuan yang bergerak dengan laju 𝑣 ke kanan. Menurut mekanika klasik, massa 1 akan tampak diam, sedangkan massa 2 akan tampak bergerak dengan laju 2𝑣. Tetapi transformasi Lorentz ternyata memberi hasil yang berbeda. Misalkan 𝑂′ bergerak ke kanan dengan laju 𝑢 = 𝑣. Maka menurut 𝑂′, kecepatan massa 1 adalah: 𝑣′1 =



𝑣1− 𝑢 𝑣−𝑣 = =0 2 𝑣1 𝑢 1− ⁄𝑐 2 1 − 𝑣 ⁄ 2 𝑐



(Karena semua kecepatan searah sumbu 𝑥, maka kita telah dan akan mengabaikan indeks bawah 𝑥, dan kecepatan massa 2 adalah (dengan 𝑣2 = −2 menurut 𝑂) 𝑣′2 =



𝑣2− 𝑢 (−𝑣) − (𝑣) −2𝑣 = = 2 𝑣2 𝑢 𝑣 1− ⁄𝑐 2 1 − (−𝑣) ⁄𝑐 2 1 + 𝑣 ⁄ 2 𝑐



Kecepatan massa gabungan 2m adalah: 𝑣=



𝑣− 𝑢 0−𝑣 = = −𝑣 𝑣𝑢 1 − ⁄ 2 1 − 0 (𝑣)⁄ 𝑐 𝑐2 Sesudah



Sebelum V2



V



Menurut 𝑂, momentum linear sebelum dan setelah tumbukan adalah:



21



𝑃𝑎𝑤𝑎𝑙 = 𝑚1 𝑣1 + 𝑚2 𝑣2 = 𝑚𝑣 + 𝑚(−𝑣) = 0 𝑃𝑎𝑘ℎ𝑖𝑟 = (2𝑚)(𝑉) = 0 Menurut 𝑂′ 𝑃′𝑎𝑤𝑎𝑙 = 𝑚1 𝑣1 ′ + 𝑚2 𝑣2 ′ = 𝑚(0) + 𝑚 (



−2𝑣 −2𝑚𝑣 )= 2 2 1+𝑣 ⁄ 2 1+𝑣 ⁄ 2 𝑐 𝑐



𝑃′𝑎𝑘ℎ𝑖𝑟 = 2𝑚(−𝑉) = −2𝑚𝑣 Karena menurut pengukuran 𝑂′ , 𝑃′𝑎𝑤𝑎𝑙 ≠𝑃′𝑎𝑘ℎ𝑖𝑟 , maka bagi 𝑂′ momentum linear tidak kekal. Bagi besaran massa terdapat pula pertambahan massa relativistik, menurut hubungan berikut: 𝑚=



𝑚0



(2.10)



2



√1−𝑢 ⁄ 2 𝑐



𝑚0 disebut massa diam dan, seperti panjang sejati dan waktu sejati, ia diukur terhadap kerangka acuan yang terhadapnya benda diam. Dalam kerangka acuan lainnya, massa relativistik m akan lebih besar daripada 𝑚0 (perhatikan bagaimana konsep ini mengatasi masalah kita di depan tentang pemercepatan objek mencapai atau melalui laju cahaya. Ketika laju objek menghampiri laju cahaya, massanya menjadi besar sekali, sehingga gaya yang bekerja menjadi kurang efektif untuk menghasilkan suatu percepatan. Pada saat massanya menjadi tidak hingga, maka tidak ada lagi percepatan yang dapat dihasilkan oleh suatu gaya hingga, dangan demikian kita tidak pernah mencapai atau melampaui laju cahaya. Marilah kita periksa bagaimana definisi massa relativistik ini mempertahankan kekekalan momentum dalam kerangka acuan𝑂 dan 𝑂′. Nyatakan massa yang diukur oleh 𝑂 dengan 𝑚1 , 𝑚2 , dan 𝑀. Anggaplah kedua objek ini memiliki massa diam 𝑚0 yang sama, maka menurut 𝑂, kedua massa itu adalah 𝑚1 =



𝑚0 √1−



𝑣2



⁄ 2 𝑐



dan 𝑚2 =



𝑚0 2



√1−𝑣 ⁄ 2 𝑐



Karena 𝑣1 = 𝑣2 = 𝑣; juga 𝑀 = 𝑚1 + 𝑚2 =



2𝑚0 2



√1 − 𝑣 ⁄ 2 𝑐



Karena massa gabungan ini diam dalam kerangka acuan 𝑂, maka massa 𝑀 adalah massa diamnya, yang selanjutnya kita nytaakan dengan 𝑀0 . Menurut 𝑂′, 𝑚′1 diam, jadi 𝑚′1 = 𝑚0. Karena 𝑚′2 bergerak 2 dengan laju 𝑣′2 = −2𝑣/(1 + 𝑣 ⁄ 2 ), maka 𝑐



22



2



1+𝑣 ⁄ 2 𝑐 ) = 𝑚0 ( 2 1−𝑣 ⁄ 2 𝑐



𝑚0



𝑚′2 =



2



√1 − 1 ( −2𝑣 𝑐 1 + 𝑣 2⁄ ) 𝑐2 Massa gabungan 𝑀′ bergerak dengan laju 𝑉 ′ = −𝑣, jadi 𝑀′ =



𝑀0 2



√1 − 𝑣 ⁄ 2 𝑐



Jika kita substitusikan hasil yang kita peroleh bagi 𝑀0, yaitu 𝑀0 =



2𝑚0 ⁄ 2 √1 − 𝑣 ⁄ 2 𝑐



Tampak bahwa definisi massa yang baru ini berhasil mempertahankan kekekalan momentum menurut 𝑂, karena 𝑃𝑎𝑤𝑎𝑙 = 𝑚1 𝑣1 + 𝑚2 𝑣2 tetap sama dengan nol. Seperti 𝑃𝑎𝑘ℎ𝑖𝑟 . Selanjutnya, marilah kita periksa pernyataan momentum awal dan akhir dalam kerangka acuan 𝑂′ : 𝑃′𝑎𝑤𝑎𝑙 = 𝑚1 ′𝑣1 ′ + 𝑚2 ′𝑣2 ′ 2



= 𝑚0 (0) + 𝑚0 ( =



1+𝑣 ⁄ 2 2𝑣 𝑐 ) (− 𝑣 2 ) 2 1−𝑣 ⁄ 2 1+ ⁄ 2 𝑐 𝑐



−2𝑚0 𝑣 2



1−𝑣 ⁄ 2 𝑐



dan ′𝑉1 ′



𝑃′𝑎𝑘ℎ𝑖𝑟 = 𝑀1



=



2𝑚0 −2𝑚0 𝑣 [−𝑣] = 2 2 𝑣 1− ⁄ 2 1−𝑣 ⁄ 2 𝑐 𝑐



Karena 𝑃′𝑎𝑤𝑎𝑙 = 𝑃′𝑎𝑘ℎ𝑖𝑟 , maka definisi baru kita tentang massa relativistik di atas telah memungkinkan kita untuk mempertahankan berlakunya kekekalan momentum dalam kedua kerangka acuan. Dan ternyata definisi massa relativistik ini berhasil mempertahankan berlakunya kekekalan momentum dalam semua kerangka acuan, tidak hanya dalam kedua kerangka acuan khusus yang kita tinjau dalam contoh kasus ini. Selain mendefinisikan massa relativistik seperti yang kita lakukan di atas, kita dapat pula mendefinisikan ulang momentum relativistik berikut: 𝑝=



𝑚0 𝑣 2



√1−𝑣 ⁄ 2 𝑐



………………………………(2.11)



Definisi ini ternyata merupakan pilihan yang terbaik, karena beberapa alasan berikut: kita dapat memperluasnya dengan mudah ke rumusan dua dan tiga dimensi, dan juga definisi ini menghindarkan kita dari kebingungan penggunaan massa relativistik pada kasus-kasus dalam mana pernyataan ini tidak berlaku. Sebagi contoh, kita tinjau percobaan berikut. Dua massa 𝑚1 dan 𝑚2 yang berjarak pisah r saling tarik menarik dengan hukum gravitasi 𝐹 = 𝐺𝑚1 𝑚2 /𝑟 2 . Kedua massa ini dihubungkan oleh sebuah pegas berkala yang mencatat gaya antara keduanya. Pengamat 𝑂′ berada dalam sebuah pesawat 23



roket yang bergerak menjauhi kedua massa tersebut dalam arah tegak lurus garis hubung 𝑚1 𝑑𝑎𝑛 𝑚2 . Jika kita menyisipkan pernyataan massa relativistik ke dalam pernyataan klasik bagi gaya di atas, maka kita akan menyimpulkan bahwa 𝑂 dan 𝑂′ akan mengamati pembacaan yang berbeda pada skala pegas yang sama. Ini jelas tidak mungkin! Seperti yang akan kita perlihatkan nanti, sungguh keliru memperlakukan semua persamaan dinamika seperti yang kita lakukan di atas. Dengan sekedar menggantikan nama klasik.



Contoh soal Andaikanlah kita menyelubungi sebuah lampu pijar 100 watt dan catu dayanya (power supply) dalam sebuah wadah tembus cahaya yang digantungkan pada sebuah timbangan yang sangat peka. Hitunglah perubahan massa yang terjadi jika lampu pijar tersebut terus menyala dalam satu tahun Penyelesaian 100 watt = 100 J/s Karena 1 tahun ≅ 𝜋 x 107 s (suatu faktor yang bermanfaat yang perlu diingat ), maka dalm 1 tahun energy yang diradiasikan adalah sekitar 3 x 109 J. Maka ∆𝐸0 3 × 109 ∆𝑚0 = 2 = ≅ 3 × 10−8 𝑐 9 × 1016



1.7 KESERAMPAKAN DAN PARADOKS KEMBAR Dalam hal ini, kita akan meninjau dua akibat dari teori relativitas khusus. Yang pertama menyangkut pengertian keserampakan dan pensikronan jam. Keserampakan adalah keadaan atau perihal yang serempak. Sedangkan pensikronan berasal dari kata sinkron yang artinya sejalan, selaras, sesuai, atau terjadi pada waktu yang sama. Bagi sebagian besar di antara kita, masalah mensinkronkan jam bukanlah suatu proses yang sulit. Sebagai contoh, kita dapat menyetel jam kita dengan langsung melihat pada jam yang berada di dekat kita. Namun metode ini mengabaikan waktu yang dibutuhkan oleh cahaya dari jarum jam untuk merambat ke mata kita. Jika kita berada 1 m dari sebuah jam, maka arloji kita akan terlambat sekitar 3 ns ( 3× 10−9 s). keterlambatan waktu yang kecil ini tidak akan membuat kita terlambat dalam beraktivitas, namun bagi seorang ilmuan yang sedang melakukan eksperimen hal itu merupakan masalah serius. Karena pengukuran yang biasa adalah dengan pengukuran selang waktu yang lebih kecil 1 ns.



24



GAMBAR 2.13 Kilatan cahaya yang dipancarkan dari sebuah titik ditengah-tengah antara kedua jam, menghidupkan kedua jam ini secara serempak, menurut O. Pengamat O’ melihat jam 2 berdetak lebih dulu daripada jam 1. Di x= 0 dan x = L masing-masing terletak sebuah jam, sedangkan di x = L/2 terletak sebuah bola lampu kamera. Kedua jam ini dibuat sedemikian rupa sehingga mereka berdetak ketika mereka menerima kilatan cahaya. Karena rambatan cahaya membutuhkan waktu yang sama untuk mencapai kedua jam tersebut, maka keduanya akan berdetak secara bersamaan pada saat L/2 c sehingga kedua jam tersebut benar-benar tersinkronkan. Dalam kerangka acuan O, terjadi dua peristiwa, yaitu :  Penerimaan sebuah sinyal cahaya oleh jam 1 di 𝑥1 = 0 , 𝑡1 = L/2 c 𝐿



 Penerimaan oleh jam 2 di 𝑥2 = 𝐿 , 𝑡2 = 2 𝑐 Dengan menggunakan persamaan transformasi Lorentz, kita dapati bahwa o’ mengamati jam 1 menerima sinyal pada saat



𝑡







1



=



𝑢2 𝑡1 − ( 𝑐 )𝑥1 2 √1 − 𝑢2 𝑐



=



𝐿 2𝑐 2



√1 − 𝑢2 𝑐



Sedangkan jam 2 pada saat



𝑡′2 =



𝑢2 𝑡2 − ( 𝑐 )𝑥2 2



√1 − 𝑢2 𝑐



𝐿 𝑢 2𝑐 − (𝑐 2 ) 𝐿 = 2 √1 − 𝑢2 𝑐



Jadi, 𝑡 ′ 2 lebih kecil daripada 𝑡 ′ 1 sehingga jam 2 tampak menerima sinyal lebih dulu daripada jam 1. Karena itu, kedua jam tersebut berdetak pada dua saat yang berbeda dengan selang waktu sebesar ′



∆𝑡 = 𝑡







1



−𝑡







2



=



𝑢𝐿⁄𝑐 2 √1 − 𝑢2 ⁄𝑐 2



Menurut O’ . penting untuk dicamkan bahwa beda waktu ini bukanlah efek pemuluran waktu- karena pemuluran waktu dicirikan oleh suku pertama persamaan transformasi Lorentz bagi t’ , sedangkan keterlambatan pensikronan dicirikan oleh suku keduanya. O’ memang 25



mengamati kedua jam tersebut berjalan lambat , sebagai akibat efek pemuluran waktu ; O’ juga mengamati bahwa jam 2 berjalan sedikit lebih cepat daripada jam 1 . selang waktu ∆t’ yang diukur O’ antara saat kedua jam tersebut mulai berdetak, memberikan dengan menggunakan persamaan diatas , ∆t’ =



𝑢𝐿 𝑐2



bagi pembacaan jam 2 ketika O melihat jam 1 pada pembacaan 0



(nol). Oleh karena itu kita peroleh kesimpulan berikut : dua peristiwa yang terjadi serempak dalam satu kerangka acuan tidaklah serempak dalam kerangka acuan lain yang bergerak relative terhadap yang pertama, kecuali jika kedua peristiwa itu terjadi pada tempat yang sama. (dalam contoh kita tadi, jika L = 0, sehingga kedua jam terletak pada titik yang sama dalam ruang , maka keduanya akan sinkron dalam semua kerangka acuan). Jadi, jam-jam yang sinkron dalam satu kerangka acuan tidaklah perlu tetap sinkron dalam kerangka acuan lain yang dalam keadaan gerak relative. Tinjau dua orang saudara yang bermukim di bumi . andaikanlah slah satunya, katakanlah yang bernama casper, tetap berdiam di bumi , sedangkan saudara kembar perempuannya, Amelia, melakukan perjalanan antariksa dengan sebuah pesawat roket menuju suatu planet jauh. Casper , yang memahami teori relativitas khusus, mengetahui bahwa jam saudari kembarnya akan berjalan lambat relative terhadap jam miliknya. Karena itu, Amelia akan lebih muda daripadanya ketika ia tiba kembali di bumi : ini tidak lain adalah apa yang tersirat dari bahasan kita tentang efek pemuluran waktu. Namun, dengan mengingat kembali bahasan tadi, kita ketahui bahwa bagi dua pengamat yang bergerak relative, masing-masing berpendapat bahwa jam saudara kembarnya yang berjalan lambat. jadi, maslah ini dapat pula kita pelajari dari sudut pandang Amelia, yang berpendapat bahwa casper dan bumilah (bersama dengan system tata surya dan galaksi ) yang melakukan perjalanan pulang-pergi menjauhinya dan kemudian kembali lagi. Dalam keadaan seperti itu, Amelia akan berpendapat bahwa jam saudara kembarnya (yang sekarang bergerak relative terhadapnya) yang berjalan lambat, sehingga bagi Amelia saudara kembarnya casper yang lebih muda daripadanya ketika mereka bertemu kembali. Memang mungkin saja timbul ketidaksepahaman tentang jam siapakah yang berjalan lambat terhadap jam milik masing-masing saudara kembar ini, namun ini hanyalah masalah pemilihan kerangka acuan belaka ; ketika Amelia tiba kembali di bumi (atau ketika bumi kembali di amerika ) semua pengamat haruslah sependapat tentang siapakah diantara kedua saudara kembarnya itu yang usianya lebih muda. Inilah paradoksnya masing-masing saudara kembar itu memperkirakan bahwa yang lainnya yang lebih muda. 26



Pemecahan bagi paradoks ini terletak pada peninjauan kita yang tidak simetris terhadap peran kedua saudara kembar itu. Hukum-hukum relativitas khusus hanya berlaku bagi kerangka lembam yang bergerak relative terhadap kerangka lainnya dengan kecepatan tetap. Kita dapat memasok roket Amelia dengan dorongan yang cukup kuat sehingga Amelia dan roketnya mengalami percepatan untuk suatu selang waktu yang singkat,sehingga pesawatnya mencapai suatu laju tetap yang meluncurkannya menuju planet tujuannya, jadi, selama perjalanan Amelia ke planet tujuannya , hapir seluruh waktunya ia habiskan dalam suatu kerangka acuan yang bergerak pada kecepatan tetap terhadap casper. Tetapi, untuk kembali ke bumi, ia harus memperlambat dan membalikkan pesawatnya. Meskipun gerak ini juga dilakukan dalam selang waktu yang sangat singkat, perjalanan kembali Amelia berlangsung dalam suatu kerangka acuan yang berbeda dari kerangka pada perjalanan perginya. “Loncatan” Amelia dari suatu kerangka acuan ke yang lainnyalah. Yang menyebabkan usia kedua saudara kembar ini tidak simetri. Hanya Amelia yang harus “meloncat” ke suatu kerangka acuan baru agar dapat kembali, dan karena itu semua pengamat akan sependapat bahwa Amelia-lah yang “sebenarnya’ bergerak, sehingga dengan demikian jam miliknya yang “sebenarnya” berjalan lambat; oleh Karena itu, Amelia-lah yang lebih muda ketika ia tiba kembali di bumi. Kita menganggap bahwa percepatan dan perlambatan berlangsung dalam selang waktu yang sangat singkat, sehingga seluruh usia Amelia terhitung selama perjalannya saja. Untuk menyederhanakan , kita akan menganggap bahwa planet jauh tersebut diam terhadap bumi; pilihan ini tidak mempengaruhi kesimpulan persoalannya, tetapi sekedar mengabaikan perlunya diperkenalkan kerangka acuan lain. Andaikan planet itu berjarak 12 tahun cahaya dari bumi, dan bahwa Amelia bergerak dengan laju 0,6c. maka menurut casper, saudarinya membutuhkan waktu 20 tahun (20 tahun× 0,6𝑐 = 12 𝑡𝑎ℎ𝑢𝑛 𝑐𝑎ℎ𝑎𝑦𝑎) untuk mencapai planet itu dan 20 tahun lagi untuk tiba kembali di bumi, dan oleh karena itu saudarinya berpergian untuk total waktu 40 tahun. (tetapi, casper tidak akan dapat mengetahui apakah saudari kembarnya telah tiba di planet itu sampai sinyal cahaya yang membawa berita tentang ketibaannya di sana mencapai bumi. Karena cahaya membutuhkan waktu 12 tahun untuk menempuh jarak bumi-planet, maka barulah 2 tahun kemudian setelah keberangkatan Amelia, casper ‘melihat” saudarinya tiba di planet itu. Delapan tahun kemudian ia kembali di bumi). Dari kerangka acuan Amelia pada roket, jaraknya ke planet menyusut dengan faktor sebesar √1 − (0,6)2 = 0,8, dan karena itu jarak ini adalah 0,8× 12 = 9,6 𝑡𝑎ℎ𝑢𝑛 𝑐𝑎ℎ𝑎𝑦𝑎. Pada laju 0,6c ini, Amelia akan mengukur lama waktu 16 tahun bagi perjalanannya menuju planet tujuannya, sehingga dengan demikian ia membutuhkkan total waktu 32 tahun bagi perjalanan 27



pergi-pulangnya. Jadi, casper berusia 40 tahun, sedangkan Amelia hanya berusia 32 tahun, dan memang benar bahwa Amelia-lah yang lebih muda setelah kembali di bumi. Kita dapat mempertegas analisis ini dengan meminta casper setiap tahun mengirimkan suatu sinyak cahaya, pada saat ia berulang tahun, kepada saudari kembarnya. Kita ketahui bahwa frekuensi sinyal yang diterima Amelia akan mengalami pergeseran Doppler. Selama perjalanan pergi, Amelia akan menerima sinyal 𝑢



𝑢



1



tersebut



pada laju (frekuensi



terima) (1/th)



×



0,4



√(1 − 𝑐 )/(1 + 𝑐 ) = ( 𝑡 ) √1,6 = 0,5/th , sedangkan untuk perjalanan balik , laju sinyal yang 𝑢



𝑢



diterimanya adalah (1/th) × √(1 − 𝑐 )/(1 + 𝑐 ) atau 2/tahun. Jadi, untuk 16 tahun pertama, selama perjalanan Amelia menuju planet, ia akan menerima 8 sinyal, sedangkan selama 16 tahun perjalanan pulangnya ia akan menerima 32 sinyal, jadi total 40 buah sinyal. Empat puluh sinyal yang diterimanya ini menunjukkan bahwa saudara kembarnya telah merayakan 40 kali pesta ulang tahun selama 32 tahun kepergiannya. 1.8 UJI PERCOBAAN TEORI RELATIVITAS KHUSUS 1. Ketidakberadaan Eter Sebelumnya kita telah membahas percobaan Michelson-Morley yang berkaitan dengan relativitas khusus. Namun kemudian tidak ditemukan bukti nyata mengenai laju cahaya terhadap arah meskipun telah dilakukan berkali-kali percobaan dengan berbagai variasi dan perbaikan kepekaan yang terus ditingkatkan. 2. Pemuluran Waktu Efek pemuluran waktu telah dibahas sebelumnya pada pemuluran muon oleh sinar kosmik, contoh lainnya yaitu peluruhan partikel berkecepatan tinggi yang dapat diteliti di labolatorium, misalnya meson pi yang memiliki usia hidup 26 x 10-9 s (26 ns) yang mana selang waktunya sangat serasi dngan prcobaan labolatorium sehingga proses tumbukan antar partikel dapat dikendalikan agar ia berhenti sebelum meluruh yang memungkinkan untuk mengukur usia hidup sejatinya. Pengukuran usia hidup meson pi yang bergerak dengan laju 𝑣⁄𝑐 = 0,913 memberi hasil 63,7 ns dalam kerangka acuan labolatorium. Usia ini jelas lebih lama dari usia hidup sejatinya karena pemuluran waktu dalam kerangka acuan meson pi yang bergerak. Jadi efek peuluran waktu terbukti kebenarannya. 3. Massa dan energy Relativitas Setiap melakukan eksperimen nuklir atau partikel, seorang fisikawan hamper pasti melakukan uji langsun dan tak langsung terhadap hubungan massa-energi teori relativitas khusus. Bukti langsung kebenaran ramalan teori relativitas khusus diperoleh beberapa 28



tahun setelah diterbitkannya makalah Einstein pada tahun 1905. Pertambahan massa karena bertambahnya kecepatan diuji dengan mengukur momentum dan kecepatan elektron berenergi tinggi yang dipancarkan dalam beberapa proses peluruhan radioaktif tertentu. Dari grafik tampak kecocokan dengan pertambahan massanya. Sebagai contoh, kita tinjau atom deuterium (hidrogen berat) yang terdiri dari atom hydrogen biasa dengan tambahan sebuah neutron pada intinya. Jumlah massa atom hidrogen dan neutron pada keadaan diam: mH + mn = (1, 67356 x 10-27 kg) + (1, 67496 x 10-27 kg) = 3,34852 x 10-27 kg



Jika massa deuterium diukur secara langsung hasilnya: mD = 3,34455 x 10-27 kg Sehingga massa seluruh inti atom lebih kecil dari pada massa partikel penyusunnya dengan Δm = 0,00397 x 10-27 kg (sekitar empat kali massa elektron). Ini setara dengan energi ΔE = (Δm)c² = 2,23 MeV, yang dikenal sebagai energi ikat deuterium. Untuk memisahkan atom deuterium menjadi atom hidrogen dan sebuah neutron memerlukan energy sebesar 2,23 MeV yang dalam proses pemisahan inti ini terubahkan menjadi massa. Pengubahan massa menjadi energi atau lebih tepatnya energi massa menjadi energy kinetik (begitupun sebaliknya) menjadi tidak asing lagi bagi para fisikawan. Contoh lain pengubahan energi menjadi massa adalah penciptaan messon pi. Dalam keadaan normal, messon pi yang massa diamnya sekitar 140 MeV (sekitar 274 kali massa elektron) tidak terdapat dialam, tetapi diciptakan pada akselerator energi-tinggi, yaitu dalam tumbukan antara partikel-partikel biasa seperti proton, sebagaimana yang diperlihatkan pada diagram berikut.



------



Dalam proses ini, energi kinetik proton sekitar 140 MeV diubah menjadi energi massa meson pi. 29



4. Ketidakubahan Laju Cahaya Jika laju cahaya bergantung pada gerak pengamat, maka dapa dinyatakan c’=c+ku, dimana c adalah laju cahaya dalam kerangka diam sumber, c’ laju cahaya diukur dalam kerangka acuan yang bergerak dan u laju relative terhadap kedua krangka acuan. Variabel k adalah bilangan yang ditentukan olh eksperimen menurut relativitas khusus k adalah O, sedangkan menurut relativitas Galileo k=1. Salah satu prcobaan yaitu yang bertujuan mempelajari pemancaran sinar X yang berdenyut cepat sambil mengorbit pada system bintang ganda. Jika laju cahaya berubah ketika gerak orbitnya bergerak mnuju dan menjauhi bumi maka awal dan akhir gerhana terjadi dalam selang waktu berbeda. Dimana efek ini tidak teramati sehimgga disimpulkan bahwa k x2 ) 𝑑



2



𝑋12 = 𝐷2 + ( 2 + 𝑦𝑛 ) 𝑑



2



𝑋22 = 𝐷2 + ( 2 − 𝑦𝑛 )



(3.13)



Dengan mengurangkan, diperoleh 𝑋12 − 𝑋22 = 2 𝑦𝑛 𝑑



(3.14)



dan Gambar 3.3 Geometri percobaan dua celah.



𝑦𝑛 =



(x1 + x2 ) (x1 − x2 ) 2𝑑



(3.15)



Dalam percobaan-percobaan dengan gelombang cahaya,



D berorde 1 m, dan 𝑦𝑛 serta d paling tinggi 1 mm; jadi X1= D, dan X2 = D, seningga x1 + x2 = 2D, dan dalam hampiran ini D



𝑦𝑛 = (x1 − x2 ) d



(3.16)



Dengan menggunakan Persamaan (3.11) bagi nilai (x1 − x2 ) pada maksimum interferensi, kitaa peroleh 𝑦𝑛 = n



D



(3.17)



d



37



Piranti lain untuk mengamati interferensi gelombang cahaya adalah kisi difraksi, yaitu piranti penghalang bercelah banyak untuk menghasilkan interferensi gelombang cahaya. Pada gambar 3.4 dilukiskan cara kerja piranti ini; maksimummaksimum interferensi berkaitan dengan panjang gelombang berbeda yang muncul pada sudut 𝜃 yang berbeda, menurut hubungan d sin 𝜃 = n 



(3.18)



d adalah jarak antar celah dan n bilangan urutan maksimum-maksimum interferensi (n = 1, 2, 3, . . . ). Keuntungan



kisi



difraksi



terletak



pada



keunggulan resolusinya-yaang memungkinkan kita memperoleh pemisahan sempurna atas panjang-panjang gelombang yang berdekatan. Jadi piranti ini sangat bermanfaat untuk mengukur panjang gelombang. Tetapi, perlu diperhatikan bahwa unttuk memperoleh nilai berarti dari sudut 𝜃, misalnya sin 𝜃 dalam rentang 0,3 hingga 0,5, d haruslah dalam orde beberapa kali panjang gelombang. Untuk cahaya tampak, hal ini tidak sulit diwujidkan, tetapi untuk radiasi dengan panjang gelombang yang sangat pendek, tidaklah mungkin membuat kisi dengan nilai d sekecil itu. Sebagai contoh, bagi sinar X dengan orde panjang gelombang 0,1 nm kit Gambar 3.4 Penggunaan kisi difraksi untuk memilah cahaya ke dalam panjang gelombang penyusunnya.



perlu membuat kisi, yang jaarak antar celahnya lebih kecil daripada 1 nm (kira-kira seperti jarak antar atom paada sebagian besaar zat). Pemecahan bagi masalah di atas telah



diketahui sejak percobaan rintisan Laue dan Bragg-yaang menggunakan atom-atom itu sendiri sebagai suatu kisi difraksi! Suatu berkas sinar-X “melihat” jarak teratur atomatom dalam sebuah kristal sebagai semacam kisi difraksi tiga dimensi. Tijau susunan atom yaang diperlihatkan Gambar 3.5, yang mewakili sebagai kecil irisan duaa dimenssi kristal.



38



Gambar 3.5 Suatu berkas sinar-X dipantulkan dari sehimpunan bidang kristal yang berjarak antara d. Berkas yang dipantulkan dari bidang kedua menempuh jarak 2d sin 𝜃 lebih panjang dari pada berkas yang dipantulakn dari bidang pertama.



Masing-masing atom ini akan memantulkan sinar-X yang dijatuhkan pada kristal pada semua arah, namun hanya pada satu arah tertentu, ‘riak gelombang’ yang dihamburkan akan berinterferensi secara maksimum sehingga menghaasilkan suaatu berkas terpantul. Dalam hal ini, pantulannya dapaat kita pandang terjadi poda suatu bidang yng digambaarkan melalui barisan atom-atom tersebut. (Keadaan ini mirip dengan pemantulan cahaya dari sebuah cerminberkas cahaya yang terpantulkan hanya terdapat pada satu arah tetentu, dan pada arah itulah berkas yaang terpaantulkan dapat kita pandang terjadi pada sebuah bidang dengan sudut datang sama dengan sudut pantul). Andaikanlah barisan-barisan atom dalam kristaal berjarak d satu sama lain; maka masing-masing bidang barisan atom ini, yang terdepan, kedua, dan seterusnya, akan memantulkan sebagian berkas sinar-X yang dijatuhkan pada kristal. Muka gelombang yang dipantulkan oleh bidang terdepan, karena gelombang yang dipantulkan bidang kedua harus menempuh suatu jarak tambahan 2d sin 𝜃, dimana 𝜃 adalah sudut datang yang diukur terhadap permukaan kristal (dalam orde optika, sudut dataang selalu kita ukur terhadap normal permukaan pantul). Jika beda jalan ini sama dengan kelipatan bulat dari panjang gelombang, maka berkas-berkas yang terpantulkan itu akan berinterferensi secara konstruktif sehingga memberikan suatu intensitas maksimum; jadi pernyataan mendasar bagi maksimum interferensi dalam difraksi sinar-X dari sebuah kristal adalah d sin 𝜃 = n 



n = 1, 2, 3, . . .



(3.19)



Hasil ini dikenal sebagai hukum Bragg bagi difraksi sinar-X. Perhatikan bahwa faktor 2 muncul dalam persamaan (3.19), sedangkan dalam pernyataan serupa dari persamaan (3.18) bagi kisi difraksi biasa, faktor ini tidak muncul.



39



CONTOH SOAL 3.1



Sebuah kristal tunggal garam dapur (NaCl) diradiasi dengan seberkas sinar-X dengan panjang gelombang 0,250 nm, dan dipantulkan Bragg yang pertama teramati pada sudut 26,3 °. Berapakah jarak antar atom bagi NaCl?



Pemecahan Dengan memecahkan hukum Bragg bagi jaarak d, kita peroleh n



0,250 𝑛𝑚



d = 2 sin 𝜃 = 2 sin(26,3 °) d = 0,282 nm



Pilihan kita bagi bidang pantul dalam Gambar 3.5 di atas adalah sebarang-kaena tak ada paatokan baagi kita untuk menentukan pilihan himpunan atom yang mana guna menggambarkan bidang-bidang pantul yang melaluinya. Gambar 3.6 memperlihatkan suatu irisan kristal yang lebih besar. Seperti yang anda lihat, ada banyak bidang pantul yang mungkin untuk dipilih, masing-masing dengan nilai 𝜃 dan d yang berbeda. (Tentu saja, karena di dan 𝜃i berkaitan, maka masing-masing tidak dapat diubah secara bebas). Jika berkas sinar-X yang kita gunakan berpanjang gelombang tunggal, maka agak sulit bagi kita untuk mendapatkan sudut dan himpunan bidang paantul yang tepat guna mengamati interfrensi. Tetapi jika kita menggunakan berkas sinar-X dengan rentang panjang gelombang yang kontinu, maka untuk tiap-tiap di dan 𝜃i , interferensinya akan terjadi bagi suatu panjang gelombang tetentu, sehingga akan terjadi suatu pola maksimum interfrensi pada sudut-sudut pantul yag berbeda seperti yang diperlihatkan pada gambar 3.6. Pola maksimum interferensi tidaklah bergantung pada panjang gelombang berkas sinar-X yang datang (yang distribusinya kontinu), melainkan pada jarak dan susunan atom dalam kristal.



40



GAMBAR 3.6 Seberkas sinar-X dapat dipantulkan dari bidang-bidang kristal yang berbeda.



Gambar 3.7 dan 3.8 memperlihatkan cuplikan pola-pola difraksi (yang disebut pola Lauce) yang diperoleh dari hamburan sinar-X dari dua kristal yang berbeda. Titiktitik terang berkaitan dengan maksimum-maksimum interferensi bagi semua panjang gelombang dari rentang panjang gelombang dari rentang panjang gelombang sinar-X datang yang kebetulan memenuhim persamaan (3.19). tentu saja, pola tiga dimensinya lebih rumit dari pada gambaaran dua dimensi yang disajikan, tetaapi titik-titik dari masing-masing panjang gelombang mempunyai tafsiran yang sama. Gambar 3.9 memperlihatkan pola yang diperoleh dari suatu cuplikan yang terdiri atas sejumlah kristal. (Tampaknya seperti diperoleh dengan memutarkan Gambar 3.7 dan 3.8 secara cepat terhadap pusatnya). Dari gambar-gambar seperti itu dapatlah direka struktur dan jarak kisi kristal yang bersangkutan.



Sinar-X datang (berpanjang gelombang kontinu)



41



GAMBAR 3.7 (Atas) Peraalatan untuk mengamati hamburan sinar-X oleh sebuah kristal. Suatu maaksimum interferensi (titik) muncul pada pelat foto apabila sehimpunan bidang kristal memenuhi persyaratan Bragg untuk suatu panjang gelombang tertentu. (Bawah) Pola Lauce dari kristal NaCl. (Sumber: Amoros, Beuger, dan de Amoros, The Lauce Method, Academic press, 1975).



GAMBAR 3.8 Pola Lauce daari sebuah kristal kuartz. Struktur dan jarak bidang-bidang kristal Kuartz yang berbeda dari NaCl yang mengakibatkan pola ini tampak berbeda dari pola Gambar 3.7.



GAMBAR 3.9 (Atas) Peralatan untuk mengamati hamburan sinar-X dari suatu cuplikan berbentuk bubuk. Karena kristal-kristal yang begitu banyaknya dalam sebutir bubuk memiliki semua kemungkinan orientasi, maka tiap-tiap sinar-X yang terhambur pada Gambar 3.7 menjadi sebuah kerucut yang membentuk pola lingakaran pada pelat foto. (Bawah) Pola difraksi (dikenal sebagai pola Debye-Scherrer) dari sebuah cuplikan berbentuk bubuk. (Sumber: Eisberg & Resnick, Quantum Physics, John Wiley & Sons, 1974).



Semua contoh yang telah kita bahas dalam paragraf ini bergantung pada sifat gelombang radiasi elektromagnet. Sekarang akan kita bahas beberapa contoh percobaan 42



dengan cahaya dan radiasi elektromagnet lainnya, yang menunjukkan bahwa tafsiran gelombangnya bukanlah merupakan deskripsi lengkap dari sifat-sifat radiasi elektromagnet.



2.2 RADIASI BENDA HITAM Pertanda pertama yang menunjukkan bahwa gambaran gelombang klasik tentang radiasi elektromagnet (yang berhasil baik menerangkan percobaan Young dan Hertz pada abad kesembilan belas dan yang dapat dianalisis secara tepat dengan persamaan Maxwell) tidak seluruhnya benar, tersimpulkan dari kegagalan teori gelombang untuk menerangkan spektrum radiasi termal yang diamati-jenis radiasi elektromagnet yang dipancarkan berbagai benda semata-mata karenba suhunya. Teori gelombang juga ternyata gagal menjelaskan hasil percobaan lain yang segera menyusul, seperti percobaan yang mempelajari pemancaran elektron dari permukaan logam yang disinari cahaya (efek foto listrik), dan hamburan cahaya oleh Gambar 3.10 Pengukuran spektrum radiasi termal. Piranti prisma digunakan untuk memisahkan berbagai panjang gelombang yang dipanaskan benda.



elektron-elektron (efek Compton). Disini kita hanya akan membahas radiasi termal; ssedangkan kedua percobaan yang terakhir disebutkan tadi akan kita bahas dalam kedua pasal berikutnya. Susunan percobaan khasnya diperlihatkan pada



Gambar 3.10. Sebuah obyek dipertahankan bersuhu T1. Radiasi yang dipancarkan obyek ini kemudian diamati dengan suatu peralatan yang peka terhadap panjang gelombang radiasi. Sebagai contoh, zat perantara dispersif (penyebar cahaya) seperti prisma dapat digunakan untuk pengamatan ini karena panjang gelombnag berbeda yang menembusinya akan teramati pada sudut 𝜃 yang berbeda pula. Dengan menggerakkan detektor radiasi ke sudut 𝜃 yang berbeda-beda, kita dapat mengukur intensitas radiasi pada suatu panjang gelombang tertentu. Karena detektor merupakan bukanlah suatu titik geometris (akan sangat tidak efektif) tetapi mengapit suatu selang sudut d 𝜃 yang sempit,



43



maka yang sebenarnya kita ukur adalah jumlah radiasi dalam selang d 𝜃 pada 𝜃, atau yang setara dengan ini, dalam selang d pada . Besaran ini kita sebut intensitas radiaant (radiant intensity) R, sehingga hasil percobaannya adalah deretan nilai R 𝑑 sebanyak nilai  berbeda yang kita pilih untuk di ukur. Apabila setelah selesai, kita mencoba untuk merajah data ini sebagai fungsi dari , maka hasilnya akan tampak mirip seperti Gambar 3.11. Bila percobaannya kemudian kita ulangi tetapi dengan menaikkan suhu T2 menjadi lebih tinggi, maka kita akan memperoleh hasil seperti yang tampak pada gambar. Dengan mengulangi percobaan ini berkali-kali, maka kita simpulakan dua sifat penting radiasi termal



GAMBAR 3.11 Hasil pengamatan intensitas radiant berikut: yang mungkin terhadap 1. Intensitas radaint total terhadap seluruh panjang berbagai panjang gelombang. gelombang berbanding lurus suhu T berpangkat Masing-masing suhu benda pemancar radiasi memberikan empat; karena intensitas total tak lain adalah luas puncak 𝑚𝑎𝑘𝑠 yang berbeda daerah di bawah kurva-kurva intensitas radiant pada pula.



Gambar 3.11, maka kita dapat menulis: ∞



∫0 𝑅 𝑑 = 𝜎𝑇 4



(3.20)



dimana telah kita perkenalkan tetapan banding 𝛼. Persamaan (3.20) ini disebut hukum Stefan dan 𝛼 dikenal sebagai tetapan Stefan-Boltzman. Dari sejumlah percobaan seperti yang dilukiskan pada Gambar 3.11, nilai tetapan 𝛼 didapati sebesar: 𝜎 = 5,6703 x 10-8 W/m2. K4 Panjang



gelombang



dimana



masing-masing



kurva



mencapai



nilai



maksimumnya, yang kita sebut 𝑚𝑎𝑘𝑠 (walaupun ia bukanlah suatu panjang gelombang maksimum), menurun jika suhu pemancar dinaikkan, ternyata sebanding dengan kebalikan suhu, sehingga 𝑚𝑎𝑘𝑠 𝛼1/T. Dari percobaan didapati bahwa nilai tetapan bandingnya adalah: 𝑚𝑎𝑘𝑠 𝑇 = 2,898 𝑥 10−3 𝑚. 𝐾



(3.21)



Hasil ini dikenal sebagai hukum pergeseran Wien; istilah “pergeseran” merujuk ke kenyataan bahwa puncak kurva intensitas bergeser jika suhu diubah.



44



CONTOH SOAL 3.2



(a) Pada panjang gelombang berapakah sebuah benda pada suhu ruang (T = 20℃) memancarkan radiasi termal maksimum? (b) Hingga suhu berapakah benda tersebut harus kita panaskan agar puncak radiasi termalnya berada pada daerah spektrum merah? (c) Berapa kali banyaknya radiasi termal yang dipanaskan benda tersebut pada suhu yang tertinggi?



Pemecahan 1.



Dengan mengubah ke suhu mutlak, T = 293 K, maka dari hukum pergeseran Wien, kita peroleh



𝑚𝑎𝑘𝑠 = 2.



2,898 𝑥 10−3 𝑚. 𝐾 = 9,89 𝜇𝑚 293 𝐾



Dengan mengambil panjang gelombang cahaya merah adalah  ≅ 650 nm, maka dengan menggunakan kembali hukum pergeseran Wien untuk mendapati T, kita peroleh:



2,898 𝑥 10−3 𝑚. 𝐾 𝑇= = 4460 𝐾 650 𝑥 10−9 𝑚 3.



Karena intensitas radiasi total berbanding lurus terhadap 𝑇 4 , maka perbandingan radiasi termal total adalah 𝑇21 (4460)4 = = 5,37 𝑥 104 3 4 (293) 𝑇1



Pada tahap ini kita akan mencoba untuk menganilis dan memahami hasil-hasil ini (ketergantungan R pada  , hukum Steven, dan hukum wien) berdasarkan termodinamika dan elekromag. Bahasakan lengkap tentang teori ini tidak akan kita sajikan di sini, sebagai gantinya kita hanya akan meninjau uraian garis besar ringkasnya. Kita dapat melihat berbagai benda karena cahaya yang mereka pantulkan. Pada suhu ruang, radiasi termal ini paling banyak terdapat dalam daerah spektrum inframerah (𝑚𝑎𝑘𝑠 ≅ 10 𝜇𝑚), pada daerah mata kita tak lagi peka. Bila benda benda tersebut kita panasi, mereka akan mulai memancarkan cahaya tampak; menurut persamaan (3.21), bila 45



T bertambah, maka 𝑚𝑎𝑘𝑠 menurun, dan untuk suhu sedang, 𝑚𝑎𝑘𝑠 akan menurun ke daerah cahaya tampak. Sebagai ci=ontoh, sepotong logam yang dipanaskan, mula-mula tampak memijar dengan memancarkan warna merah tua, dan bila suhunya terus dinaikkan warnanya berangsur berubah menjadi semakin kuning. Sayangnya, radiasi yang dipancarkan benda biasa tak hanya bergantung pada suhu, tetapi juga pada sifat-sifat lainnya, seperti rupa benda, sifat permukaannya, bahan pembuatnya. Radiasinya juga bergantung pada apakah ia memantulkan atau tidak memantulkan radiasi dari lingkaran sekitar yang jatuh padanya. Untuk menghilangakn beberapa hambatan ini, kita tidak akan meninjau benda biasa, melainkan yang permukaannya sama sekali hitam (benda hitam). Jika sebuah benda sama sekali hitam, maka cahaya yang jatuh padanya tidak ada yang ia pantulkan sehingga sifat-sifat permukaannya dengan demikian tidak dapat diaamati. Namun demikian, perluasan ini masih belum cukup menyederhanakan persoalan untuk memungkinkan menghitung spektrum radiasi yang terpancarkan. Karena itu, kita memperluasnya lebih lanjut ke suatu jenis benda hitam istimewa- sebuah rongga, misalnya bagian dalam dari sebuah kotak logam, dengan sebuah lubang kecil pada salah satu dindingnya. Lubang itulah, bukan kontaknya, yang berperan sebagai benda hitam. Radiasi dari luar yang menembusi lubang ini akan lenyap pada bagian dalam kotak dan kecil kemungkinan untuk keluar kembali dari lubang tersebut; jadi tidak ada pantulan yang terjadi pada benda hitam (lubang) tersebut. Karena radiasi yang keluar dari lubang itu merupakan cuplikan radiasi di dalam kotak, maka pemahaman tentang hakikat radiasi di dalam kotak akan memungkinkan kita untuk memahami radiasi yang keluar melewati lubang kotak itu. Perhitungan klasik bagi energi radiant yang dipancarkan untuk tiap-tiap panjang gelombang



sekarang



berbagi



menjadi



beberapa



tahap



perhitungan.



Tanpa



memperlihatkan pembuktiannya, berikut dikemukakan bagian-bagian penting dari penurunannya. Pertama, yang menyangkut perhitungan jumlah radiasi (jumlah gelombang) untuk masing-masing panjang gelomabang, kemudian sumbangan tiap-tiap gelombang bagi energi total dalam kotak, dan terakhir intensitas radiant yang berkaitan dengan energi itu. 1.



Kotak berisi gelombang-gelombang berdiri elektromagnet. Jika semua dinding kotak adalah logam, maka radiasi dipantulkan bolak-balik dengan simpul (node) medan elektrik terdapat pada tiap-tiap dinding (medan listrik haruslah nol di dalam sebuah konduktor). Ini sama seperti persyaratan yang berlaku bagi gelombang berdiri lain, seperti yang terjadi pada tali tegang atau kolom udara di dalam sebuah pipa organ. 46



2.



Jumlah gelombang berdiri dengan panjang gelombang antara  dan  + 𝑑 adalah N() 𝑑 =



8𝜋𝑉



4



𝑑



(3.22)



V adalah volume kotak. Untuk gelombang berdiri satu dimensi, seperti pada tali tegang sepanjang L, maka panjang gelombang yang diperkenankan adalah  = 2𝐿/𝑛, (n = 1, 2, 3, . . . ). Jumlah gelombang berdiri yang mungkin dengan panjang gelombang antara



1 dan 2 adalah n2 – n1 = 2L (1/2 - 1/1 ), sehingga dalam selang antara  dan  + 𝑑 akan terdapat sebanyak N() d = (2L/2) d gelombang yang berbeda. Perluasan hasil ini untuk gelombang elektromagnet tiga dimensi, memberikan Persamaan (3.22). 3.



Tiap-tiap gelombnag memberi saham energi KT bagi radiasi di dalam kotak. Hasil ini diperoleh dari termodinamika klasik. Radiasi dalam kotak berada dalam keadaan kesetimbangan termal dengan dinding pada suhu T. Radiasi ini terpantulkan oleh dinding kotak karena ia diserap dinding dan kemudian dipancarkan dengan segera oleh atom-atom dinding, yang dalam proses ini bergetar pada frekuensi radiasi. pada suhu T, energi kinetik rata-rata sebuah atom yang bergetar adalah ½ KT (seperti yang diperoleh bagi suatu gas ideal). Untuk suatu osialtor harmonik sederhana, energi kinetik rata-ratanya sama dengan energi potensial rata-rata, sehingga energi total rta-ratanya dalah KT.



4.



Untuk memperoleh intensitas radiant dari kerapatan energi (energi per satuan volume), kalikan dengan c/4. Hasil ini juga diperoleh dari teori elektromagnet dan termodinamika klasik. Dengan menggabungkan unsur-unsur di atas, maka intensitas radiant yang kita perkirakan adalah: Intensitas radiant = (jumlah gelombang per satuan volume) x (energi per gelombang) x (energi radiant per rapat energi)



R () =



8𝜋



4



𝐶



KT 4



(3.23)



Hasil ini dikenal sebagai rumus Rayleigh-Jeans. Penurunannya menggunakan teori klasik elektromagnet dan termodinamika, yang merupakan usaha maksimal kita dalam menerapkan fisika klasik untuk memahami persoalan radiasi benda hitam. Pada Gambar 3.12, diperlihatkan perbandingan hasil perhitungan intensitas radiant dengan menggunakan hukum Rayleigh-Jeans terhadap data hasil percobaan yang kita bahas di depan.



47



GAMBAR 3.12 Kegagalan rumus RayleighJeans klasik menghasilkan kurva ramalan yang cocok dengan spektrum pengamatan intensitas radiant. Pada daerah panjang gelombang yang panjang, ramalan teori klasik tampak menghampiri data pengamatan, tetapi pad daerah panjang gelombang pendek, rumus klasik ternyata sama sekali gagal.



Intensitas radiant yang dihitung dengan Persamaan (3.23) tampak menghampiri data percobaan untuk daerah panjang gelombang yang panjang, tetapi pada daerah panjang gelombang pendek, teori klasik ternyata sama sekali gagal. Kegagalan hukum Rayleigh-Jeans pada daerah panjang gelombang pendek ini dikenal sebagai bencana ultraviolet (ultra-violet catastrophe), yang memperlihatkan suatu permasalahan serius yang dihadapi fisika klasik, mengingat teori elektromagnet dan termodinamika, yang mendasari hukum Rayleigh-Jeans, telah di uji secara seksama dalam berbagai percobaan dan didapati cocok dengan hasil pengamatan percobaan. Untuk kasus radiasi benda hitam ini, sehingga diperlukan suatu teori fisika yang baru. Fisika baru memberi tafsiran benar terhdap radiasi termal ini dikemukakan oleh fisikawan Jerman. Max Planck. Bencana ultraviolet disebabkan karena intensitas radiant yang diramalkan hukum Rayleigh-Jeans menjadi sangat besar pada daerah panjang gelombang pendek (atau pada frekuensi yang tinggi). Yang diperlukan adalah suatu cara untuk membuat R



0 bila 



0 atau v



∞. Menurut nalar Plank, radiasi yang



terpantul dari dinding rongga logam berasal dari radiasi yang diserap dan kemudian dipancarkan kembali dengan segera oleh atom-atom dinding rongga; selama selang waktu radiasi. karena energi suatu sistem yang bergetar bergantung pada frekuensinya, maka Planck mencoba menemukan suatu cara untuk memperkecil jumlah gelombang berdiri berfrekuensi tinggi dengan memperkecil jumlah osilator berfrekuensi tinggi dalam dinding rongga. Ia melakukan ini dengan mengemukakan suatu anggapan berani yang kemudian menjadi landasan bagi teori fisika baru, fisika kuantum. Berkaitan dengan teori ini dirumuskan suatu ragam mekanika baru, yang dikenal sebagai mekanika kuantum.



48



Plank mengemukakan bahwa suatu atom yang bergetar hanya dapat menyerap atau memancarkan energi kembali dalam bentuk buntelan-buntelan energi (yang disebut kuanta). Jika energi kuanta berbanding lurus dengan frekuensi radiasi, maka bila frekuensinya meningkat, energinya akan turut pula menjadi besar; tetapi karena tidak satu pun gelombang yang dapat memiliki energi melebihi kT, maka tidak ada gelombang berdiri yang energi kuantumnya lebih besar dari pada kT. Ini secara efektif membatasi intensitas radiant frekuensi-tinggi (panjang gelombang pendek), dan dengan demikian memecahkan persoalan bencana ultraviolet. Max Planck (1858-1947, warga Jerman). Karyanya dalam bidang distribusi spektrum radiasi, yang membuka jalan ke teori kuantum, dihargai dengan penganugerahan hadiah Nobel tahun 1918. Dalam tahun-tahun terakhirnya, ia banyak menulis tentang agama dan filsafat. (kelak akan kita pelajari dari fisika kuantum bahwa semua besaran terhitung dinyatakan dalam rata-rata atau probabilitas. Jadi, lebih tepat dikatakan bahwa sumbangan Planck adalah menemukan cara membuat sekecil mungkin probabilitas satu modus (mode) getaran berenergi lebih besar daripada kT). Dalam reori Planck, setiap osilator dapat memaancarkan atau menyerap energi hanya dalam jumlah yang merupakan kelipatan bulat dari suatu energi dasar 𝜀, E = n𝜀



n = 1, 2,3, . . .



(3.24)



n menyatakan jumlah kuanta. Selanjutnya energi setiap kuanta ini ditentukan oleh frekuensi menurut 𝜀=ℎ𝑣



(3.25)



h adalah suatu tetapan banding, yang sekarang dikenal sebagai tetapan Planck. Berdasarkan tanggapan ini, spektrum intensitas radiant yang dihitung Planck adalah 𝑐



8𝜋



ℎ𝑐



1



𝑅 () = (4) ( 4 ) [(  ) 𝑒 ℎ𝑐/𝐾𝑇 − 1]



(3.26)



(Hasil ini akan diturunkan dalam Pasal 12.6). Kesesuaian antara percobaan dan rumus Planck diilustrasikan pada Gambar 3.13, yang memperlihatkan betapa baiknya kurva rumus Planck berimpit dengan data pengamatan. Perhatikan bahwa anggapan Planck tentang kediskretan energi getaran memberi hasil yang menggantikan energi rata-rata KT, yang muncul dalam persamaan (3.23), dengan besaran dalam tanda kurung persegi pada persamaan (3.26). Karena kita ketahui bahwa hukum Rayleigh-Jeans, yaitu persamaan (3.23), cukup baik pada daerah panjang 49



gelombang panjang, maka kita perkirakan bahwa untuk 𝜆 besar, persamaan (3.26) akan member kembali persamaan (3.23). Penurunan hukum Stefan dari rumus Planck memberikan hubungan tetapan Stefan-Boltzman dan tetapan Planck berikut: 2𝜋 5 𝑘 4



𝜎 = 15𝜃2 ℎ2



(3.27)



Karena kita mengetahui 𝜎 dari percobaan, maka kita dapat menentukan nilai tetapan Planck dari hubungan ini, dan hasilnya adalah: ℎ = 6.626 × 10−34 𝐽. 𝑠



2.3 EFEK FOTOELEKTRIK Pada fotoelektrik permukaan sebuah logam disinari dengan seberkas cahaya dan sejumlah electron terpancar dari permukaannya. Dalam studi eksperimental terhadap fotoelektrik, kita mengukur bagaimana laju dan energy kinetik electron yang terpancar bergantung pada intensitas dan panjang gelombang sumber cahaya. Percobaan ini harus dilakukan dalam ruang hampa, agar electron tidak kehilangan energinya karena bertumbukan dengan molekul-molekul udara. Susunan percobaan ini diperlihatkan pada gambar 3.14. Laju pancaran electron diukur sebagai arus listrik pada rangkaian luar dengan menggunakan ammeter, sedangkan energy kinetiknya



ditentukan dengan mengenakan suatu potensial



perlambatan (retarding potential) pada anoda sehingga electron tidak memiliki energy yang cukup untuk “memanjati” bukti potensial yang terpasang. Gambar 3.14 Peralatan untuk mengamati efek fotoelektrik. Cahaya yang menyinari permukaan logam (katoda) menyebabkan electron terpental keluar. Ketika electron bergerak menuju anoda, pada rangkaian luar terjadi arus elektrik yang diukur dengan Ammeter A.



Secara eksperimen, tegangan perlambatan terus diperbesar hingga pembacaan arus pada ammeter menurun ke nol. Tegangan yang bersangkutan ini disebut potensial henti (stopping potential) 𝑉𝑠 . Karena electron yang berenergi tertinggi tidak dapat melewati potensial henti ini, maka pengukuran 𝑉𝑠 merupakan suatu cara untuk menentukan energy kinetic maksimum electron 𝐾𝑚𝑎𝑘𝑠: 𝐾𝑚𝑎𝑘𝑠 = 𝑒𝑉𝑠 50



(3.28)



e adalah muatan electron. Nilai khas 𝑉𝑠 adalah dalam orde beberapa volt. Dari berbagai percobaan seperti ini, kita pelajari fakta-fakta terinci efek fotoelektrik berikut: 1. Laju pemancaran electron bergantung pada intensitas cahaya. 2. Laju pemancaran electron tak bergantung pada panjang gelombang cahaya dibawah suatu panjang gelombang tertentu; diatas nilai itu arus secara berangsur-angsur menurun hingga menjadi nol pada suatu panjang gelombang pancung (cutoff wavelength) 𝜆𝑐 . Panjang gelombang 𝜆𝑐 ini biasanya terdapat pada spectrum daerah biru dan ultraviolet. 3. Nilai 𝜆𝑐 tidak bergantung pada intensitas sumber cahaya, tetapi hanya bergantung pada jenis logam yang digunakan sebagai permukaan fotosensitif. Di bawah 𝜆𝑐 , sebarang sumber cahaya, selemah apapun, akan menyebabkan pemancaran terjadinya pemancaran fotoelektron; diatas 𝜆𝑐 , tidak satupun cahaya, sekuat apapun, dapat menyebabkan terjadinya pemancaran fotoelectron. 4. Energy kinetic maksimum electron yang dipancarkan tidak bergantung pada intensitas cahaya, tetapi hanyalah bergantung pada panjang gelombangnya; energy kinetic ini didapati bertambah secara linear terhadap frekuensi sumber cahaya. 5. Apabila sumber cahaya dinyatakan, arus segera akan mengalir (dalam selang waktu 10−9 𝑠. Menurut teori gelombang cahaya, sebuah atom akan menyerap energy dari gelombang electromagnet dating yang sebanding dengan luasnya yang menghadap ke gelombang datang. Dan sebagai tanggapan terhadap medan elektrik gelombang, electron atom akan bergetar, hingga tercapai cukup energy untuk melepaskan sebuah electron dari ikatan atomnya. Penambahan kecemerlangan sumber cahaya, memperbesar laju penyerapan energy, karena medan elektriknya bertambah, sehingga laju pemnacaran electron juga akan bertambah, yang sesuai dengan hasil pengamatan percobaan. Tetapi penyerapan ini terjadi pada semua panjang gelombang, sehingga keberadaan panjang gelombang pancung sama sekali bertentangan dengan gambaran gelombang cahaya. Pada panjang gelombang yang lebih besar dari pada 𝜆𝑐 pun, teori gelombang mengatakan bahwa seharusnya masih mungkin bagi suatu gelombang elektromagnetik memberikan energy yang cukup guna melepaskan electron. Kita dapat menafsirkan secara kasar waktu yang diperlukan sebuah atom untuk menyerap energy secukupnya guna melepaskan sebuah electron. Sebagai sumber cahaya



51



kita pilih sebuah laser berintensitas sedang, seperti laser helium-neon. Kelauaaran daya yang dikeluarkan dari laser ini paling tinggi 10−3 𝑊, yang penampang bekasnya terbatasi pada luas sekitar beberapa millimeter persegi (10−5 𝑚2 ). Diameter khas atom adalah dalam orde 10−10 𝑚, jadi luasnya dalam orde 10−20 𝑚2 . Karena itu fraksi intensitas sinar laser yang jatuh pada atom adalah sekitar 1020 𝑚2 /10−5 𝑚2 ≅ 10−15. Jadi, hanya 10−18 𝑊=10−18J/s≅ 6 𝑒𝑉/𝑠 daya yang dapat diserap atom, dan untuk menyerap energy sebanyak beberapa eV diperlukan waktu sekitar satu detik. Dengan demikian menurut teori gelombang cahaya, kita memperkirakan tidak akan melihat fotoelektron terpancar hingga beberapa detik setelah sumber cahaya dinyalakan. Dalam praktek kita dapati bahwa berdasarkan fotoelektrik bertama dipancarkan dalam selang waktu 10−9 𝑠. Dengan demikian, teori gelombang cahaya gagal meramalkan keberadaan panjang gelombang pancung dan waktu tunda yang teramati dalam percobaan. Teori efek fotoelektrik yang benar barulah dikemukakan Einstein pada tahun 1905. Teorinya ini didasarkan pada gagasan Planck tentang kuantum energy, tetapi ia mengembangkannya satu langah lebih kedepan. Einstein menganggap bahwa kuantum energy bukanlah sifat istimewa dari atom-atom dinding rongga radiator, tetapi merupakan sifat radiasi itu sendiri. Energy radiasi elektromagnetik bukannya diserap dalam bentuk aliran kontinu gelombang, melainkan dalam bentuk foton. Sebuah foton adalah suatu kuantum energy elektromagnetik yang diserap atau dipancarkan, dan sejalan dengan usulan Planck. Tiap-tiap foton dari radiasi berfrekuensi 𝑣 memiliki energy: 𝐸 = ℎ𝑣



(3.29)



h adalah tetapan planck. Dengan demikian tetapan foton-foton berfrekuensi tinggi memiliki energy yang lebih besar- energy foton cahaya biru lebih besar daripada energy foton cahaya merah. Maka foton haruslah memiliki momentum, dan sejalan dengan rumusan klasik, momentum sebuah atom berenergi E adalah: 𝑝=



𝐸



(3.30)



𝑐



Dalam foton harislah berlaku bahwa 𝑚0 = 0 bagi sebuah foton. Dengan demikian foton akan berperilaku sebagai sebuah “partikel” tanpa massa diam. Tentu saja, Einstein menganggapnya benar pada awak teorinya; teori relativitas khusus tidak memperkenankan kita “menyusuli” sebuah berkas cahaya, karena itu gerk foton tidak pernah dapat dihentikan. Massa bagi foton haruslah 0 bagi sebuah foton atau sebarang partikel yang bergerak dengan laju cahaya; karena jika tidak demikian, energy 𝑚𝑐 2 akan menjadi tak hingga. 52



Dengan menggabungkan persamaan (3.29) dan (3.30) kita dapati hubungan langsung berikut antara panjang gelombang dan momentum foton: ℎ



𝑝=𝜆



(3.31)



Teori Enstein segera terbukuti dapat menjelaskan semua fakta efek fotoelektrik yang diamati. Andaikanlah kita menganggap bahwa sebuah electron terikat dalam logam dengan energy 𝑊, yang dikenal sebagai fungsi kerja (work function). Logam yang berbeda memiliki fungsi kerja yang berbeda pula; salah satu contoh daftarnya diperlihatkan pada Tabel 3.1. Tabel 3.1 Beberapa fungsi kerja fotoelektrik



Bahan 𝑾(𝒆𝑽)



Bahan 𝑾(𝒆𝑽)



Na



2.28



Al



4.08



Co



3.90



Cu



4.70



Zn



4.31



Ag



4.73



Pt



6.35



Pb



4.14



Untuk mengeluarkan sebah electron dari permukaan suatu logam, kita harus memasok energy sekurang-kurangnya sebesar 𝑊. Jika ℎ𝑣 < 𝑊, tidak terjadi efek fotoelektrik; jika ℎ𝑣 > 𝑊, maka electron akan terpental keluar dan kelebihan energy yang dipasok berubah menjadi energy kinetiknya. Energy kinetic maksimum 𝐾𝑚𝑎𝑘𝑠 yang dimiliki electron yang terpental keluar dari permukaan logam adalah: 𝐾𝑚𝑎𝑘𝑠 = ℎ𝑣 − 𝑊



53



(3.32)



Untuk electron yang berada jauh dibawah permukaan logam, dibutuhkan energy yang lebih besar daripada 𝑊 dan beberapa diantaranya keluar dengan energy kinetic yang lebih rendah. Sebuah foton yang memasok energy sebesar 𝑊, adalah tepat sama dengan energy yang dibutuhkan untuk melepaskan sebuah electron, berkaitan dengan cahaya yang panjang gelombangnya sama dengan panjang gelombang pancung 𝜆𝑐 .



Pada



panjang gelombang ini, tidak ada kelebihan energy yang tersisa bagi energy fotoelektron, sehingga persamaan (3.32) tersederhana menjadi: 𝑊 = ℎ𝑣 =



ℎ𝑐



(3.33)



𝜆𝑐



Dan dengan demikian: 𝜆𝑐 =



ℎ𝑐



(3.34)



𝑊



Karena kita memperoleh satu fotoelektron untuk setiap foton yang terserap, maka penaikkan intensitas sumber cahaya akan berakibat semakin banyak fotoelektron yang dipancarkan, namun demikian semua fotoelektron ini akan memiliki energy kinetic yang sama, karena semua foton memiliki energy yang sama. Terakhir, waktu tunda sebelum terjadi pemancaran fotoelektron diperkirakan singkat-begitu foton pertama diserap, arus fotoelektrik akan mulai mengalir. Jadi, semua fakta eksperimen efek fotoelektrik sesuai dengan perilaku kuantum radiasi elektromagnetik. Robert Millikan memberikan bukti yang lebih meyakinkan tentang kesesuaian ini dalam serangkaian percobaan yang dilakukan pada tahun 1915. Salah satu cuplikan hasil percobaannya diperlihatkan pada gambar 3.15. Dari kemiringan garisnya, yang tidak lain adalah rajahan persamaan (3.32), diperoleh tetapan Planck: ℎ = 6.57 × 10−34 𝐽. 𝑠 Gambar 3.15 Hasil percobaan Milikan untuk efek fotoelektrik pada natrium. Kemiringan garisnya ℎ



adalah ; penentuan eksperimental dari kemiringan 𝑒



ini memberikan suatu cara untuk menentukan tetapan Planck. Perpotongannya dengan sumbu datar memberikan frekuensi pancung; tetapi pada saat milikan melakukan percobaanya, potensial kontak elektroda-elektroda tidak diketahui secara tepat sehingga skala vertical tergeser beberapa persepuluh volt. Kemiringan kurva tak terpengaruh oleh koreksi ini.



54



Nilai ini sangat sesuai dengan nilai yang diturnkan dari pengukuran tetapan Stefan-Boltzmann, seperti pada persamaan (3.27). Kesesuaian yang baik ini, yang diturnkan dari dua percobaan berbeda, yang satu melibatkan penyerapan dan yang lainnya pemnacaran radiasi electromagnet, memperlihatkan bahwa tetapan Planck mempunyai arti penting lebih daripada sekedar untuk menerangkan satu percobaan. Dewasa ini tetapan planck dipandang sebagai salah satu tetapan alam, dan telah diukur dengan ketelitian yang sangat tinggi dalam berbagai percobaan. Nilai yang sekarang diterima adalah ℎ = 6.62618 × 10−34 𝐽. 𝑠



55



CONTOH SOAL 3.3 (a) Berapakah energy dan momentum sebuah foton



cahaya merah yang



berpanjang gelombang 650 nm? (b) Berapakah panjang gelombang sebuah foton yang berenergi 2.40 eV? Penyelesaian: (a) Biasanya kita ingin mengetahui hubungan antara panjang gelombang cahaya dan energy fotonnya. Oleh karena itu, marilah kita menuliskan kembali 𝑐



persamaan (3.29), dengan mengingat bahwa untuk cahaya, 𝑣 = 𝜆: 𝐸 = ℎ𝑣 = 𝐸=



ℎ𝑐 𝜆



(6.63 × 10−34 𝐽. 𝑠)(300 × 108 𝑚/𝑠) (650 × 10−9 𝑚)



𝐸 = 3.06 × 10−19 𝐽 Dengan mengubahnya kedalam satuan elketron-volt, kita peroleh: 𝐸=



3.06 × 10−9 𝐽 = 1.91 𝑒𝑉 1.60 × 10−19 𝐽/𝑒𝑉



Persoalan semacam ini akan menjadi sederhana jika kita nyatakan gabungan



ℎ𝑐 dalam satuan 𝑒𝑉. 𝑛𝑚, seperti: 𝐸=



ℎ𝑐 1240 𝑒𝑉. 𝑛𝑚 = = 1.91 𝑒𝑉 𝜆 650 𝑛𝑚



Mometumnya didapati dengan cara yang sama: 𝑝=



ℎ 1 ℎ𝑐 1 1240 𝑒𝑉. 𝑛𝑚 𝑒𝑉 = = = 1.91 𝜆 𝑐 𝜆 𝑐 650 𝑛𝑚 𝑐



Momentum ini dapat pula diperoleh langsung dari energy: 𝑝= (b) 𝝀 =



𝐸 1.91 𝑒𝑉 = = 1.91 𝑒𝑉/𝑐 𝑐 𝑐 ℎ𝑐 𝐸



=



1240 𝑒𝑉.𝑛𝑚 2.40 𝑒𝑉



= 517 𝑛𝑚



56



CONTOH SOAL 3.4



Fungsi kerja bagi logam tungsten adalah 4.52 eV. (a) Berapakah panjang gelombang pancung 𝜆 bagi tungsten? (b) Berapakah energy kinetic maksimum elektron- elektron yang dipancarkan apabila digunakan radiasi dengan panjang gelombang 200.0 nm? (c) Berapakah potensial henti untuk kasus ini? Penyelesaian: (a) 𝜆𝑐 =



ℎ𝑐 𝑊



=



1240 𝑒𝑉 𝑛𝑚 4.52 𝑛𝑚



(b) 𝐾𝑚𝑎𝑘𝑠 = ℎ𝑣 − 𝑊 = =



= 274 𝑛𝑚



ℎ𝑐 𝜆



−𝑊



1240 𝑒𝑉. 𝑛𝑚 − 4.52 𝑒𝑉 200 𝑛𝑚



= 1.68 𝑒𝑉 (c) 𝑉 =



𝐾𝑚𝑎𝑘𝑠 𝑒



=



1.68 𝑒𝑉 𝑒



= 1.68 𝑉



2.4 EFEK COMPTON Cara lain radiasi berinteraksi dengan atom adalah melalui efek Compton, dimana radiasi dihamburkan oleh elektron hampir bebas yang terikat lemah pada atomnya. Interprestasi gelombang memprediksi bahwa ketika terjadi radiasi elektromagnetik dari sebuah partikel bermuatan, maka radiasi yang dipancarkan tersebut akan memiliki frekuensi yang sama dengan radiasi yang datang dari segala penjuru. Pada tahun 1922, Arthur H. Compton menunjukkan bahwa jika interprestasi kuantum dari radiasi elektromagnetik diterima, maka radiasi yang dihamburkan akan memiliki frekuensi yang lebih kecil dari pada radiasi datang dan juga bergantung pada sudut hamburnya. Analisa Compton, sebagai akibatnya menyertakan tampilan hamburan radiasi elektromagnetik dari partikel bermuatan sebagai peristiwa tumbukan sempurna antar foton dan partikel bermuatan bebas. Pada keadaan awal foton memiliki energy 𝐸 yang diberikan oleh 𝐸 = ℎ𝑣 = ℎ𝑐/𝜆



(3.35)



57



Dan momentumnya adalah 𝑝=



𝐸



(3.36)



𝑐



Elektron dalam keadaan diam, memiliki energi diam m0c2. Setelah hamburan foton E’ dan momentum p’ dan bergerak pada arah yang membentuk sudut θ terhadap arah foton datang. Elektron memiliki energi total Ee dan pe dan begerak pada arah membentuk sudut terhadap ϕ foton datang. Dalam interaksi ini berlaku persyaratan kekekalan energi dan momentum, yakni: Eawal = Eakhir E+m0c2= E’+ Ee (px)awal = (px)akhir p = pe cos ϕ + p’ cos θ Gambar 3.16 Geometri Hamburan foton



(py))awal = (py)akhir 0 = pe sin ϕ + p’ sin θ



sudut ϕ dihilangkan dengan menggabungkan persamaan-persamaan momentum : pe cosϕ = p –p’ cos θ pe sin ϕ = p’ sin θ kuadratkan dan kemudian jumlakan, menjadi , pe2 = p2 – 2pp’ cos θ + p’2 dengan menggunakan hubugan relativistik antara energi dan momentum Ee2= c2pe2 + me2c4 Maka dengan menyisipkan Eedan pe , kita peroleh (E + me2 – E’)2= c2(p2 – 2pp’cos θ + p’2)+me2c4 Kita dapati 1 1 1 − = (1 − 𝑐𝑜𝑠 θ) 𝐸′ 𝐸 𝑚𝑒 𝑐 2 Sehingga ℎ



Δλ= λ’ –λ = 𝑚 𝑐 (1 − 𝑐𝑜𝑠 θ) 𝑒



Λ adalah panjang gelombang foton datang dan λ’ adalah panjang gelombang foton hambur. Besaran h/mec dikenal sebagai panjang gelombang compton dari elektron yang memiliki nilai 0,002426 nm; namun perlu diingat bahwa ini bukanlah suatu panjang



58



gelombang dalam arti sebenarnya, melainkan semata-mata suatu perubahan panjang gelombang. Peragaan eksperimen pertama dari jenis hamburan ini dilakukan oleh Arthur compton pada tahun 1923. Diagram percobaan ini diperlihatkan pada gambar dibawah.



Gambar 3.17 Diagram skematis peralatan hamburan Compton. Panjang gelombang 𝜆 sinar X hamburan diukur dengan detector yang dapat digerakkan keberbagai kedudukan 𝜃. Beda panjang gelombang 𝜆′ − 𝜆 berubah terhadap 𝜃



Pada



percobaan



ini



seberkas



sinar-X



ditembakkan pada sebuah sasaran hambur, yang oleh Compton dipilih unsur karbon. Hasil percobaan asli Compton ini diperlihatkan pada gambar disamping. Pada setiap sudut, muncul dua buah puncak, yang berkaitan dengan foton-foton sinar-X hambur dengan dua energi atau panjang gelombang yang berbeda. Panjang gelombang dari salah satu puncak ini tidak berubah terhadap perubahan sudut; puncak ini berkaitan dengan hamburan foton sinar-X oleh elektron-elektron “terdalam” yang trikat erat pada atom. Karena eratnya ikatan elektron ini pada atom,



Gambar 3.18 hasil-hasil percobaan Compton bagi hamburan sinar X



maka foton yang terhambur oleh elektron ini tidak mengalami kehilangan energi. Akan tetapi panjang gelombang puncak yang lain sangat bergantung pada perubahan sudut, dan perubahan panjang gelombang ini tepat sesuai dengan yang diramalkan rumus Compton.



3.



59



2.5 PROSES FOTON LAINNYA 1. Bremsstrahlung dan Produksi sinar-X Apabila sebuah muatan elektrik, misalnya elektron, dipercepat atau diperlambat, maka ia memancarkan energi elektromagnetik. Andaikan kita mempunyai seberkas elektron, yang telah mencapai energi eV menumbuk suatu sasaran, elektronnya diperlambat sehingga pada akhirnya berhenti, karena bertumbukan dengan atom-atom materi sasaran dan elektron itu memancarkan foton. Jika energi kinetik lektron sebelum tumbukan adalah K, dan setelah tumbukan adalah K’, maka energi foton adalah hv = K – K’ elektron biasanya akan melakukan banyak tumbukan sebelum diam, maka ia juga akan memancarkan foton yang berbeda-beda pula; yang berkaitan dengan kehilangan energi yang kecil. Oleh karena itu, panjang gelombang terpendek yang dipancarkan ditentukan oleh energi yang telah dihilangkan selama tumbukan. hv = K ℎ𝑐 λ𝑚𝑖𝑛



= eV



λ𝑚𝑖𝑛 =



ℎ𝑐 𝑒𝑉



2. Produksi Pasangan Proses lain yang dapat terjadi apabila foton menumbuk atom adalah produksi pasangan, dimana seluruh energi foton akan hilang dan dua partikel terciptakan, yakni sebuah elektron dan sebuah positron. Positron adalah sebuah partikel yang massanya sama dengan elektron namun bermuatan positif. Energi foton yang hilang dalam proses ini berubah menjadi energi relativistik positron E+ dan elektron E-. hv = E+ + E= (𝑚𝑒 𝑐 2 +K+) + (𝑚𝑒 𝑐 2 +K-) Karena K+ dan K-selalu positif, maka foton terkecil adalah 2𝑚𝑒 𝑐 2 = 1,02 MeV agar proses ini dapat terjadi. Dapat dilambangkan dengan foton  elektron + positron Jika proses ini dibalik menjadi elektron + positron  foton biasa disebut dengan pemusnahan positron.



60



2.6 APAKAH FOTON ITU? 1. Pengertian Foton Foton adalah partikel cahaya yang terjadi karena radiasi tampak, biasanya karena suhu tinggi. Foton memiliki beberapa sifat, yaitu: 1. Tidak memiliki massa diam 2. Bergerak dengan laju cahaya 3. Memenuhi hubungan E= ℎ𝑣, 𝑝 = ℎ⁄𝜆 , 𝑑𝑎𝑛 𝐸 = 𝑝𝑐 4. Mengalami gaya gravitasi seperti partikel-partikel lain.



Secara matematis, foton sama halnya dengan partikel elementer seperti elektron namun berupa “titik –titik”. Sedangkan dalam sudut pandang fisika, foton tidak memiliki ukuran fisik dan mereka tidak dapat dibelah karena tidak memiliki unsurunsur penyusunnya. 2. Hakikat Foton Pembahasan mengenai hakikat foton dapat didekati dengan pendekatan paradoks. Paradoks fisika yang paling terkenal adalah dualisme gelombang-partikel. Ya, foton memiliki keduanya. Beberapa percobaan yang menyangkut efek interderensi dan difraksi, memperlihatkan bahwa radiasi elektromagnet berinteraksi seperti gelombang, sedangkan sejumlah percobaan lain memperlihatkan bahwa radiasi elektromagnet berinteraksi seperti kuantum partikel yaitu foton. Jika gambaran gelombang dan partikel kita anggap valid, tapi merupakan kemungkinan yang tidak setara, maka kita harus menganggap bahwa cahaya yang dipancarkan sebuah sumber cahaya hanya dapat merambat sebagai gelombang atau sebagai partikel. Andaiknlah kita menempatkan seperangkat alat celah-ganda pada salah satu sumber dan sel fotoelektrik pada sisi lain. Maka cahaya yang dipancarkan menuju celah-ganda tersebut berperilaku sebagai gelombang, sedangkan yang menuju sel elektrik berperilaku sebagai partikel. Jadi, kita seolah terjebak dalam suatu kesimpulan yang harus diterima, yaitu foton adalah partikel dan juga gelombang, dan hanya akan menunjukkan salah satu aspeknya bergantung pada percobaan yang kita lakukan. Dilema sifat rangkap gelombang-partikel ini tidak dapat dijelaskan dengan suatu penjelasan yang sederhana. Yang dapat kita lakukan adalah menyatakan bahwa



61



keduanya diperlukan untuk memberikan suatu deskripsi lengkap mengenai gejala fisika,dan memang kedua sifat ini bersifat saling melengkapi.



Latihan Soal 1. Hitung frekuensi foton yang dihasilkan ketika sebuah electron 20 keV menjadi diam akibat tumbukan dengan sebuah nucleus berat. 2. Carilah panjang gelombang maksimum foton yang melepaskan sebuah molekul dengan energy ikat 15 MeV. 3. Tentukanlah panjang gelombang dan frekuensi foton 1,0 keV. 4. Sebuah stasiun radio beroperasi pada frekuensi 103,7 MHz dengan keluaran daya sebesar 200 kW. Tentukanlah laju emisi kuanta dari stasiun radio tersebut. 5. Tinjaulah sebuah permukaan potassium yang berjarak 75 cm dari sebuah bola lampu 100 watt. Anggaplah bahwa energy yang diradiasikan oleh bola lampu tersebut 5% dari daya masukannya. Perlakukanlah setiap atom potassium bagaikan piringan berdiameter 1Å, tentukanlah waktu yang dibutuhkan setiap atom tersebut untuk menyerap sejumlah energy yang setara dengan fungsi kerja sebesar 2,0eV, mengikuti interprestasi gelombang cahaya. 6. Ketika sebuah eksperimen fotolistrik dilakukan menggunakan kalsium sebgai emitor, dijumpailah potensial-potensial penahan sebagai berikut ini: 𝜆, Å



2536



3132



3650



4047



𝑣, 𝐻𝑧 × 1015



1,18



0,985



0,822



0,741



𝑣2 , 𝑉



1,95



0,98



0,50



0,14



Tentukan konstanta Planck dari data tersebut. 7. Energy kinetic fotoelektrik berada dalam rentang nol hingga 4 × 10−19 𝐽 ketika cahaya dengan panjang gelombang 3000Å jatuh disuatu permukaan. Berapakah potensial penahan untuk cahaya itu? 8. Emitor didalam tabung fotolistrik memiliki panjang gelombang ambang 6000Å. Tentukanlah panjang gelombang cahaya yang dating ketabung tersebut jika potensial penahan untuk cahaya ini adalah 2,5 V.



62



9. Sebuah foton sinar X 0,3 MeV bertumbkan dengan sebuah electron yang diam. Dengan menggunakan kekekalan energy dan momentum, tentukanlah kecepatan lompatan electron tersebut. 10. Sebuah sinar X dengan panjang gelombang 0,300Å membuat hamburan Compton 60°. Carilah panjang gelombang foton terhambur tersebut dan energy electron setelah terjadinya hamburan. 11. Dalam eksperimen Compton, sebuah electron memperoleh energy kinetic sebesar 0,100 MeV ketika sebuah sinar X berenergi 0,500 MeV menerpanya. Tentukanlah panjang gelombang foton terhamburnya jika electron tersebut pada awalnya berada dalam keadaan diam. 12. Jika energy maksimum yang diberikan pada sebuah electron dalam hamburan Compton adalah 45 keV, berapakah panjang gelombang foton datangnya? 13. Tunjukkan bahwa electron bebas yang diam tidak dapat menyerap foton. (oleh karenanya hamburan Compton harus berlangsung dengan electron-elektron bebas.) 14. Bila logam natrium disinari dengan cahaya berpanjang gelombang 4,20 × 102 nm, maka potensial hentinya didapati sebesar 0,65V ; bila panjang gelombangnya diubah menjadi 3,10 × 102 𝑛𝑚, maka potensial hentinya menjadi 1,9 V. dengan hanya menggunakan data ini dan nilai laju cahaya, serta muatan electron, carilah fungsi kerja logam natrium dan nilai tetapan Planck. 15. Anggaplah suatu sumber cahaya 100 W melepaskan seluruh energinya dalam bentuk cahaya tampak, dengan rata-rata panjang gelombang foton sekitar 550 nm. Berapa banyak foton per detik yang menumbuk selembar kertas berukuran 20cm x 30 cm yang terletak sejauh 1m sumber cahaya tersebut?



63



3 Sifat Gelombang Dari Partikel



Pada



1924,



mengemukakan



Louis



de



bahwa



Broglie radiasi



elektromagnetik kadang-kadang dapat berperilaku sebagai gelombang dan pada saat lainnya sebagai partikel, maka mungkin obyek materi, seperti electron,



pada



saat-saat



bertindak sebagai gelombang.



64



tertentu



SIFAT GELOMBANG DARI PARTIKEL 3.1 Hipotesis de Broglie Pada 1924, Louis de Broglie mengemukakan bahwa radiasi elektromagnetik kadangkadang dapat berperilaku sebagai gelombang dan pada saat lainnya sebagai partikel, maka mungkin obyek materi, seperti electron, pada saat-saat tertentu bertindak sebagai gelombang. Dengan perkataan lain, de Broglie mengemukakan bahwa jika obyek-obyek materi melewati suatu celah yang lebarnya sekitar panjang gelombangnya, maka materi itu akan mengalami difraksi seperti yang diperlihatkan foton-foton pada eksperimen celah tunggal. Untuk sebuah foton, berlaku v = E/h dan 𝜆 = h/p. tampak bahwa ruas kiri persamaanpersamaan ini menyangkut aspek gelombang foton ( frekuensi, panjang gelombang), sedangkan pada ruas kanan muncul aspek partikelnya (energy,momentum). Jembatan antara kedua ruas persamaan ini adalah tetapan Plank. Berpegang pada kesetangkupan (simetri) alam, de Broglie menerka bahwa panjang gelombang yang berkaitan dengan materi akan memenuhi hubungan serupa yang berlaku untuk foton. Oleh karena itu, ia mempostulatkan bahwa sebuah obyek materi akan memiliki panjang gelombang yang di berikan oleh 𝜆=



ℎ 𝑝



=



ℎ 𝑚𝑣



Terdapat suatu perbedaan penting antara foton dan obyek bermassa dalam hal sifat-sifat gelombang dan partikel yang berkaitan. Karena 𝜆𝑣 = c bagi foton , maka hanya ada satu aturan yang diperlukan untuk memperoleh panjang gelombang dan frekuensi dari sifat partikel foton, yakni energy dan momentum. Sebuah obyek bermassa, dipihak lain, memerlukan aturan-aturan yang berbeda untuk panjang gelombangnya (𝜆=h/p) dan frekuensinya (v=E/h). Untuk menguji hipotesis yang dilakukan oleh Louise de Broglie pada tahun 1927, Davisson dan Germer di Amerika Serikat dan G.P. Thomson di Inggris secara bebas meyakinkan hipotesis Louise de Broglie dengan menunjukkan berkas elektron yang terdifraksi bila berkas ini terhambur oleh kisi atom yang teratur dari suatu kristal. Davisson dan Germer melakukan suatu eksperimen dengan menembakkan elektron berenergi rendah yang telah diketahui tingkat energinya kemudian ditembakkan pada atom dari nikel yang 65



diletakkan dalam ruang hampa. Berdasarkan hasil pengamatan Davisson dan Germer terhadap elektron-elektron yang terhambur ternyata dapat menunjukkan adanya gejala interferensi dan difraksi. Dengan demikian hipotesis de Broglie yang menyatakan partikel dapat berkelakuan sebagai gelombang adalah benar. Contoh Soal : 1. Tentukan panjang gelombang de Broglie sebuah butiran 0,01 kg yang memiliki kecepatan 10 m/det. Diketahui : m = 0,01 kg v = 10 m/s h = 6,63 x 10-34 J.s Ditanya



: Panjang gelombang de Broglie (𝜆dB) ?



Jawab



: 𝜆= =



ℎ 𝑚𝑣 (6,63 x 10−34 J.s) (0,01 𝑘𝑔)(10𝑚/𝑠)



= 6,63 x 10-33 m



2. Hitung panjang gelombang de Broglie dari neutron 0,05 eV. Diketahui : K=0,05 eV Ditanya : Panjang gelombang de Broglie (𝜆dB) ? Jawab



:𝜆= = =



=



ℎ 𝑝 ℎ √2𝑚0 𝐾 ℎ𝑐 √2(𝑚0 𝑐 2 )𝐾 12,4 𝑥 103 𝑒𝑉.Å √2(940𝑥106 𝑒𝑉)(0,05𝑒𝑉)



= 1,28 Å



66



3.2. Hukum Difraksi Bragg Teori tentang difraksi sinar-X dikembangkan oleh Sir William H. Bragg pada tahun 1913. Bragg memperlihatkan bahwa sebuah bidang susunan atom-atom dalam sebuah Kristal, yang disebut bidang Bragg, akan memantulkan radiasi tepat seperti cahaya yang dipantulkan dari sebuah bidang cermin.



Jika kita tinjau radiasi yang dipantulkan bidang-bidang Bragg berurutan yang sejajar yang jarak antaranya d, maka tampak bahwa mungkin bagi berkas-berkas yang terpantuldari tiap-tiap bidang berinterferensi konstruktif sehingga menghasilkan berkas terpantul yang berintensitas tinggi. Persyaratan interferensi konstruktif adalah bahwa selisih jalan antara kedua sinar, yakni 2d sin θ, harus sama dengan kelipatan bula panjang gelombang , dengan demikian memberikan hukum Bragg n𝜆 = 2d sin θ jika n dan d diketahui, panjang gelombang berkas dating dapat ditentukan dengan mengukur sudut hamburan 2 θ,antara berkas transmisi dan yang terdifraksi. Dalam sebarang Kristal, kebanyakan keluarga bidang Bragg yang berbeda, jaraknya sendiri-sendiri, dapat dibentuk dengan mengiris Kristal dalam berbagai cara. Tiap-tiap keluarga ini dapat menghasilkan difraksi. Oleh karena itu, jika sebuah berkas sinar-X dilewatkan melalui Kristal-kristal yang diarahkan secara acak, seperti dalam contoh bahan bubuk atau suatu lembaran tipis. Suatu lingkaran tertentu akan bertalian dengan suatu orde tertentu oleh suatu keluarga bidang yang tertentu pula. Dalam soal-soal yang dipecahkan berikut, kita hanya meninjau difraksi dari bidang-bidang Bragg Utama yang jarak bidangnya berukuran antar atom. 67



Percobaan difraksi electron pertama kali dilakukan oleh CJ. Davisson dan L.H. Germer di Bell Telephon Laboratories. Mereka mengarahkan sebuah berkas electron 54eV pada sebuah Kristal tunggal logam nikel, yang jarak antar atomnya diketahui dari pengukuran difraksi sinar-X, sebesar 2,15 Å, kemudian mereka mengukur intensitas electron-elektron yang terhambur sebagai fungsi sudut hamburan. Jika tidak terdapat efek difraksi,maka diharapkan intensitas electron terhambur akan turun secara monoton dengan sudut hamburan, dengan tak adanya difraksi yang besar dari electron yang dating dari sebarang sudut tertentu. Ternyata mereka mendapatkan bahwa terdapat suatu puncak mencolok dalam intensitas electron pada sudut hamburan 500. Dengan suatu koreksi keci, diperoleh bahwa panjang gelombang yang dihitung sesuai dengan panjang gelombang de Broglie, yang dengan demikian membuktikan kebenaran hipotesis de Broglie. Tak lama kemudian setelah percobaan Davisson dan Germer, G.P. Thomson , pada 1927, mempelajari transmisi electron melalui lembaran – lembaran tipis logam. Jika electron- electron berperilaku sebagai partikel, maka suatu bayangan kabur akan dihasilkan dalam berkas yang ditransmisikan . Ternyata, Thomson mendapatkan pola-pola difraksi lingkaran, yang hanya dapat diterangkan dari sudut pandang teori gelombang, yang selanjutnya membuktikan kebenaran hipotesis de Broglie. Contoh Soal : 1. Sebuah berkas neutron terhambur dari suatu contoh bahan yang tak diketahui, dimana teramati suatu pundak pantulan Bragg yang terpusat pada sudut 220. Berapakah jarak antar bidang Bragg ? Diketahui : K = 0,083 eV θ = 220 Ditanya : Jarak antara bidang Bragg (d) ? Jawab



: 𝜆=



ℎ 𝑝



= =



=



ℎ √2𝑚0 𝐾 ℎ𝑐 √2(𝑚0 𝑐 2 )𝐾 12,4 𝑥 103 𝑒𝑉.Å √2(940𝑥106 𝑒𝑉)(0,083𝑒𝑉) 68



= 0,993 Å Dengan menganggap puncak pantulan ini berkaitan dengan difraksi orde pertama ( n=1), kita peroleh



d=



𝜆 2 𝑠𝑖𝑛𝜃



=



0,993Å 2 𝑠𝑖𝑛 220



= 1,33 Å



2. Sebuah Kristal besar digunakan untuk menyaring neutron-neutron berenergi tunggal (single-energy) dari sebuah berkas neutron yang keluar dari reactor. Jarak antara bidang-bidang Bragg adalah 1,1 Å . Jika sudut Bragg diatur 300, berapakah energy neutron yang dilihat pada sudut ini untuk pantulan orde-pertama? Penyelesaian Diketahui : d=1,1 Å 𝜃 = 300



Ditanya : Energi neutron/K ? Jawab :



𝜆 = 2 𝑑 𝑠𝑖𝑛𝜃 = 2 (1,1 Å) sin 300 = 1,1 Å 𝜆= = = K= K=



ℎ 𝑝 ℎ √2𝑚0 𝐾 ℎ𝑐 √2(𝑚0 𝑐 2 )𝐾 (ℎ𝑐)2 2(𝑚0 𝑐 2 )λ2 (12,40 × 103 𝑒𝑉.Å)2 2(940×106 𝑒𝑉)1,1 Å2



K= 0,0676 eV



69



3.3. Hubungan Ketidakpastian Bagi Gelombang Klasik Dalam pasal ini kita menyelidiki perbedaan penting lainnyaantara partikel klasik dan gelombang. Marilah kita tinjau sebuah gelombang berbentuk y = y1 sin k1 x, seperti yang diperlihatkan pada gambar di bawah ini adalah sebuah gelombang yang terus-menerus mengulang bentuknya tanpa akhir dari x = −∞ hingga x = +∞. (panjang gelombangnya, dipihak lain, tertentukan secara pasti, sama dengan 2𝜋/𝑘1 ). Jika kita menggunakan sebuah gelombang untuk menyatakan sebuah partikel maka gelombang itu harus memiliki salah satu sifat penting partikel berikut: ia harus bersifat setempat (localized), atau dapat dikungkung ke dalam suatu bagian ruang kecil (misalnya dalam ukuran atom atau inti atom). Gelombang sinus murni tidak dapat digunakan untuk menentukan letak setempat partikel.



Gambar 4.1 sebuah gelombang sinus murni yang merentang dari -∞ hingga Sekarang, tinjaulah apa yang terjadi apabila kita memadukan gelombang yang pertama tadi dengan gelombang lain yang panjang gelombangnya agak berbeda (jadi, k yang berbeda), sehingga 𝑦 = 𝑦1 Sin 𝑘1 𝑥 + 𝑦2 sin 𝑘2 𝑥. Pola khas yang dihasilkan, yang bagi kasus gelombang suara dikenal sebagai “layanan” (beat), diperlihatkan pada gambar 4.2. Polanya tetap berulang terus-menerus dari x = −∞ hingga x = +∞, tetapi sekarang kita sedikit mengetahui tentang “letak” gelombangnya pada nila-nilai x tertentu dimana zat perantaranya tampak kurang “bergelombang” dari pada tempat lainnya (atau sekurang-kurangnya “bergelombang” dengan amplitudo yang lebih kecil). Dalam gambar 4.2, kita akan mengamati getaran pada titik x = xA, tetapi tidak pada x = xB. Status pengetahuan kita tentang “letak” gelombang tampaknya mulai lebih baik, namun dengan bayaran ketidakpastian pada panjang gelombangnya yaitu, bahwa pemanduan dua gelombang dengan panjang gelombang berbeda mengakibatkan kita tidak dapat lagi menentukan secara pasti panjang gelombangnya.



70



Gambar 4.2 Superposisi dua gelombang sinus dengan panjang gelombang yang hampir sama menghasilkan layangan. Perbedaan panjang gelombang dari kedua gelombang sinus ini adalah 10 persen tetapi kedua amplitudo sama.



Gambar 4.3 resultan perpaduan sejumlah besar gelombang sinus (dengan panjang gelombang yang berbeda-beda dan mungkin pula amplitudo yang berbeda). Jika kita lanjutkan dengan menjumlahkan lagi beberapa gelombang dengan panjang gelombang yang berbeda (bilangan gelombang k yang berbeda), dengan amplitudo dan fase yang dipilih secara tertentu, maka pada akhirnya kita akan mencapai suatu keadaan seperti yang diperlihatkan pada gambar 4.3. Amplitudo gelombang seperti itu adalah nol di luar suatu bagian ruang sempit ∆𝑥 (∆𝑥 tidak tertentukan secara pasti, tetapi merupakan taksiran kasar bahwa di situ gelombang memiliki amplitudo yang cukup besar). Untuk mencapai keadaan ini, kita harus memadukan sejumlah besar gelombang dengan bilangan gelombang k yang berbeda. Jadi gelombang paduannya menyatakan suatu rentang bilangan gelombang (panjang gelombang) yang kita tunjukan dengan ∆𝑘. Apabila kita kita mempunyai sebuah gelombang sinus murni, ∆𝑘 adalah nol (karena hanya ada satu k) sehingga ∆𝑥 menjadi tidak hingga (gelombangnya mencakup seluruh ruang). Bila kita memperbesar ∆𝑘 (dengan menambahkan lebih banyak gelombang), maka pada saat yang sama kita memperkecil ∆𝑥 (gelombang menjadi lebih terkungkung). Tampaknya kita mempunyai suatu hubungan berbanding terbalik antara ∆𝑥 dan ∆𝑘 yaitu, bila salah satu mengecil, maka yang lain membesar. Hubungan matematik hampiran antara ∆𝑥 dan ∆𝑘 ini adalah ∆𝑥∆𝑘 ~ 1



(4.3)



71



Tanda sama bergelombang dimaksudkan “dalam orde besarnya.” (karena ∆𝑥 dan ∆𝑘 tidak tertentukan secara pasti, maka besarnya disini hanya merupakan taksiran, sehingga dengan demikian persamaan (4.3) adalah petunjuk kasar mengenai hubungan antara keduanya). Persamaan (4.3) menyatakan bahwa hasil kali dari ∆𝑥, jarak lebar gelombang, dengan ∆𝑘, rentang bilangan gelombang yang dikandungnya, besarnya dalam orde satuan. Untuk sebarang gelombang, berlaku aturan bahwa kedudukannya hanyalah dapat ditentukan secara pasti dengan bayaran pengetahuan kita tentang kepastian bilangan gelombang menjadi berkurang. Pernyataan ini, dan ungkapan matematikanya yang diberikan dalam persamaan (4.3). Hubungan ini dapat kita tafsirkan dengan cara lain sebagai berikut. Andaikanlah kita berupaya untuk mengukur panjang gelombang sebuah gelombang klasik, seperti gelombang air. Ini dapat kita lakukan dengan mengukur jarak antara dua puncak gelombang yang berdekatan. Andaikanlah gelombang itu adalah suatu pulsa yang sangat sempit dengan hanya suatu puncak gelombang (gambar 4.14) maka pengukuran 𝜆-nya menjadi sangat sulit, dan kita cenderung membuat kesalahan besar, mungkin dalam orde satu panjang gelombang. Ini berarti, apabila perluasan ruang dari gelombang itu adalah ∆𝑥 ~𝜆, maka Δ𝜆 ~ 𝜆. (Ingat, tanda ~ berarti, “dalam orde”). Maka untuk gelombang ini, kita peroleh ∆𝑥∆𝑘 ~ 𝜆2 . Andaikanlah gelombang itu kemudian meluas hingga mencapai beberapa panjang gelombang, sehingga ∆𝑥 ~𝑁𝜆. Maka sekarang 𝜆-nya dapat kita tentukan dengan ketelitian yang lebih tinggi. Namun, untuk pencacahan jumlah bilangan bilangan gelombangnya dalam ∆𝑥, kita masih membuat kesalahan dalam orde satu panjang gelombang (mungkin ½ atau ⅓ atau ¼, tetapi masih dalam orde satuan) dibagi N, jadi sekarang ∆𝜆 ~𝜆/𝑁, dan sekali lagi ∆𝑥Δ𝜆 ~𝜆2. Hubungan ketidakpastian ini, yang mengaitkan “ukuran” suatu gelombang dengan ketidakpastian dalam pengukuran panjang gelombangnya, ternyata setara dengan persamaan (4.3) Marilah sekarang kita mencoba mengukur frekuensi suatu gelombang (gelombang suara, misalnya). Andaikanlah kita dapat mengamati setiap getarannya (pada suatu osiloskop, misalnya) dengan suatu peralatan pencacah yang memadai. Jika kita mencacah selama selang waktu 1 detik dan mencatat 100 getaran, maka kita memperoleh frekuensi 100Hz. Tetapi, kita tak dapat yakin bahwa getaran 100Hz ini telah kita cacah secara pasti. Berapa jauhkah getaran keseratus satu telah berlalu ketika selang 1 detik 72



terakhir? Jika separuh getaran yang telah berlalu, maka frekuensi sebenarnya dalah 100,5Hz. Andaikanlah sekarang kita mencacah untuk selang waktu 2 detik. Maka kita dapat mencatat 200 getaran, dan akan menyimpulkan frekuensi 100 Hz kembali, namun kita akan tidak yakin kembali tentang berapa jauhkah getaran kedua ratus satu telah berlalu. Jika separuh getaran telah berlalu lagi, maka kita akan mempunyai 200,5 getaran dalam 2 detik, atau frekuensi sebenarnya adalah 100,25Hz. Ketidakpastian dalam frekuensi (sekarang 0,25Hz) telah mengecil dengan faktor 2 apabila kita melipatduakan selang waktu pengukuran kita.



Gambar 4.4 dua kelompok gelombang yang berbeda Oleh karena itu, di sini kita memperoleh pula hubungan kebalikan seperti yang kita simpulkan sebelum ini: ketidakpastian dalam frekuensi, ∆𝑣, berbanding terbalik dengan ketidakpastian dalam selang waktu, ∆𝑡, masa pengukuran dilakukan, dengan menggunakan frekuensi sudut 𝜔 = 2𝜋𝑣 kita dapat menuliskan hubungan ini sebagai berikut: ∆𝜔∆𝑡 ~ 1



(4.4)



Ini adalah hubungan ketidakpastian keduayang kita peroleh bagi gelombang klasik, dan serupa dengan Persamaan (4.3) dalam arti bahwa ia memberikan suatu hubungan



antara taksiran ketidakpastian pengukuran semua besaran



yang



bersangkutan. Contoh Soal Dalam suatu percobaan pengukuran panjang gelombang dari gelombang air, 10 puncak gelombang tercacah dalam jarak 200 cm. Taksiran ketidakpastian minimum dalam panjang gelombang yang dapat diperoleh dari percobaan ini. Jawab : k=



2𝜋 𝜆 73



−2𝜋



dk= ∆k=



𝜆2 2𝜋



∆𝜆



𝜆2 2𝜋



∆x(



𝜆2



∆𝜆 ~



𝑑𝜆



∆𝜆) ~ 1



1 𝜆2 ∆𝑥 𝜆



= 2



1 202 200 2𝜋



~ 0.3 cm



3.4. Hubungan Ketidakpastian Heisenberg Hubungan ketidakpastian bagi gelombang klasik berlaku bagi semua gelombang, dan karena itu kita seharusnya dapat pula menerapkannya pada gelombang de Broglie. Dengan menggunakan hubungan mendasar de Broglie p = h/𝜆 bersama dengan pernyataan k=2π/𝜆 kita dapatkan p=hk/2π, yang mengaitkan momentum sebuah partikel dengan bilangan gelombang dari gelombang de Broglienya . Mengingat gabungan h/2π sering kali muncul dalam mekanika gelombang , maka untuknya diberikan lambing khusus ħ ( “h coret) ħ=



ℎ 2π



= 1,05 × 10−34 J.s



= 6,58 × 10−16 eV.s Dengan menggunakan ħ maka p = ħk………….(4.5) sehingga ∆k = ∆p/ ħ. Dengan demikian hubungan ketidakpastian Heisenberg diperoleh ∆x ∆px ~ ħ……………..(4.6) Persamaan di atas berlaku bagi gerak sepanjang suatu arah tertentu, yang menyatakan ketidakpastian dalam kedudukan dan momentum hanya pada arah tersebut. Hubungan serupa yang tidak bergantungan dapat diterapkan pula pada arah-arah lainnya, jadi berlaku pula ∆y∆py~ħ atau ∆z ∆pz ~ ħ.



74



Hubungan de Broglie E = hv dapat dituliskan sebagai E = h𝜔. Jadi , ∆𝜔 = ∆E / ħ , sehingga ∆E ∆t ~ ħ……………(4.7) Persamaan diatas dikenal dengan hubungan ketidakpastian Heisenberg, yang adalah pernyataan matematis dari asa ketidakpastian Heisenberg. Asas ini mengatakan bahwa tidak ada satupun percobaan yang dapat dilakukan sedemikian rupa sehingga memberikan ketidakpastian di bawah batas-batas yang diungapkan pada persamaan – persamaan di atas. Warner Heisenberg (1901-1976, warga Jerman). Sangat terkenal karena asas ketidakpastiannya, ia juga mengembangkan suatu rumusan lengkap mengenai teori kuantum yang didasarkan pada matriks. Hubungan-hubungan ini memberikan suatu taksiran ketidakpastian minimum yang dapat diperoleh dari beraneka percobaan; pengukuran kedudukan dan momentum sebuah partikel akan memberikan sebaran nilai selebar Δx dan Δ𝑝𝑥 . Kita mungkin dapat saja melakukan pengukuran yang ketelitiannya menyimpang jauh daripada yang diberikan (4.6) dan (4.7), tetapi yang lebih baik daripada itu tidak dapat kita capai. (Mungkin seringkali anda jumpai bahwa hubungan-hubungan ini ditulis dengan h/2 atau h, ketimbang h, pada ruas kanan, atau juga dengan >ketimbang dengan ~



yang



memperlihatkan kesamaan. Perbedaan ini tidak terlalu penting,karena (4.6) dan (4.7) hanya memberikan taksiran. Ketidakpastian Δx dan Δ𝑝𝑥 yang sebenarnya bergantung pada distribusi bilangan gelombangnya (panjang gelombangnya) yang digunakan untuk membatasi gelombang pada daerah (selang) Δx; distribusi yang lebih rapi memberikan Δx Δ𝑝𝑥 = h/2, sedangkan distribusi lainnya akan memberikan Δx Δ𝑝𝑥 > h/2. Dengan demikian, cukup aman bagi kita untuk menggunakan h sebagai suatu taksiran). Hubungan-hubungan ini memberi pengaruh yang sangat jauh pada pandangan kita terhadap alam. Dapat diterima bila dikatakan bahwa terdapat ketidakpastian dalam menentukan letak sebuah gelombang air. Namun permasalahannya menjadi lain bila pernyataan yang sama diterapkan pada gelombang deBrogli, karena akan tersirat bahwa terdapat pula ketidakpastian dalam menentukan letak partikel. Persamaan (4.6) dan (4.7) mengatakan bahwa alam menetapkan suatu batas ketelitian yang dapat kita 75



gunakan untuk melakukan sejumlah percobaan; tidak peduli sebaik apapun peralatan ukur kita dirancang ,kita tidak dapat melakukan pemgukuran yang lebih teliti daripada yang disyaratkan oleh persamaan (4.6) dan (4.7) .



Gambar 4.5 Momentum sebuah partikel yang terbatasi kedudukannya dalam selang ∆x. Pengukurannya berulang kali, tiap nilai pi diukur sebanyak ni kali. Momentum rataђ ratanya nol, dan distribusinya memiliki lebar ∆p ~ Δ𝑥 .



Tentu saja, sebuah partikel klasik tidak dapat langsumg bergerak dari keadaan diam bila tidak dikenai daya. Karena itu, bagaimana partikel dapat terjadi memiliki momentum tidak nol? Dilema kita disini berpangkal dari perkataan partikel. Telah kita bahwa istilah “partikel” dan “gelombang” tidaklah berdiri sendiri dalam fisika kuantum, yang mengungkapkan bahwa deskripsi yang tepat dari suatu sistem fisika haruslah melibatkan kedua aspek ini. Perilaku gelombanglah yang menyebabkan terjadinya penyebaran distribusi momentum bila jarak ruang L diperkecil. (Analogi gelombang klasiknya adalah: pemendekan secara berangsur panjang sebuah senar gitar yang dipetik, lewat tekanan jari yang menggeser sepamjamg senarnya, menyebabkan senar tersebut bergetar dengan frekuensi yamg semakin tinggi, sehingga dengan demikian menjadi semakin lebih cepat getarannya. Perlu dicatat bahwa semua analogi klasik lain yang terbatas ruang lingkupnya. Hendaklah kita jangan terlalu bersungguhsungguh menanggapi analogi ini). Untuk menentukan letak sebuah partikel, kita harus menentukan



amplitudo



gelombang



deBroglie-nya,



yang



dilakukan



dengan



menjumlahkan semua macam komponen gelombangnya; semakin kecil L dibuat, maka menurut persamaan (4.3), semakin banyak gelombang yang harus dijumlahkan. Masing-masing gelombang yang beraneka panjang gelombangnya ini, yang pada umumnya merambat melalui zat perantara dengan laju yang brbeda- beda, terpantul bolak-balik antara kedua dinding pemantul. Ketika kedua dinding berada di ± ∞ hanya satu gelombang yang diperlukan , tidak ada disersi atau pantulan yang terjadi,dan 76



perilau partikel tidak berubah terhadap waktu. Ketika kedua titik didekatkan, lebih banyak gelombang yang diperlukan, disperse dan pantulan kini dapat terjadi, dan kadang gelobang berpada menghaslkan satu ketidak seinbangan sesaat antara gelobang yang bergerak kekanan dan yang bererak kekiri, yang kita amati sebagai nilap px yang tiak nol. Pengukuran yang banyak akan mungkin memperlihatkan bahwa jumlah gerak partkel kekanan sama banyaknya dengan gerak kekiri, sehngga momentum rata-rata pavsama dengan nol, karena momentum yang berlawanan saling menghapuskan. Ratarata besar momentumnya I p Iav tidaklah nol. (I p Iav hanyalah nol jika semua p adalah nol). Semakin dekat jarak suatu dinding , semakin banyak pantuln yang terjadi, dan semakin besar peluang bagi beberapa komponen momentum berinterferenisi secara maksimum sehingga memberikan suatu momentum besar



pada arah tertentu.



Akibatnya, partikel akan mulai “bererak” semakin cepat; meskipun pav masih tetap nol, I p Iav menjadi semakin besar. Oleh karena itu, ∆𝑝ampaknya berkaitan dengan I p Iav , yang berkaitan dengan (p2)av . definisi yang pasti dari , ∆𝑝 adalah ∆𝑝 = √(𝑝2 ) 𝑎𝑣 − (𝑝𝑎𝑣)2



…………….. . . . . (4.8)



Perhatikan kesamaan definsi ini dengan konsep statistic deviasi standar dari sebuah besaran x yang memiliki nilai rata-rata x, 1



2 𝜎 = √𝑁 ∑𝑁 𝑖=1(𝑥𝑖 − 𝑥)



1



2− 2 = √𝑁 ∑𝑁 𝑖=1 𝑥𝑖 𝑥



= √(𝑥)𝑎𝑣 2 − (𝑥𝑎𝑣)2 Contoh Soal Sebuah meson pi bermuatan memiliki energy diam 140 MeV dan waktu hidup 26 ns. A. Carilah ketidakpastian energy meson pi ini, dinyatakan dalam MeV dan juga dalam perbandingan terhadap energy diamnya. B. Ulangi perhitungan ini bagi meson pi yang tidak bermuatan , yang memiliki energy diam 135 MeV dan waktu hidup 8,3 × 10-17 s. 77



C. Ulangi lagi bagi meson rho, yang memiliki energy diam 765 MeV dan waktu hidup 4,4 ×



10-24 s.



Penyelesaian : A. Diketahui = E=140 MeV ∆t = 26 × 10-9 s Ditanya : Ketidakpastian energy meson pi ? Jawab : ∆E =



=



ħ ∆𝑡 6,58×10−18 𝑒𝑉.𝑠 26×10−9 𝑠



= 2,5 × 10−14 MeV ∆𝐸 𝐸



=



2,5 × 10−14 𝑀𝑒𝑉 140𝑀𝑒𝑉



= 1,8 × 10−16



B. Diketahui = E=135 MeV ∆t = 8,3 × 10-17 s Ditanya : Ketidakpastian energy meson pi ? Jawab : ∆E =



=



ħ ∆𝑡



6,58×10−18 𝑒𝑉.𝑠 8,3 × 10−17 s



= 7,9 × 10−6 MeV ∆𝐸 𝐸



=



7,9 × 10−6 𝑀𝑒𝑉 135𝑀𝑒𝑉



= 5,9 × 10−8



C. Untuk Meson rho Diketahui = E=765 MeV ∆t = 4,4 × 10-24 s Ditanya : Ketidakpastian energy meson pi ? Jawab : ∆E =



=



ħ ∆𝑡 6,58×10−18 𝑒𝑉.𝑠 4,4 × 10−24 s



= 1,5 × 108 eV = 150 MeV



78



∆𝐸 𝐸



=



150𝑀𝑒𝑉 765𝑀𝑒𝑉



= 0,20



Contoh Soal 2 Taksirkanlah kecepatan minimum sebuah bola bilyard ( m=100 g) yang geraknya terbatasi pada meja bilyard berukuran 1 m. Penyelesaian : Diketahui : m = 100 g ∆x = 1 m Ditanya : kecepatan minimum (∆v ) ? Jawab : ∆px =



∆vx =



ħ ∆𝑥



∆px 𝑚



= =



1,05×10−34 𝐽.𝑠 1𝑚



= 1× 10−34 kg.m/s



1×10−34 𝑘𝑔.𝑚/𝑠 0,1 𝑘𝑔



= 1×10−22 m/s



3.5. Paket Gelombang Kedudukan sebuah gelombang sinus (atau kosinus) murni sama sekali tidak terbatasi. Ia meluas dari -∞ hingga +∞. Sebaliknya kedudukan sebuah partikel klasik , terbatasi secara tegas. Sebuah paket gelombang dapat dipandang sebagai superposisi sejumlah besar gelombang, yang berinterferensi secara maksimum disekitar partikel, sehingga menghasilkan sebuah gelombang resultan dengan amplitudo yang lebih besar. Sebaliknya pada tempat yang jauh dari partikel, mereka berinterferensi secara minimum, sehingga gelombang resultannya memiliki amplitudo yang lebih kecil



pada tempat dimana



partikelnya kita perkirakan tidak ditemukan. Kita memperkirakan bahwa deskripsi matematika paket gelombang akan melibatkan penjumlahan (superposisi ) sejumlah gelombang dengan panjang gelombang yang berbeda-beda.Tinjau sebuah gelombang dengan bilangan gelombang k1 kemudian menambahkan padanya sebuah gelombang lain dengan bilangan gelombang yang hamper sama k2 = k1+∆k. Komponen-komponen gelombanya pada X=0 bergetar dengan fase 79



sama, sehingga gelombang resultannya memiliki amplitudo yang sama disana. Semakin jauh dari x = 0, perbedaan kecil dalam kedua panjanggelomangakan menyebabkan fase kedua gelombang sinus ini menjadi berlawanan, sehingga gelombang resultannya memiliki amplitudo nol. Dengan sedikit manipulasi trigonometri kita peroleh hasil Y(x) = A cos k1x + a cos k2x ∆𝑘



= 2A cos ( 2 x) cos (



𝑘₁+𝑘₂ 2



x)



(4.10)



Suku persaman (4. 10) diatas memberikan perubahan amplitudo gelombang resultan dalam selubung yang dicirikan oleh suku kosinus yang pertama. Sekarang kita tinjaugelombang-gelombang ini sebagai gelombang ramat, yang deskripsi matematiknya diperoleh dari persamaan (4.10) dengan mensubstitusikan (kx – 𝜔t) pada kx. Frekunsi sudutnya adalah 𝜔 dan v = 𝜔 /k adalah kecepatan fase gelombangnya laju dengannya satu komonen bergerak gelombang bergerak melalui zat perantara. Kedua komponen gelombang ini diperlihatkan lagi pada gambar 4.18 untuk t=0 dan waktu t berikutnya. Pada umumnya kecepatan fase v1 =𝜔1/k1 dan v2 = 𝜔2/k2 dapat tidak sama. perhatikan bahwa selubungnya bergerak dengan kecepatan yang berbeda dari masing-nasing komponen gelombangnya. Sekali lagi kita dapat menurunkan pernyataan eksplisit lagi bagi gelombang resultannya dengan melakukan sedikit maniulasi trigonometri yang memberikan hasil : Y(x,t) = A cos (k1x –𝜔1t) + A cos (k2x -𝜔2t) ∆𝑘



= 2A cos ( 2 x −



∆ω 2



𝑘₁+𝑘₂



t)cos(



2



x−



ω₁+ω₂ 2



t)



(4.11)



Dimana ∆ω = ω2-ω1 .jadi, selubungnya bergerak dengan laju v = ∆ω/∆k,sedangkan gelombang didalamnya bergerak dengan laju (ω1 –ω2)/(k1 + k2), yang mana , jika ∆ωdan∆k kecil, tidak terlalu berbeda jauh dari v1 dan v2. Superposisi dari hanya dua gelombang saja tampak tidak menyerupai paket gelombang pada gambar 4.3. Hampiran yang lebih baik dapat kita buat dengan menjumlahkan lebih banyak gelombang sinus dengan bilangan gelombang ki yang berbeda, dan amplitude A(ki) yang mungkin pula bebeda:



80



Y(x) = ∑𝑠𝑒𝑏𝑎𝑟𝑖𝑠 𝐴 (ki) cos kix



(4.12)



𝑘



Gambar 4. 8 paket



Kecepatan grup sebuah



gelmbang.



Gambar



kiri



memperlihatkan “ gambar potret pada t = 0 dari gelombang y1,y2 dan jumlahnya (y1 memiliki panjang gelombang satuan, sedangkan y2 adalah ,109satuan). Gelombang bergerak dengan kecepatan 3 satuan perdetik, sedangkan gelombang 2 dengan 2,5 satuan per detik. Gambar potret pada t = 1 detik diperlihatkan disebelah kanan. Kedua gelombang tidak sefase hinggs jarak 7,5 satuan, Jadi titik tengah layangan “bergerak dengan kecepatan 7,5 satuan perdetik, yang dalam kasus ini lebih besar daripada v1 dan v2. Jika terdapat banyak bilangan gelombang yang berbeda dan jika mereka sangat berdekatan, maka jumlah dalam persamaan (4.12) dapat digantikan dengan suatu integral : 1



𝑦(𝑥) = ∫ 𝐴(𝑘) cos 𝑘𝑥𝑑𝑘



(4.13)



Integralnya diambil untuk seluruh rentang bilangan gelombang yang diperkenkan (dapat terjadi dari 0 hingga ∞). Andaikanlah, kita mempunyai suatu rentang bilangamn gelombang dari k0 - ∆k/2 hingga k0 + ∆k/2. Jika semua gelombang memiliki amplitude A yang sama, maka dari persamaan (4.13) ,bentuk paket gelombangnya dapat diperlihatkan 𝑦(𝑥) =



2𝐴 𝑥



∆𝑘



sin ( 2 𝑥) cos 𝑘0X



(4.14)



Hampiran bentuk paket gelombang yang lebih baik dapat diperole dengan mengamil A(k) berubah-ubah; sebagai contoh, bentuk fungsi Gauss A(k) = 2 /2(∆𝑘)2



𝑒 −(𝑘−𝑘₀)



memberikan



Y(x) ∞ 𝑒 −(𝑘−𝑘



0 )2 /2(∆𝑘)2



cos 𝑘₀x



(4.15)



Disini terdapat lagi gelombang selubung yang memodulasikan gelombang kosinus dan memperkecil amplitudonya diluar daerah selebar ∆x, seperti yang diperlihatkan pada 81



gambar 4.19. untuk membatasi gelombang inin pada daerah sekecil ∆x, kita telah menggunakan lagi rentang bilangan gelombang yang besar



Gambar 4.9



Contoh dua paket gelombang yang



berbeda. Bagi masing-masing paket gelombang, terdapat suatu fungsi modulasi yang memperkecil amplitude kosinus diluar daerah ∆x. Gambar 4.9 haruslah dipandang sebagai gambarv potert paket gelombang pada suatu waktu tertentu, seperti t= 0. Begitu pula, persamaan (4.14) dan (4.15) hanya menyatakan gelombang pada t = 0. Untuk mengubahnya kebentuk gelombang rambat maka kita harus menggantikan kx dengan kx – ωt, seperti yang kita akukan pada persamaaan (4.11). dalam kasus dua gelombang yang kita gunakan bagi persamaan (4.11), kita dapati bahwa gelombang selubungnya bergerak dengan laju ∆ω/∆k. Kasus sederhana ini kita perluas keklasus dimana terdapat banyak bilangan gelmbang tidak sama dengan mendefinisikan kecepatan grup sebagai berikut : Vgrup =



𝑑ω



(4.16)



dk



Selubung paket gelombang



ini bergerak pada kecepatan grup, sedangkan



didalamnya, setiap komponen gelombang bergerak dengan kecepatan fase masing-masing Vfase =



ω



(4.17)



k



Kecepatan fase hanya bermakna bagi satu komponen



gelombang, tidak



terdefinisikan bagi paket gelombang. Jadi, sebuah partikel yang terbatasi kedudukannya pada suatu bagi ruang tertentu tidak hanya dinyatakan oleh satu gelombang de Broglie dengan energy dan frekuensi tertentu, tetapi oleh sebuah paket gelombang yang merupakan superposisi dari sejumlah besar gelomban. Selubung gelonbangnya bergerak dengan kecepatan grup 𝑑𝜔/𝑑𝑘. Jika kita tidak dapat merumuskan suatu teori memetika yang meramalkan hasil dari satu kali pengukuran, maka kita dapat berupaya untuk memperoleh suatu teori matematik yang meramalkan perilaku statistic dari suatu system (tau dari sejumlah besar



82



h𝜔 begitu pula, momentum dan bilangan gelombang berkaitan melalui hubungan p = hk. Jadi, kecepatan grup vg =𝑑𝜔/𝑑𝑘dapat pula dinyatakan dalam cara berikut : vg =



𝑑𝜔 𝑑𝑘



𝑑𝜔



𝑑𝐸



𝑑𝑝



1



𝑑𝐸



= ( 𝑑𝑘 ) (𝑑𝑝 ) (𝑑𝑘 ) = (ℎ) (𝑑𝑝) (ℎ)



(4.18)



𝑑𝐸



vg =𝑑𝑝 Kecepatan grup bukanlah sifat gelombang komponennya melainkan merupakan sifat zat perantara dalam mana paket gelombang itu bergerak. Sekarang kita buat anggapan berikut, yang sangat pokok gbagi mekanika mendasar dari teori kuantum. Kita menganggap bahwa tanggapan zat perantara terhadap paket gelombang, diberikan oleh dE/dp, yang identik dengan tanggapan zat perantara pada bagian partikel. Yaitu 𝑑𝐸



𝑑𝐸



(𝑑𝑝)paket gelombang = (𝑑𝑝)partikel



(4.19)



Dalam pernyataan untuk energy sebuah partikel, hanya energy kinetic K yang bergantung pada momentum, sehingga dE/dp = dK/dp ; karena K =p2/2m bagi sebuah partikel tidak relativistic, maka dK/dp = p/m, yang tidak lain adalah kecepatan partikel sedangkan ruas kiri adalah kecepan grup dari paket gelmbang. Dengan demikian kita telah memperoleh hasil penting berikut. Kecepatan sebuah partikel materi sama dengan kecepatan grup paket gelombang yang bersangkutan. Jadi bahasan ini dapat dirangkum sebagai berikut. Sebuah partikel yang terbatas geraknya dalam suatu bagian ruang dilukiskan sebagai oleh sebuah paket gelombang, yang adalah superposisi gelombang-gelombang deBroglie. Peket gelomabang bergerak dengan laju yang sama dengan laju pertikel. 3.6. Probabilitas dan Keacakan Pengukuran sekali terhadap kedudukan atau momentum partikel dapat dilakukan



seteliti yang



Lalu, bagaimanakah perilaku



dapat



dicapai



oleh keterampilan eksperimental kita.



gelombang sebuah



partikel



dapat



kita



amati



?



Bagaimanakah ketidakpastian dalam kedudukan dan momentum mempengaruhi percobaan kita ? Perlemparan sebuah mata uang atau dadu bukanlah suatu proses acak, akan tetapi hakikat keacakan hasilnya itu menunjukan bahwa pengetahuan kita tentang keadaan sistemnya lah yang kurang lengkap. Apabila kita menganalisis hasil yang 83



bakal diperoleh berdasarkan probabilitas, maka kita sebenarnya mengakui kelemahan kita untuk melakukan analisisnya secara pasti. Perilaku acak dari sebuah sistem yang tunduk pada hukum-hukum fisika kuantum adalah suatu aspekalam mendasar, bukanlah hasil dari keterbatasan pengetahuan kita tentang sifat-sifat sistemnya. 3.7. Amplitudo Probabilitas Masih ada satu lagi persoalan terakhir yang perlu di bahas, yaitu apakah yang di tanyakan oleh amplitudo gelombang deBroglie? Dalam setiap gejala penghambatan gelombang,suatu besaran fisika seperti perpindahan atau tekanan mengalami perubahan terhadap jarak dan waktu. Lalu, sifat fisika apakah yang mengalami perubahan ketika gelombang deBroglie merambat ? Dalam salah satu pasal di depan, kita tidak pernah membahas sebuah pertikel yang terbatasi kedudukannya dengan sebuah paket gelombang. Jika partikelnya terbatasi pada suatu partikel bagian ruang berukuran , maka paket gelombang yang menyatakan partikel tersebut hanya memiliki amplitudeyang besar dalam derah itu, sedangkan di luarnya amplitudo paket gelombangnya kecil. Artinya, amplitudo paket gelombang itu besar pada tempat di mana partikelnya berada, dan kecil pada daerah dimana kemungkinanan mendapatkan pertokel itu kecil. Jadi, amplitudo



gelombang deBrogli (sebuah



partikel)pada sembarang titik berkaitan dengan probabilitas untuk menemukan partikel yang bersangkutan pada titik tersebut. Analogi dengan fisika klasik, bahwa intensitas sebuah gelombang berbanding lurus dengna kuatdrat amplitudonya,maka probabilitas ini juga berbanding lurus dengan kuadrat amplitudo gelombang deBroglie. Dalam bab berikut kita akan membahas kerangka matematika untuk menghitung amplitudo bagi sebuah partikel yang berada dalam beraneka ragam situasi, dan juga membahas definisi probabilitas yang lebih matematis. Kesulitan kita untuk menafsirkan secara tepat amplitudo gelombang ini sebagian disebabkan karena amplitudo gelombang adalah suatu besaran kompleks. (Suatu variable kompleks, seperti amplitudo probabilitas, adalah variable yang mengandung suatu bagian imaginer, yang berbanding lurus dengan akar kuadrat dari -1, yang dilambangkan denga ; lihat pasal 5.6.). karena kita tidak dapat mengungkapkan variable-variable tersebut dengan sistem bilangan real (tidak imajiner) kita, maka kita tidak dapat menafsirkan atau mengukur langsung amplitudo gelombangnya. Tetapi, probabilitas di definisikan dalam nilai mutlak dari kuadrat



84



amplitudo; karena hasilnya selalu merupakan suatu bilangan real, maka kita tidak sulit menafsirkannya. Meskipun ampitudo gelombang deBroglie tidak mudah di tafsirkan, gelombang deBroglie memiliki ciri khas dari sebuah gelombang klasik yang berperilaku baik. Sebagai contoh, ia dipantulkan dan di bahas, ia memenuhi asas superposisi, dan gelombang-gelombang deBroglie yang merambat dalam arah-arah yang berlawanan dapat berpadu membentuk sebuah gelombang berdiri.



85



Latihan Soal 1. Hitung panjang gelombang de Broglie sebuah benda bermassa 2 kg yang kecepatannya 25 m/det! 2. Tentukan potensial pemercepat yang diperlukan sehingga sebuah electron yang dipercepat memiliki panjang gelombang de Broglie 1Å, yang merupakan ukuran jarak antar atom dalam sebuah Kristal. 3. Hitung energy sebuah proton yang panjang gelombang de Broglienya 0,5 fm ! 4. Jika kita ingin mengamati sebuah obyek berukuran 2,5 Å, berapakah energy minimum foton yang dapat digunakan ? 5. Neutron yang dating pada sebuah Kristal natrium klorida ( jarak antar atomnya 2,81 Å ) mengalami difraksi orde pertama dari bidang-bidang Bragg utama pada sudut 200. Berapakah energy neutron termal ini ? 6. Sebuah berkas sempit electron 60 keV melewati sebuah lembaran tipis polikristalin perak. Jarak antaratom perak adalah 4,08 Å. Hitung jari-jari orde pertama pola difraksi, dari bidang-bidang Bragg, pada sebuah layar detector yang ditempatkan 40 cm di belakang lembaran ini! 7. Sebuah bahan Kristal memiliki sekumpulan bidang Bragg yang terpisah dengan jarak 1,1Å . Berapakah orde tertinggi pantulan Bragg, bagi neutron – neutron 2 eV ? 8. Kecepatan sebuah proton diukur hingga mencapai ketidakpastian 6 ×105 m/s. Dalam daerah sepanjang berapakah elektronnya masih mungkin ditemukan 9. Sebuah partikel memiliki energy diam 1385 MeV dan usia hidup 2,0 × 10-23 s. Bila energy diamnya diukur, berapakah rentang pengukuran khas yang akan diperoleh ? 10. Sebuah partikel memiliki energy diam 1236 MeV dengan ketidakpastian pengukurannya secara eksperimental sebesar 120 MeV. Berapakah usia hidup minimumnya ?



86



4 Persamaan Schrödinger



Persamaan Schrodinger diajukan pada tahun



1925



oleh



fisikawan



Erwin



Schrodinger (1887-1961). Persamaan ini pada awalnya merupakan jawaban dari dualitas partikel-gelombang yang lahir dari gagasan de Broglie yang menggunakan persamaan kuantisasi cahaya Planck dan prinsip fotolistrik Einstein untuk melakukan kuantisasi pada orbit electron.



87



Bab 4 Persamaan Schrödinger



Persamaan Schrodinger diajukan pada tahun 1925 oleh fisikawan Erwin Schrodinger (18871961). Persamaan ini pada awalnya merupakan jawaban dari dualitas partikel-gelombang yang lahir dari gagasan de Broglie yang menggunakan persamaan kuantisasi cahaya Planck dan prinsip fotolistrik Einstein untuk melakukan kuantisasi pada orbit elektron. Selain Schrodinger, dua orang fisikawan lainnya yang mengajukan teorinya masing-masing adalah Werner Heisenberg dengan Mekanika Matriks dan Paul Dirac dengan Aljabar Kuantum. Ketiga teori ini merupakan tiga teori kuantum lengkap yang berbeda dan dikerjakan terpisah namun ketiganya setara. Teori Schrodinger kemudian lebih sering digunakan karena rumusan matematisnya yang relatif lebih sederhana. Meskipun banyak mendapat kritikan persamaan Schrodinger telah diterima secara luas sebagai persamaan yang menjadi postulat dasar mekanika kuantum. Persamaan Schrodinger merupakan persamaan pokok dalam mekanika kuantum – seperti halnya hukum gerak kedua yang merupakan persamaan pokok dalam mekanika Newton – dan seperti persamaan fisika umumnya persamaan Schrodinger berbentuk persamaan diferensial. Dalam kasus fisika kuantum takrelativistik, diferensial yang digunakan persamaan Schrodinger adalah diferensial parsial orde kedua. Berbeda dari hukum Newton, pemecahan persamaan Schrödinger, yang disebut fungsi gelombang, memberikan informasi tentang perilaku gelombang dari partikel. 4.1. Pembenaran Persamaan Schrodinger Baik hukum Newton, persamaan Maxwell, maupun persamaan Schrodimger tidak dapat diturunkan dari seperangkat asas dasar, namun pemecahan yang diperoleh darinya ternyata sesuai dengan pengamatan percobaan. Persamaan Schrodinger hanya dapat dipecahkan secara eksak untuk beberapa potensial sederhana tertentu; yang paling sederhana adalah potensial konstan dan potensial osilator harmonik. Kedua kasus sederhana ini memang tidak “fisis,” dalam artian bahwa pemecahannya tidak dapat diperiksa kebenarannya dengan percobaan-tidak ada contoh di alam yang berkaitan dengan gerak sebuah pertikel yang terkukung dalam sebuah kotak satu dimensi, ataupun sebuah osilator harmonik mekanika kuantum ideal (meskipun kasus seperti ini seringkali merupakan hampiran yang cukup baik bagi situasi fisis yang sebenarnya). Namun demikian, berbagai kasus sedrhana ini cukup bermanfaat dalam memberikan gambaran tentang teknik umum pemecahan persamaan Schrodinger yang akan dibahas dalam bab ini. Untuk menjadi persamaan Schrodinger, diperlukan beberapa kriteria. Berikut adalah kriteria agar sebuah persamaan dapat disebut sebagai persamaan Schrodinger. 88



1. Tidak melanggar hukum kekekalan energi. Dalam pembahasan ini, hanya akan disinggung mengenai hukum kekekalan energi non relativistik. Oleh karena itu, K  V  E . 2. Bentuk persamaan diferensial apa pun yang kita tulis, haruslah taat asas terhadap hipotesis deBrogile-jika kita pecahkan persamaan matematikanya bagi sebuah partikel dengan momentum p, maka pemecahan yang kita dapati haruslah berbentuk sebuah fungsi gelombang dengan sepanjang gelombang λ yang sama dengan h/p. dengan menggunakan persamaan p = hk, maka enrgi kinetic dari gelombang deBrogile partikel bebas haruslah K = p²/2m = ђ²k²/2m. 3. Persamaannya “berperilaku baik”. Maksut baik disini antara lain: a. Fungsi bernilai tunggal (tidak boleh ada dua probabilitas untuk menemukan partikel di satu titik yang sama) b. Bernilai hingga (terukur) c. Fungsinya kontinu (berubah secara terus menerus hingga tidak sadar bahwa suatu saat nilai tersebut akan habis/diskrit/grafik tidak terputus) Dengan mendefinisikan 𝜓(x) sebagai 𝜓(x,t=0), maka 𝜓(x) = A sin kx Persamaan haruslah mengandung potensial V, jika V yang muncul berpangkat satu, maka agar taat asas dengan kekekalan energi (V + K = E). K harus pula muncul dalam bentuk pangkat satu. Telah didapati K = h2k22m, sehingga satu-satunya cara untuk memperoleh suku yang mengandung k2 adalah dengan mengambil turunan kedua dari 𝜓(x) = A sin kx terhadap x. 𝑑2 𝜓 𝑑𝑥 2



= -k2 𝜓 = -



2𝑚 ℎ2



ℎ2 𝑑2 𝜓 𝑑𝑥 2



− 2𝑚 ℎ2 𝑑2 𝜓 𝑑𝑥 2



Persamaan − 2𝑚



K𝜓=-



2𝑚 ℎ2



(E-V) 𝜓



+V𝜓=E𝜓



+ V 𝜓 = E 𝜓 adalah persamaan Schrödinger bebas waktu satu dimensi



𝑖ℏ



𝜕ᴪ ℏ2 𝜕 2 ᴪ 𝜕 2 ᴪ 𝜕 2 ᴪ =− + + ( )+𝑉 𝜕𝑡 2𝑚 𝜕𝑥 2 𝜕𝑦 2 𝜕𝑧 2 𝜕ᴪ



Persamaan di atas, dengan 𝑖ℏ 𝜕𝑡 adalah fungsi Schrodinger yang mendefinisikan partikel yang bergerak dalam tiga dimensi dengan energi tertentu dan berada di bawah pengaruh medan potensial V tertentu. Bentuk khusus persamaan Schrodinger yaitu persamaan Schrodinger bebas waktu tiga dimensi adalah,



89



𝜕 2 ᴪ 𝜕 2 ᴪ 𝜕 2 ᴪ 2𝑚 (𝐸 − 𝑉)ᴪ = 0 + + + 𝜕𝑥 2 𝜕𝑦 2 𝜕𝑧 2 ℏ2 Bentuk ini lebih sering digunakan karena energi dan medan potensial sistem fisika umumnya hanya bergantung pada posisi. Walaupun rumusan matematis persamaan Schrodinger lebih sederhana dibandingkan Mekanika Matriks dan Aljabar Kuantum, pemecahan persamaan ini tetap membutuhkan pengetahuan matematika lanjut. Berikut adalah langkah-langkah untuk menemukan persamaan Schrodinger suatu fungsi. 1. Langkah pertama yang dilakukan adalah menentukan energi kinetik dan potensial sistem dan mensubstitusikannya ke dalam persamaan −



ℎ2 𝑑2 𝜓 2𝑚 𝑑𝑥 2



+ V 𝜓 = E 𝜓.



2. Langkah kedua adalah merubah persamaan di atas kedalam sistem koordinat yang sesuai dengan sistem yang ditinjau. Untuk sistem atom hidrogen sistem koordinat yang sesuai adalah sistem koordinat bola. 3. Langkah ketiga adalah melakukan pemisahan variabel. Persamaan Schrodinger mengandung tiga koordinat ruang yang saling ortogonal dan harus dipisahkan menjadi 3 persamaan berbeda yang hanya mengandung satu koordinat ruang. 4. Langkah keempat adalah memecahkan ketiga persamaan tersebut secara simultan. Hasil yang diperoleh merupakan bilangan-bilangan kuantum yang memerikan struktur sistem berdasarkan tingka-tingkat energi yang menyusun sistem tersebut. Struktur sistem ini selanjutnya dipergunakan untuk meramalkan perilaku sistem dan interaksinya dengan sistem lain. Penerapan persamaan Schrodinger pada sistem fisika memungkinkan kita mempelajari sistem tersebut dengan ketelitian yang tinggi. Penerapan ini telah memungkinkan perkembangan teknologi saat ini yang telah mencapai tingkatan nano. Penerapan ini juga sering melahirkan ramalan-ramalan baru yang selanjutnya diuji dengan eksperimen. Penemuan positron – yang merupakan anti materi dari elektron – adalah salah satu ramalan yang kemudian terbukti. Perkembangan teknologi dengan kecenderungan alat yang semakin kecil ukurannya pada gilirannya akan menempatkan persamaan Schrodinger sebagai persamaan sentral seperti halnya yang terjadi pada persamaan Newton selama ini. Contoh soal: Sebuah benda bermassa m dijatuhkan dari ketinggian H di atas sebuah tangki air. Ketika memasuki air, ia mengetahui gaya apung B yang lebih besar daripada beratnya. (Kita abaikan viskos (gesek) oleh air pada benda). Carilah perpindahan dan kecepatan benda, dihitung dari saat dilepaskan hingga ia muncul kembali ke permukaan air.



90



Pemecahan: Kita pilih sebuah koordinat dengan y positif ke atas, dan mengambil y = 0 pada permukaan air. Selama benda jatuh bebas, ia hanya dipengaruhi gaya gravitasi. Maka, dalam daerah 1 (di atas air), hukum kedua Newton memberikan −mg = m



𝑑2 𝑦 𝑑𝑡 2



yang memiliki pemecahan 𝑣1 (t) = 𝑣𝑜1 – gt 1



𝑦1 (t) = 𝑦𝑜1 + 𝑣𝑜1 t + 2gt2 𝑣𝑜1 dan 𝑦𝑜1 adalah kecepatan dan ketinggian awal pada saat t=0. Ketika benda memasuki air, gayanya menjadi B – mg, sehingga hukum kedua Newton menjadi B – mg = m



𝑑 2 𝑦2 𝑑𝑡 2



yang memiliki pemecahan 𝐵



𝑣2 (t) = 𝑣𝑜2 + (𝑚 – g) t 1 𝐵



𝑦2 (t) = 𝑦𝑜2 + 𝑣𝑜2 t + 2 (𝑚 – g) t2 Keempat pemecahan ini memiliki empat koefisien tidak tertentukan: 𝑣𝑜1 , 𝑦𝑜1 , 𝑣𝑜2 , dan 𝑦𝑜2 . (Perhatikan bahwa 𝑣𝑜2 dan 𝑦𝑜2 bukanlah nilai pada saat t=0, tetapi tetapan yang akan ditentukan kemudian). Kedua tetapan pertama diperoleh dengan menerapkan syarat awal-pada saat t=0 (ketika benda dilepaskan) 𝑦𝑜1 = H dan 𝑣𝑜1 = 0, karena benda dilepaskan dari keadaan diam. Oleh karena itu, pemecahan dalam daerah 1 adalah 𝑣1 (t) = – gt 1



𝑦1 (t) = H - 2gt2 Langkah berikut adalah menerapkan syarat batas pada permukaan air. Misalkan t1 adalah saat ketika benda memasuki air. Syarat batasnya menghendaki bahwa v dan y kontinu pada daerah batas antara air dan udara, yakni: 𝑦1 (𝑡1 )= 𝑦2 (𝑡1 ) dan 𝑣1 (𝑡1 )= 𝑣2 (𝑡1 ) Persyaratan pertama mengatakan bahwa bendanya tidak lenyap pada suatu saat tertentu dan kemudian muncul kembali di suatu titik lain pada saat berikutnya. Persyaratan kedua setara dengan mensyaratkan lajunya berubah secara mulus pada permukaan air. (Jika syarat ini tidak dipenuhi, maka 𝑣1 (𝑡1 - ∆𝑡) ≠ 𝑣2 (𝑡1 - ∆𝑡) meskipun ∆𝑡 → 0, sehingga percepatan akan menjadi tak terhingga). Untuk menerapkan syarat batas ini, kita harus terlebih dahulu mencari 𝑡1 , yang diperoleh dengan mencari waktu 𝑡1 , ketika 𝑦1 menjadi nol. 1



𝑦1 (𝑡1 ) = H - 2g𝑡1 2 = 0 91



sehingga 2𝐻



𝑡1 = √ 𝑔



Dengan demikian, laju benda ketika menyentuh air, 𝑣1 (𝑡1 ) adalah 2𝐻



𝑣1 (𝑡1 ) = - g𝑡1 = - g √ 𝑔 = - √2𝑔𝐻 Maka syarat batas memberikan 2𝐻



1



𝐵



2𝐻



𝑦2 (𝑡1 ) = 𝑦𝑜1 + 𝑣𝑜1 √ 𝑔 + 2 (𝑚 – g)√ 𝑔 = 0 dan 𝐵



2𝐻



𝑣2 (𝑡1 ) = 𝑣𝑜1 + (𝑚 – g)√ 𝑔 = - √2𝑔𝐻 Kedua persamaan ini dapat dipecahkan secara serempak untuk memperoleh 𝑦𝑜2 dan 𝑣𝑜2 , yang menghasilkan 𝑣𝑜2 = -(B/m) √2𝐻/𝑔 dan 𝑦𝑜2 = H(1+B/mg). Jadi, pemecahan lengkap dalam daerah 2 adalah 𝑣2 (𝑡) = − 𝐻𝐵



𝐵 𝑚



𝑦2 (𝑡) = 𝐻 + 𝑚𝑔 –



𝐵 𝑚



2𝐻



𝐵



√ 𝑔 + (𝑚 – g) t 2𝐻



1



𝐵



√ 𝑔 t + 𝑣2 (𝑡1 ) = 2 (𝑚 – g)t2



Persamaan bagi 𝑣1 , 𝑦1 , 𝑣2 , dan 𝑦2 memberikan perilaku gerak benda dari saat t = 0 hingga ia muncul kembali ke permukaan air. Hasil-hasil ini dapat kita terapkan untuk menghitung sifat gerak lainnya; sebagai contoh, kita dapat mencari kedalaman maksimum yang dicapai benda, yang terjadi ketika 𝑣2 = 0. Jika kita ambil 𝑡2 sebagai waktu pada saat hal ini terjadi, maka 𝑣2 (𝑡2 ) = −



𝐵 𝑚



2𝐻



𝐵



√ 𝑔 + (𝑚 – g) 𝑡2 = 0



𝑡2 =



𝐵 𝐵−𝑚𝑔



2𝐻



√𝑔



Kedalaman D adalah nilai 𝑦2 pada saat 𝑡2 ini, yaitu 𝐻𝐵



𝐵



2𝐻



1



𝐵



−𝑚𝑔𝐻



D = 𝑦2 (𝑡2 ) = (H + 𝑚𝑔) – 𝑚 √ 𝑔 𝑡2 + 2 (𝑚 – g) 𝑡2 2 = 𝐵−𝑚𝑔 4.2.



Probabilitas dan Normalisasi



Arti dari fungsi gelombang 𝜓(x) belum seluruhnya jelas. Fungsi 𝜓(x) menyatakan suatu gelombang dalam pengertian yang lumrah, artinya, ia memiliki panjang gelombang dan bergerak dengan kecepatan fase yang jelas. Namun pertanyaan akan muncul ketika hendak menafsirkan amplitudonya. Apakah yang dinyatakan oleh amplitudo 𝜓(x), dan variabel fisika apakah yang bergetar? Karena pada dasarnya suatu fungsi gelombang 𝜓(x) tidak memiliki arti fisis. Amplitudo fungsi gelombang 𝜓(x) sudah tentu bukanlah perpindahan, seperti pada gelombang air atau senar piano, juga 92



bukan gelombang tekanan seperti pada gelombang bunyi. Jelas, ia merupakan suatu jenis gelombang yang berbeda, yang nilai mutlaknya memberikan probabilitas untuk menemukan partikelnya pada suatu titik tertentu. Maka dari itu rapat probabilitas |𝜓|2 dx diperlukan untuk dapat mengetahui sifat-sifat partikel dalam fungsi gelombang. Untuk sekarang, aturan ini diterima dalam pengertian bahwa sebuah partikel tunggal dalam ruang tidak memiliki dimensi fisika; karena dimensi sebuah titik dalam ruang adalah nol, maka probabilitas untuk menemukan sebuah partikel di sebuah titik adalah selalu nol, tetapi untuk selang dx, probabilitasnya tidak nol. Jika mendefinisikan P(x) sebagai rapat probabilitas (probabilitas per satuan panjang, dalam ruang satu dimensi), maka tafsiran 𝜓(x) menurut resep Schrödinger adalah



P(x) dx = |𝜓(x)|2 dx Tafsiran |𝜓|2 ini membantu untuk memahami persyaratan kontinu 𝜓(x); tidak menghendaki probabilitasnya berubah secara takkontinu, tetapi seperti halnya dengan semua gelombang lainnya yang berperilaku baik, amplitudonya hendaklah berubah secara tidak mencolok dan kontinu. Probabilitas untuk menemukan partikel antara 𝑥1 dan 𝑥2 adalah jumlah semua probabilitas P(x) dx dalam selang infinitesimal antara 𝑥1 dan 𝑥2 , yang tentu saja adalah suatu integral: Probabilitas untuk menemukan partikel antara 𝑥1 dan 𝑥2 = 𝑥



𝑥



1



1



∫𝑥 2 𝑃(𝑥)𝑑𝑥 = ∫𝑥 2|𝜓(𝑥)|2 𝑑𝑥 Dari aturan ini, diperoleh dalil berikut, bahwa probabilitas untuk menemukan partikel di suatu titik sepanjang sumbu x, adalah 100 persen, sehingga berlaku +∞



∫−∞ |𝜓(𝑥)|2 𝑑𝑥 = 1 Persamaan di atas dikenal sebagai syarat normalisasi. Perhatikan bahwa tetapan A tidak tertentukan dari pemecahan persamaan diferensialnya; dan ternyata, selama persamaan Schrödinger linear, maka jika 𝜓(x) adalah pemecahannya, hasil kali 𝜓(x) dengan sebarang tetapan juga merupakan pemecahan. Sebuah fungsi gelombang yang tetapan pengalinya ditentukan menurut persamaan diatas dikatakan ternormalisasikan; jika tidak, ia dikatakan 𝑥



tidak ternormalisasikan. Jika normalisasinya telah dilakukan secara tepat, maka persamaan ∫𝑥 2 𝑃(𝑥)𝑑𝑥 1



=



𝑥 ∫𝑥 2|𝜓(𝑥)|2 𝑑𝑥 1



akan selalu menghasilkan suatu probabilitas yang terletak antara 1 dan 0.



Setiap pemecahan persamaan Schrödinger yang menghasilkan |𝜓(𝑥)|2 bernilai takhingga, harus dikesampingkan-tidak pernah titik mana pun. Dalam praktik, “mengesampingkan” suatu pemecahan dengan mengembalikan faktor pengalinya sama dengan nol. Sebagai contoh, jika 93



pemecahan matematika bagi persamaan diferensialnya menghasilkan 𝜓(x) = A𝑒 𝑘𝑥 + B𝑒 −𝑘𝑥 bagi seluruh daerah x > 0, maka harus disyaratkan A = 0 agar pemecahannya mempunyai makna fisika; jika tidak, |𝜓(𝑥)| akan menjadi takhingga untuk x menuju takhingga. (Tetapi, jika berlakunya pemecahan ini dibatasi dalam selang 0 < x < L, maka tidak boleh mengambil A=0). Jika pemecahannya dikehendaki berlaku pada seluruh daerah negatif sumbu x < 0, maka harus mengambil B = 0. Jika menghitung probabilitas yang berkaitan dengan setiap koordinat, maka dapat ditemukan hasil yang mungkin dari suatu pengukuran satu kali atau rata-rata hasil dari sejumlah besar pengukuran berkali-kali. Sebagai contoh, andaikanlah ingin untuk mencari rata-rata kedudukan sebuah partikel dengan mengukur koordinatnya x-nya. Dengan melakukan sejumlah besar pengukuran berkali-kali, didapati bahwa dengan mengukur nilai 𝑥1 sebanyak 𝑛1 kali, 𝑥2 sebanyak 𝑛2 kali, dan seterusnya, maka dengan cara yang lazim, didapati nilai rata-ratanya, yaitu 𝑥𝑎𝑣 =



𝑛1 𝑥1 + 𝑛2 𝑥2+⋯ 𝑛1 + 𝑛2 +⋯



=



∑ 𝑛𝑖 𝑥𝑖 ∑ 𝑛𝑖



Jika mengetahui probabilitas untuk menemukan partikel pada setiap titik 𝑥𝑖 , maka 𝑛𝑖 berkaitan dengan P(𝑥𝑖 ), sehingga dengan mengubah penjumlahannya menjadi integral, diperoleh +∞



∫−∞ 𝑃(𝑥)𝑥𝑑𝑥



𝑥𝑎𝑣 =



+∞



∫−∞ 𝑃(𝑥)𝑑𝑥



dengan demikian, +∞



𝑥𝑎𝑣 = ∫



|𝜓(𝑥)|2 𝑥𝑑𝑥



−∞



Langkah terakhir dapat dilakukan jika fungsi gelombangnya ternormalisasikan, karena dengan +∞



demikian penyebut dari ∫−∞ 𝑃(𝑥)𝑑𝑥 sama dengan satu. Dengan cara yang sama, nilai rata-rata sebarang fungsi dari x dapat dicari sebagai berikut: +∞



[𝑓(𝑥)]𝑎𝑣 = ∫−∞ |𝜓(𝑥)|2 𝑓(𝑥)𝑑𝑥 Nilai



rata-rata



yang



dihitung



menurut



+∞



𝑥𝑎𝑣 = ∫−∞ |𝜓(𝑥)|2 𝑥𝑑𝑥



dan



[𝑓(𝑥)]𝑎𝑣



=



+∞



∫−∞ |𝜓(𝑥)|2 𝑓(𝑥)𝑑𝑥 dikenal sebagai nilai ekspektasi (expectation values). Contoh soal: Sebuah elektron terperangkap dalam suatu daerah satu dimensi sepanjang 1,0x10-10 m (diameter khas atomik). (a) berapa banyak energi yang harus dipasok untuk mengeksitasi elektron dari keadaan dasar ke ekadaan eksitasi pertama? (b) pada keadaan dasar, berapakah probabilitas untuk menemukan elektron dalam daerah dari x=0,090x10-10 m hinggan 0,110x10-10 m?



94



Pemecahan: (a) pada keadaan dasar, energinya adalah E0 .







 



 2 2 1,05 x10 34 J .s 3,14 E0    6,0 x10 18 J  37ev 2 2  31  10 2mL 2 9,1x10 kg 10 m 2







2







Pada keadaan eksitasi pertama, energinya adalah 4 E0 . Jadi, beda energi yang harus dipasok adalah 3



E0 atau 111 ev.



(b)



𝑥 ∫𝑥 2 𝑃(𝑥)𝑑𝑥 1



=



𝑥 ∫𝑥 2|𝜓(𝑥)|2 𝑑𝑥 1



x2



2 2 x 2x  x 1   sin 2 dx    sin   0,0038  0,38% L x1 L L  x1  L 2 x



4.3. Beberapa Penerapan Persamaan Schrodinger dapat diterapkan dalam berbagai persoalan fisika. Dimana pemecahan persamaan Schrodinger yang disebut fungsi gelombang, memberikan informasi tentang perilaku gelombang dari partikel. Partikel Bebas Yang dimaksudkan dengan sebuah “partikel bebas” adalah sebuah partikel yang bergerak tanpa dipengaruhi gaya apa pun dalam suatu bagian ruang; yaitu, F = 0, sehingga V(x) = tetapan, untuk semua x. ℎ2 𝑑2 𝜓 𝑑𝑥 2



Berikut diterapkan resepnya, dengan menuliskan kembali Persamaan − 2𝑚



+ V 𝜓 = E 𝜓 dengan



potensial yang sesuai (V = 0); ℎ2 𝑑2 𝜓 𝑑𝑥 2



− 2𝑚



=E𝜓



atau 𝑑2 𝜓 𝑑𝑥 2



= -k2 𝜓



di mana k2 = Persamaan



𝑑2 𝜓 𝑑𝑥 2



2𝑚𝐸 ħ2



= -k2 𝜓 adalah bentuk persamaan yang telah lazim dikenal; dengan k2 selalu positif,



maka pemecahannya adalah



95



𝜓(x) = A sin kx + B cos kx Dari persamaan k2 =



2𝑚𝐸 , ħ2



didapati bahwa nilai energi yang diperkenankan adalah: E=



Perhatikan bahwa persamaan E =



ħ2 k2 2𝑚



ħ2 k2 2𝑚



tidak lain adalah energi kinetik sebuah partikel dengan



momentum p = ħk atau, setara dengan ini, p=h/λ; telah membentuk persamaan Schrödinger yang menghasilkan pemecahan bagi partikel bebas yang berkaitan dengan satu gelombang deBroglie. Partikel dalam Sumur Potensial Sumur potensial adalah yang tidak mendapat pengaruh potensial. Hal ini berarti bahwa partikel selama berada dalam sumur potensial, merupakan electron bebas. Kita katakana bahwa electron terjebak di sumur potensial, dan kita anggap bahwa dinding potensial sangat tinggi menuju ∞, atau kita katakana sumur potensial sangat dalam. Dalam gambar (5.1) berikut kita akan menggambarkan sumur potensial. Daerah I dan daerah II adalah daerah-daerah dengan V = ∞, sedangkan di daerah II, yaitu antara 0 dan L, V =. Kita katakana bahwa lebar sumur potensial ini adalah L. 0≤ 𝑥 ≤ 𝐿



V(x) = 0, V(x) = ∞



x< 0, 𝑥 > 𝐿,



Gambar 4.1 partikel dalam sumur potensial daerah II Pada sumur potensial yang dalam, daerah I dan III adalah daerah dimana kemungkinan berada electron bisa dianggap nol, Ψ1(x) = 0 dan Ψ2(x) = 0. Sedangkan pada daerah dua Kita dapat member spesifikasi pada gerak partikel = 0 dan x = L disebabkan oleh dinding keras tak berhingga. Sebuah partikel tidak akan kehilangan Energinya jika bertumbukan dengan dinding, energy totalnya tetap konstan. Dari pernyataan tersebut maka enrgi potensial V dari partikel itu menjadi tak hingga di kedua sisi sumur, sedangkan V konstan di dalam sumur, dapat dikatakan V memiliki Energi tak hingga, maka partikel tidak mungkin ditemukan di luar sumur, sehingga fungsi gelombang Ψ = 0 untuk 0≤ 𝑥 ≤ 𝐿. Maka yang perlu dicari adalah nilai Ψ di dalam sumur, yaitu antara x = 0 dan x = L. persamaan Schrodinger bebas waktu adalah : ℎ² 𝑑²



− 2𝑚 𝑑𝑥² 𝜑n = En𝜑n Dengan 96



𝑑²𝜑 = −𝑘²𝜑 𝑑𝑥² Dimana k=



√2𝑚𝐸𝑛 ℎ



sesuai dengan persamaan gelombang maka : Ψ(x) = A sin kx + B cos kx Pemecahan ini belum lengkap, karena belum ditentukan nila A dan B, juga belum menghitung nilai energy E yang diperkenankan. Untuk menghitungnya, akan diterapkan persyaratan bahwa Ψ(x) harus kontinu pada setiap batas dua bagian ruang. Dalam hal ini akan dibuat syarat bahwa pemecahan untuk x < 0 𝑑𝑎𝑛 𝑥 > 0 bernilai sama di x = 0. Begitu pula pemecahan untuk x > 𝐿 𝑑𝑎𝑛 𝑥 < 𝐿 haruslah bernilai sama di x = L. jika x =0, untuk x < 0 jadi harus mengambil Ψ(x) = 0 pada x = 0. Ψ(0) = A sin 0 + B cos 0 Ψ(0) = 0 + B.1 = 0 Jadi, didapat B = 0. Karena Ψ = 0 untuk x > 𝐿, maka haruslah berlaku Ψ(L) = 0, Ψ(L) = A sin kL + B cos kL = 0 Karena telah didapatkan bahwa B = 0, maka haruslah berlaku: A sin kL = 0 Disini ada dua pemecahan yaitu A = 0, yang memberikan Ψ(x) = 0 dan Ψ²(x) = 0, yang berarti bahwa dalam sumur tidak terdapat partikel (Pemecahan tidak masuk akal) atau sin kL = 0, maka yang benar jika: kL = 𝜋, 2𝜋. 3𝜋, … . 𝑛 = 1,2,3 …. dengan : k=



√2𝑚𝐸𝑛 ℎ



=



𝑛𝜋 𝐿



dari kedua persamaan di atas dan diperoleh bahwa energi partikel mempunyai harga tertentu yaitu harga eigen. Harga eigen ini membentuk tingkat energisitas yaitu: En =



𝑛²𝜋²ħ² 2𝑚𝐿²



Dimana enrgi yang kita tinjau disini berbeda dengan energy Born dimana pada energy Born menyatakan enrgi tingkat atomic sedangkan tingkat energy pada persamaan Schrodinger menyatakan tingkat energy untuk electron. Fungsi gelombang sebuah partikel di dalam sumur yang berenrgi En ialah: Ψn = A sin



√2𝑚𝐸𝑛 ħ



x



Untuk memudahkan E1 = ħ²𝜋²/2𝑚𝐿², yang mana tampak bahwa unit energy ini ditentukan oleh massa partikel dan lebar sumur. Maka E = n²E1 dan seterusnya. Karena dalam kasus ini energy yang diperoleh hanya laju tertentu yang diperkenenkan dimiliki partikel. Ini sangat berbeda dengan kaasus klasik, 97



misalnya manic-manik (yang meluncur tanpa gesekan sepanjang kawat dan menumbuk kedua dinding secara elastic) dapat diberi sembarang kecepatan awal dan akan bergerak selamanya, bolak-balik, dengan laju tersebut. Dalam kasus kuantum, hal ini tidaklah mungkin, karena hanya laju awal tertentu yang dapat memberikan keadaan gerak tetap, keadaan gerak khusus ini disebut keadaan stasioner (disebut keadaan “stasioner” karena ketergantungan pada waktu yang dilibatkan untuk membuat Ψ(x,t), |𝛹(𝑥, 𝑡)|² tidak bergantung waktu). Hasil pengukuran energy sebuah partikel dalam sebuah sumur potensial harus berada pada salah satu keadaan stasioner, hasil yang lain tidaklah mungkin. Pemecahan bagi Ψ(x) belum lengkap, karena belum ditentukan tetapan A. untuk menentukannya, ditinjau kembali persyaratan +∞



normalisasi, yaitu ∫−∞ |𝛹(𝑥)|² 𝑑𝑥 = 1. karena Ψ(x) = 0 Kecuali untuk 0≤ 𝑥 ≤ 𝐿 𝑠𝑒ℎ𝑖𝑛𝑔𝑔𝑎 𝑏𝑒𝑟𝑙𝑎𝑘𝑢 : 𝐿



∫ |𝐴2 | 𝑠𝑖𝑛2 (𝑘𝐿)𝑑𝑥 = 1 0



Maka diperoleh A = √2/𝐿 . dengan demikian, pemecahan lengkap bagi fungsi gelombang untuk 0≤ 𝑥 ≤ 𝐿 𝑎𝑑𝑎𝑙𝑎ℎ ∶ 2 𝐿



Ψn = √ sin



𝑛𝜋𝑥 𝐿



n = 1,2,3…



Dalam gambar 4.2 dan 4.3 akan dilukiskan berbagai tingkat energy, fungsi gelombang dan rapat probalitas |𝛹|² yang mungkin untuk beberapa keadaan terendah. Keadaan energy terendah, yaitu pada n=1, dikenal sebagai keadaan dasar dan keadaan dengan energy yang lebih tinggi (n> 1) dikenal sebagai keadaan aksitasi.



Gambar 4.2 tingkat energy dalam sumur secara konstan



98



Gambar 4.3 probalitas keberadaan electron dalam sumur potensial Kita lihat disini bahwa energy electron mempunyai nilai-nilai tertentu yang diskrit, yang ditentukan oleh bilangan bulat n, Nilai diskrit ini terjadi karena pembatasan yang harus dialami oleh Ψ2 yaitu bahwa ia harus berada dalam sumur potensial. Ia harus bernilai nol di batas-batas dinding potensial dan hal itu akan terjadi bila lebar sumur potensial L sama dengan bilangan bulat kali setengah panjang gelombang. Jika tingkat energy untuk n = 1 kita sebut tingkat energy yang pertama, maka tingkat energy yang kedua pada n=2, tingkat energy yang ketiga pada n=3 dan sterusnya. Jika kita kaitkan dengan bentuk gelombangnya, dapat kita katakana bahwa tingkat-tingkat energy tersebut sesuai dengan jumlah titik simpul gelombang. Dengan demikian maka diskritasi energy electron terjadi secara wajar melalui pemecahan persamaan Schrodinger. Persamaan En =



𝑛²𝜋²ħ² 2𝑚𝐿²



memperlihatkan bahwa selisih energy antara satu tingkat dengan tingkat



berikutnya, misalnya antara n=1 dan n=2, berbanding terbalik dengan kuadrat lebar sumur potensial. Makin lebar sumur ini, makin kecil selisih energy tersebut, artinya tingkat-tingkat energy semakin rapat. Untuk L sama dengan satu satuan misalnya, selisih energy untuk n=2 dan n=1 adalah E2 – E1 = 3ħ²/8m dan jika L 10 kali lebih lebar maka selisih ini menjadi E2-E1= 0,03ħ²/8m.



Gambar 4.4 Pengaruh lebar sumur terhadap energy Jadi makin besar L maka perbedaan nilai tingkat-tingkat energy akan semakin kecil dan untuk L semakin lebar maka tingkat-tingkat energy tersebut akan semakin rapat sehingga kontinyu. Contoh: Perlihatkan bahwa nilai rata-rata dari x adalah L/2 dan tidak bergantung pada keadaan kuantum. Pemecahan:



99



+∞



Menggunakan persamaan 𝑥𝑎𝑣 = ∫−∞ |𝜓(𝑥)|2 𝑥𝑑𝑥; karena 𝜓 = 0 kecuali untuk 0 ≤ x ≤ L, maka digunakan 0 dan L sebagai batas-batas integral, sehingga 𝑋𝑎𝑣 =



2 𝐿



𝐿



∫0 (𝑠𝑖𝑛2



𝑛𝜋𝑥 ) 𝐿



𝑥 𝑑𝑥



Bentuk ini dapat diintegralkan secara parsial, atau dicari pada tabel integral; hasil adalah 𝑋𝑎𝑣 =



𝐿 2



Perhatikan bahwa hasil ini tidak bergantung pada n. Jadi, pengukuran rata-rata kedudukan partikel tidak menghasilkan informasi mengenai keadaan kuantumnya.



Partikel dalam Sebuah Kotak (Dua Dimensi) Apabila tinjauan di depan diperluas ke kasus fisika dua dan tiga dimensi, ciri-ciri utama pemecahannya masih tetap sama, namun ada suatu ciri khas baru penting yang diperkenalkan. Jika potensial merupakan fungsi dari x dan y, maka 𝜓 harus pula bergantung pada x dan y dan turunan terhadap x, harus diganti dengan turunan terhadap x dan y. Karena itu, dalam dua dimensi diperoleh ħ2 𝜕 2 𝜓 (x, y) 𝜕 2 𝜓 (x, y) − + ( ) + 𝑉(𝑥, 𝑦)𝜓(𝑥, 𝑦) = 𝐸𝜓(𝑥, 𝑦) 2𝑚 𝜕𝑥 2 𝜕𝑦 2 “Kotak” dua dimensi sekarang dapat didefinisikan sebagai berikut: V(x,y) = 0 =∞



0 ≤ x ≤ L, 0 ≤ y ≤ L untuk yang lainnya



Pemecahan persamaan diferensial parsial memerlukan teknik yang lebih rumit daripada yang perlu di tinjau. Seperti kasus sebelumnya, dicurigai bahwa 𝜓(x,y) = 0 di luar kotak, agar probabilitas bernilai nol di sana. Di dalam kotak, di tinjau pemecahan-pemecahan yang terpisahkan (separable); artinya, fungsi dari x dan y yang ditinjau dapat dinyatakan sebagai hasil kali sebuah fungsi yang hanya bergantung pada y: 𝜓(𝑥, 𝑦) = 𝑓(𝑥) 𝑔(𝑦) bentuk masing-masing fungsi dari f dan g di ruas kanan sama seperti Persamaan 𝜓(x) = A sin kx + B cos kx f(x) = A sin 𝑘𝑥 𝑥 + B cos 𝑘𝑥 𝑥 g(y) = C sin 𝑘𝑦 𝑦 + D cos 𝑘𝑦 𝑦 Syarat kontinu pada 𝜓(𝑥, 𝑦) menghendaki bahwa pemecahan di luar dan di dalam kotak bernilai sama pada daerah batas kotak; jadi 𝜓 = 0 di x = L (untuk semua y) dan 𝜓 = 0 di y = L (untuk semua x). Persyaratan pada x = L menghendaki bahwa sin 𝑘𝑥 𝐿 = 0, sehingga merupakan 𝑘𝑥 𝐿 merupakan kelipatan bilangan bulat dari 𝜋. Semua bilangan bulat ini tidak perlu sama, karena itu masing-masing disebut 𝑛𝑥 dan 𝑛𝑦 untuk membantu membedakan mereka. Jadi, diperoleh: 𝜓(𝑥, 𝑦) = 𝐴′ sin



100



𝑛 𝜋𝑦 𝑛𝑥 𝜋𝑥 sin 𝑦 𝐿 𝐿



Hasilkali A dan C telah dinyatakan dengan 𝐴′ . Koefisien 𝐴′ , sekali lagi didapati dengan menggunakan syarat normalisasi, yang dalam dua dimensi menjadi ∬ 𝜓 2 𝑑𝑥 𝑑𝑦 = 1 Untuk kasus ini, syaratnya adalah 𝐿



𝐿



∫ 𝑑𝑦 ∫ 𝐴′2 𝑠𝑖𝑛2 0



0



𝑛𝑦 𝜋𝑦 𝑛𝑥 𝜋𝑥 𝑠𝑖𝑛2 𝑑𝑥 = 1 𝐿 𝐿



yang memberikan 𝐴′ = Terakhir



dengan



Persamaan −



ħ2 2𝑚



menyisipkan



𝜕2 𝜓 (x,y)



(



𝜕𝑥 2



+



kembali



𝜕2 𝜓 (x,y) 𝜕𝑦 2



2 𝐿



pemecahan



bagi



) + 𝑉(𝑥, 𝑦)𝜓(𝑥, 𝑦) = 𝐸𝜓(𝑥, 𝑦),



ψ(𝑥, 𝑦) maka



ini



ke



dalam



didapati



bahwa



energinya adalah: 𝐸=



ħ2 𝜋 2 (𝑛 2 + 𝑛𝑦 2 ) 2𝑚𝐿2 𝑥



Bandingkan hasil ini dengan Persamaan E =



ħ2 𝑘2 2𝑚



=



ħ2 𝜋 2 𝑛 2 . 2𝑚𝐿2



Dan jika E0 = ħ2 𝜋 2 /2𝑚𝐿2 maka 𝐸 = 𝐸0 (𝑛𝑥 2 + 𝑛𝑦 2 ).



Osilator Harmonik Sederhana Osilator klasik yang dapat ditinjau adalah benda bermassa 𝑚 yang diikatkan pada sebuah pegas dengan tetapan pegas k sehingga menderita gaya pegas 𝐹 = −𝑘𝑥, di mana x adalah perpindahan benda dari keadaan setimbang. Osilator seperti ini dapat di analisis dengan menggunakan hukum Newton yang mengungkapkan frekuensi 𝜔𝑜 = √𝑘/𝑚 dan periode 𝑇 = 2𝜋√𝑚/𝑘. Osilator harmonik ini memiliki energi kinetik maksimum di 𝑥 = 0; energi kinetiknya nol pada titik balik 𝑥 = ± 𝐴𝑜 , di mana 𝐴𝑜 amplitudo geraknya. Pada titik balik, osilator berhenti sejenak, kemudian berbalik arah geraknya. Tentu saja, geraknya terbatasi pada daerah − 𝐴𝑜 ≤ 𝑥 ≤ + 𝐴𝑜 . Mengapa sistem seperti ini di analisis dengan menggunakan mekanika kuantum? Meskipun dalam alam nyata tidak pernah menjumpai contoh osilator kuantum satu dimensi, terdapat sejumlah sistem yang berperilaku menghampiri sistem ini -- misalnya, vibrasi sebuah molekul diatomik (dua atom). Ternyata, hingga orde hampiran terendah, setiap sistem pada daerah minimum sebuah potensial berperilaku seperti sebuah osilator harmonik sederhana. Sebuah gaya 𝐹 = −𝑘𝑥 memiliki potensial 𝑉 = 1⁄2 𝑘𝑥 2 , jadi diperoleh Persamaan Schrödinger: −



ħ2 𝑑 2 𝜓 1 + 𝑘𝑥 2 𝜓 = 𝐸𝜓 2𝑚 𝑑𝑥 2 2



ħ2 𝑑 2 𝜓 𝑑𝑥 2



Semua pemecahan Persamaan − 2𝑚



+



1 2



𝑘𝑥 2 𝜓 = 𝐸𝜓 harus menuju nol bila 𝑥 → ±∞, dan untuk



limit 𝑥 → ±∞ perilakunya haruslah seperti eksponensial −𝑥 2 . Oleh karena itu, dicoba dengan 𝜓(𝑥) = 101



2



𝐴𝑒 −𝑎𝑥 , di mana 𝐴 dan 𝑎 adalah tetapan yang ditentukan dengan mengevaluasikan Persamaan −



ħ2 𝑑 2 𝜓 2𝑚 𝑑𝑥 2



+



1 2



𝑘𝑥 2 𝜓 = 𝐸𝜓 bagi pilihan 𝜓(𝑥) ini. Di mulai dengan mengevaluasi



𝑑2 𝜓 . 𝑑𝑥 2



𝑑𝜓 2 = −2 𝑎𝑥 (𝐴𝑒 −𝑎𝑥 ) 𝑑𝑥 𝑑2 𝜓 2 2 = −2𝑎 (𝐴𝑒 −𝑎𝑥 ). −2𝑎𝑥 (−2𝑎𝑥)𝐴𝑒 −𝑎𝑥 2 𝑑𝑥 dan kemudian menyisipkan 𝜓(𝑥) dan



𝑑2 𝜓 𝑑𝑥 2



ħ2 𝑑 2 𝜓 𝑑𝑥 2



ke dalam − 2𝑚



+



1 2



𝑘𝑥 2 𝜓 = 𝐸𝜓 untuk melihat apakah



pilihan ini memberikan suatu pemecahan. −



ħ2 1 2 2 2 2 (−2𝑎𝐴𝑒 −𝑎𝑥 + 4𝑎2 𝑥 2 𝐴𝑒 −𝑎𝑥 ) + 𝑘𝑥 2 (𝐴𝑒 −𝑎𝑥 ) = 𝐸𝐴𝑒 −𝑎𝑥 2𝑚 2 2



Pembagian dengan faktor sekutu 𝐴𝑒 −𝑎𝑥 memberikan ħ2 𝑎 2𝑎2 ħ2 2 1 − 𝑥 + 𝑘𝑥 2 = 𝐸 𝑚 𝑚 2 Selanjutnya karena kita mencari pemecahan yang berlaku bagi semua x, maka semua koefisien dari 𝑥 2 haruslah saling menghapuskan dan semua tetapan yang sisa haruslah sama. (Sebagai misal, tinjau 𝑏 𝑎



persamaan 𝑎𝑥 + 𝑏 = 0. Persamaan ini tentu saja berlaku bagi 𝑥 = − , tetapi bila menginginkan persamaan ini berlaku bagi sebarang dan semua x, maka haruslah persyaratan 𝑎 = 0 dan 𝑏 = 0). Jadi 2𝑎2 ħ2 1 − + 𝑘=0 𝑚 2 dan ħ2 𝑎 − =𝐸 𝑚 Yang menghasilkan 𝑎=



√𝑘𝑚 2ℏ



Dan 𝐸 = ½ℏ√𝑘⁄𝑚 Pernyataan energi ini dapat pula kita nyatakan dalam frekuensi klasik 𝜔0 = √𝑘⁄𝑚 sebagai 𝐸 = ½ℏ𝜔0 Pemecahan yang didapati ini berhubungan dengan keadaan dasar dari osilator. Pemecahan umum yang lebih sulit diperolehnya adalah persamaan yang berbentuk  n ( x)  Af n ( x)e ax , yang mana f n (x) 2



adalah sebuah polinomial dalam x dengan pangkat tertingginya xn. Energi yang bersangkutan adalah 𝐸 = (𝑛 + ½)ℏ𝜔0 Contoh: Sebuah osilator harmonik dua dimensi memiliki energi , di mana nx dan ny adalah bilangan bulat yang dimulai dari nol. (a) Ujilah kebenaran hasil ini dengan menggunakan energi osilasi satu dimensi. (b) 102



Sketsakan suatu diagram tingkat energi yang memperlihatkan nilai E dan bilangan kuantum nx dan ny. (c) Perlihatkan bahwa setiap energi ini terdegenerasi dengan derajat degenerasi sama dengan nx + ny + 1. Pemecahan: 1. a. Gerak x dan y tidak saling berhubungan dan keduanya mengontribusikan energi sebesar



0 n  12  tetapi nilai n tidak harus sama untuk dua gerak yang tidak berhubungan. Maka, 1 1   E  0  n x    0  n y    0 n x  n y  1 2 2   energi totalnya b.



c. Tingkat energi 𝑁ℏ𝜔0 memiliki perbedaan nilai N yang mungkin dalam nilai quantum n x, ny. Kedua nilai nx dan ny memiliki rentang dari 0 sampai N –1 tapi dengan jumlah nilai yang ditetapkan untuk N. Nilai yang mungkin dari nx adalah N, dan untuk setiap nilai nx adalah nilai ny telah ditetapkan. Total degenerasi untuk setiap tingkat menjadi N= nx + ny +1



4.4. Ketergantungan Pada Waktu ℎ2 𝑑2𝜓



Bila diketahui pemecahan tidak bergantung waktu  (x) dari persamaan − 2𝑚 𝑑𝑥 2 + V 𝜓 = E 𝜓 untuk energi E, maka fungsi gelombang bergantung waktunya 𝜓(𝑥, 𝑡), didapati menurut rumus



 ( x, t )   ( x)e it Frekuensi ω diberikan oleh hubungan deBroglie







E h



Untuk melihat bagaimana perkalian dengan e it memberikan suatu gelombang, kita tinjau bagaimana fungsi gelombang partikel bebas. Persoalan menjadi sederhana jikakita menuliskan kembali persamaan 𝜓(x) = A sin kx + B cos kx ke dalam bentuk eksponensial kompleks e ikx dan e ikx yang mana bentuknya adalah



 ( x)  A' e ikx  B' e ikx Tetapan A’ dan B’ dapat dicari dari tetapan A dan B. Jadi, bagi fungsi gelombang bergantung waktu yang bersangkutan, diperoleh



 ( x, t )  ( A' e ikx  B' e ikx )e it  A' e i ( kx t )  B' e  103



i ( kx t )



4.5 Potensial Tangga dan Halang Dalam jenis persoalan umum berikut, kita akan menganalisis apa yang terjadi apabila sebuah partikel yang sedang bergerak dalam suatu daerah berpotensial tetap tiba – tiba bergerak memasuki suatu daerah berpotensial berbeda yang juga tetap nilainya. Dalam bahasan ini kita akan mengambil E sebagai energy total (yang tetap) dari partikel dan V0 sebagai nilai energy potensial tetapnya. 1. Apabila E > V0, maka pemecahan persamaan Schrodingernya berbentuk



𝜓(𝑥) = 𝐴 sin 𝑘𝑥 + 𝐵 cos 𝑘𝑥 Dimana



𝑘=√



2𝑚 (𝐸 − 𝑉0 ) ħ2



A dab B adalah dua tetapan yang dapat ditentukan dari syarat normalisasi dan kekontinuan. Sebagai contoh, tinjau potensial tangga yang di perlihatkan pada Gambar 4.5



Gambar 4.5 Potensial tangga dengan tinggi v0 𝑉(𝑥) = 0



𝑥 V0, maka kita dengan mudah dapat menuliskan pemecahan persamaan Schrodinger dalam kedua daerah ini sebagai berikut :



𝜓0 (𝑥) = 𝐴 sin 𝑘0 𝑥 + 𝐵 cos 𝑘0 𝑥



2𝑚



𝑘0 = √ ħ2



𝑥0



Hubungan antara keempat tetapan A,B,C,dan D dapat dicari dengan menerapkan persyaratan bawa 𝜓(𝑥) dan 𝜓 ′ (𝑥) = 𝑑𝜓/𝑑𝑥 haruslah kontinu pada batas kedua daerah, jadi 𝜓0 (0) = 𝜓1 (0), 𝜓 ′ 0 (0) = 𝜓1′ (0). Kita juga dapat menggunakan persamaan 𝑒 𝑖𝜃 = cos 𝜃 + 𝑖 sin 𝜃 untuk mentransformasikan kedua pemecahan ini dari bentuk sinus dan kosinus ke dalam bentuk kompleks, yakni : 𝜓0 (𝑥) = 𝐴′ 𝑒 𝑖𝑘0 𝑥 + 𝐵′𝑒 −𝑖𝑘0 𝑥



𝑥0



104



Apabilla ketergantungan pada waktu dimaksukkan dengan mengalikan masing – masing suku dengan 𝑒 −𝑖𝜔𝑡 , maka kita dapat menafsirkan masing – masing gelombang ini. Ingatlah bahwa (𝑘𝑥 − 𝜔𝑡) adalah fase Gelombang yang bergerak dalam arah x positif, sedangkan (𝑘𝑥 + 𝜔𝑡) adalah fase Gelombang yang bergerak dalam arah x negative, dan bahwa kuadrat nilai mutlak dari tiap – tiap koefisien memberikan intensitas dari komponen Gelombang yang bersangkutan. Kita dapat menganalisis semua pemecahan di atas dari sudut pandang energy kinetic. Pada daerah dimana energy kinetic partikel adalah terbesar, momentum linear 𝑝 = √2𝑚𝐾 atau pula menjadi yang terbesar, dan panjang Gelombang deBroglie 𝜆 = ℎ/𝑝 akan menjadi yang terkecil. Jadi, panjang Gelombang deBroglie dalam daerah 𝑥 > 0 lebih kecil dari pada yang di dalam daerah 𝑥 < 0. 2. Apabila E < V0, maka kita peroleh pemecahan berbeda :



𝜓(𝑥) = 𝐴𝑒 𝑘𝑥 + 𝐵𝑒 −𝑘𝑥 Dimana



𝑘=√



2𝑚 (𝑉0 − 𝐸) ħ2



Jika daerah pemecahan ini meliputi dari +∞ atau -∞, kita harus menjaga agar 𝜓 tidak menjadi takhingga dengan menggambil A atau B sama dengan nol, jika daerahnya hanya mencakup koordinat x yang berhingga, hal ini tidak perlu dilakukan. Sebagai salah satu contohnya, jika dalam soal sebelumnya, E lebih kecil dari pada V 0, maka pemecahan bagi 𝜓1 menjadi 𝜓1 (𝑥) = 𝐶𝑒 𝑘1 𝑥 + 𝐷𝑒 −𝑘1 𝑥



𝑘1 = √



2𝑚 ħ2



(𝑉0 − 𝐸)



Secara klasik, partikelnya tidak pernah dapat ditemukan pada daerah 𝑥 > 0, karena energy totalnya tidak cukup untuk melampaui potensial tangga. Tetapi, mekanika kuantum memper Rapat probabilitas dalam daerah 𝑥 > 0 adalah |𝜓1 |², yang sebanding dengan 𝑒 −2𝑘1 𝑥 . Jika kita definisikan jarak terobosan Δ𝑥 sebagai jarak dari 𝑥 = 0 hingga ke titik dimana probabilitasnya menurun menjadi 1/e, maka 𝑒 −2𝑘1 𝑥 = 𝑒 −1 Δ𝑥 =



1 1 ħ = 2𝑘1 2 √2𝑚 (𝑣0 − 𝐸)



Agar partikel dapat memasuki daerah 𝑥 > 0, ia harus sekurang – kurangnya mendapat tambahan energy sebesar V0 – E agar dapat melampaui tangga potensial. Tentu saja, ini melanggar kekekalan energy bila partikel memperoleh sebarang tambahan energy secara tiba – tiba, tetapi menurut hubungan ketidakpastian ΔΕ ∆t ~ħ, kekekalan energy tidak berlaku pada selang waktu yang lebih kecil dari pada ∆𝑡 kecuai hingga suatu jumlah energy sebesar ΔΕ ~ħ/∆t. Energy harus kita kembaikan dalam selang waktu



105



∆𝑡 =



ħ 𝑉0 − 𝐸 + 𝐾



Karena partikel bergerak dengan laju v = √2𝐾/𝑚, maka jarak yang dapat ditempuhnya adalah 1 2𝐾 ħ ∆𝑥 = √ 2 𝑚 𝑉0 − 𝐸 + 𝐾 Dengan mendiferensiasikan persamaan di atas, maka nilai maksimum ini dapat kita cari yaitu 1 ħ ∆𝑥𝑚𝑎𝑘𝑠 = √ 2 2𝑚(𝑉0 − 𝐸)



Latihan Soal 1. Andaikan sebuah partikel yang berada di dalam sebuah sumur tak hingga satu dimensi



memancarkan foton ketika meloncat dari satu keadaan ke keadaan lain yang lebih rendah, tanpa ada batasan terhadap perubahan n – nya. Daftarkan semua energi foton yang mungkin, dunyatakan dalam E0 yang dipancarkan ketika sebuah partikel dalam sumur tak hingga satu dimensi berpindah dari keadaan dengan n = 4 keadaan dasar. 2. Gerakan sebuah elektron terbatasi dalam suatu daerah satu dimensi dalam mana energi



minimumnya adalah 1,0 eV. (a) Berapakah ukuran daerah tersebut? (b) Berapa banyak energi yang harus dipasok untuk menggerakkan elektron ke energi eksitasi pertamanya? (c) Dari suatu keadaan eksitasi tertentu, elektron melepaskan energi sebanyak 24,0 eV ketika berpindah ke keadaan dasar. Berapakah nilai bilangan kuantum n dari keadaan eksitasi tersebut? 3. (a) Sebuah partikel terperangkap dalam suatu daerah satu dimensi selebar L. Pada



keadaan eksitasi keduanya (n = 3) , perlihatkan bahwa probabilitas untuk menemukannya antara x=0 dan x=L/3 adalah 1/3. (b) Perlihatkan bahwa untuk keadaan dengan bilangan kuantum n , probabilitas untuk menemukannya adalah x = 0 dan x = L/ n adalah 1/n. 4. Berapakah energi minimum sebuah proton atau neutron yang terbatas geraknya dalam



ruang berukuran inti atom (1,0x10-14 m)? 5. Apakah tingkat berikutnya (di atas E = 50E0) dari sebuah partikel dalam potensial kotak



dua dimensi yang degenerasinya lebih besar daripada 2?



106



6. Perlihatkan dengan substitusi langung bahwa Persamaan (5.31) memberikan



pemecahan bagi persamaan Schrödinger dua dimensi, Persamaan (5.28). Carilah hubungan antara kx, ky, dan E. 7. Dengan menggunakan syarat normalisasi, perlihatkan bahwa untuk osilator harmonis



sederhana satu dimensi, tetapan A memiliki nilai (m0  )1 4 8. (a) Nilai berapakah yang diperkirakan bagi (p)av untuk osilator harmonik sederhana?



Pergunakan saja alasan simetri. (b) Kekekalan energi bagi osilator harmonik dapat digunakan untuk mengaitkan p2 dan x2. Pergunakan hasil ini, bersama dengan nilai (x2)av dari soal 19 , untuk mencari (p2)av. (c) Hitunglah Δp, dengan menggunakan hasil dari (a) dan (b). 9. Keadaan eksitasi pertama dari osilator harmonik memiliki fungsi gelombang berbentuk



. Ikuti metode yang diuraikan dalam Pasal 5.5 untuk menghitung a dan energi E. Carilah tetapan A dari syarat normalisasi. 10. Dengan menggunakan fungsi gelombang dari Persamaan (5.53) bagi potensial tangga,



terapkan syarat batas pada ѱ dan dѱ/dx untuk mencari B’ dan C’ dinyatakan dalam A’, bagi potensial tangga apabila partikel datang dari arah negatif x. Hitunglah nisbah |B’|2/|A’|2 dan |C’|2/|A’|2 dan tafsirkan semua hasil ini.



107



5 Model Atom Rutherford-Bohr



Atom memancarkan dan menyerap radiasi



elektromagnet.



Spektrum



radiasinya bermacam-macam – cahaya tampak (𝜆 ~ 500 nm), sinar X (𝜆 ~ 1 nm), sinar ultraviolet (𝜆 ~ 10 nm), sinar inframerah



(𝜆



~



0,1



µm),



dan



seterusnya. Ternyata, dari pengamatan terhadap radiasi yang dipancarkan dan diserap inilah kita pelajari sebagian besar hal yang kita ketahui tentang atom.



108



MODEL ATOM RUTHERFORD-BOHR Sasaran dalam bab ini adalah memahami beberapa hal terinci dari struktur atom, berdasarkan pengetahuan tentang atom yang kita peroleh lewat percobaan. Untuk “mempelajari” cara kerja suatu sistem mekanik, kita biasanya berusaha (1) memahami sifatsifat tiap bagiannya, dan (2) mengamati keterkaitan kerja tiap bagian tersebut dalam menghidupkan kerja sistem. Sebagai contoh, andaikanlah kita hendak “memahami” cara kerja sistem persneling mobil. Yang terlebih dahulu kita pelajari adalah komponen-komponennya, mulai dari mur, baut, roda-gigi, dan berbagai sumbu putarnya, kemudia cara kerja persneling. Setelah itu, tentunya barulah kita mengerti tentang pemilihan nisbah jumlah gigi pada berbagai roda gigi, laju putaran, keuntungan mekanis, dan seterusnya. Kita lalu dapat mengatakan bahwa kita “memahami” cara kerja sebuah persneling. Prosedur yang sama ini ternyata dapat pula kita lakukan tanpa pengamatan visual-seorang montir yang baik memperoleh informasi tentang persneling tadi, dan mencapai tingkat pengetahuan yang sama, dengan hanya menggunakan indera peraba; ini dimungkinkan karena bagian-bagian persneling dan ketidakteraturannya (ukuran roda-gigi, mur, dan baut) semuanya kurang lebih seukuran jari kita, sehingga kita dapat “merasa” struktur bagian-bagian sistem itu dan cara kerjanya. Tetapi untuk sebuah arloji mekanik, tampaknya kita mengalami kesulitan besar utuk memahaminya. Tanpa indera penglihatan, kita tidak mampu “merasakan” cara kerja arloji atau mencirikan berbagai komponennya, karena ukuran jari kita terlalu besar untuk merasakan keterubahan (variasi) dan ketidakteraturan komponen-komponen arloji. Di pihak lain, panjang gelombang cahaya lebih besar daripada ukuran bagian-bagian arloji, karena itu kita dapat melihat semua bagian arloji dan juga melihat bagaimana mereka berinteraksi. Kesimpulannya, semakin kecil suatu sistem, semakin kecil pula sarana atau alat yang harus kita pakai untuk mempelajarinya. Kembali sejenak ke usaha kita mempelajari cara kerja persneling. Kita dapati bahwa setelah seluruh bagiannya kita selidiki, barulah kita “memperoleh” gambaran tentang cara kerjanya. (Bentuknya dapat berupa suatu diagram pada kertas atau suatu kesan ingatan). Gambaran ini dapatkita pandang sebagai suatu “model” mengenai cara kerja sebuah persneling. Kerincian suatu model tentunya tidak dapat melebihi percobaan yang dapat kita lakukan, dan memang kita tidak perlu mengenalnya lebih terinci karena tujuan pembuatan suatu model adalah kegunaannya. Untuk sekedar mengendarai sebuah mobil misalnya, “model” cara kerja persneling tidaklah perlu terlalu terinci, cara penggunaan tangkai persneling sudahlah cukup; tetapi dalam merancang sebuah mobil memang kita memerlukan model yang lebih terinci.



109



Lalu, apa manfaatnya semua uraian di atas dalam kaitannya dengan usaha kita untuk mempelajari atom? Lewat beberapa percobaan, kita memperoleh sejumlah pengetahuan yang kemudian kita gunakan untuk membangun suatu model atom guna membangun kesan ingatan kita untuk memahami struktur atom. Kita akan mempelajari beberapa percobaan mendasar dengan atom, dan memperlihatkan bagaimana suatu model atom disusun berdasarkan kumpulan pengetahuan yang kita peroleh dari semua percobaan itu. Sama halnya dengan mempelajari persneling mobil, model ini hanyalah membantu kita untuk memahami beberapa ciri utama struktur atom saja, karena belum terlalu terinci untuk menjelaskan semua kemungkinan percobaan yang dapat dilakukan dengan atom. Karena itu, perlu pula kita selidiki bagaimana perilaku gelombang elektron mengubah model atom paling sederhana (yang akan kita bahas padabab 7) dan atom-atom kompleks yang dibangun dari “batu-bata” (building blocks) yang sama (yang bahasannya menyusul pada Bab 8).



5.1 SIFAT-SIFAT DASAR ATOM Sebelum kita menyusun suatu model atom, marilah kita rangkumkan beberapa sifat dasar atom. 1.



Atom sangatlah kecil, jari-jarinya sekitar 0,1 nm(0,1 X 10-9 m). Dengan demikian, mencoba “melihat” sebuah atom dengan menggunakan cahaya tampak (𝜆 ≅ 500 nm), lewat efekdifraksi misalnya adalah usaha yang sia-sia. Ini dikarenakan jari “tangan” cahaya tampak terlalu besar untuk menyelidiki struktur “arloji” atom yang sangat halus. (Kita dapat menaksir secara kasar ukuran maksimum sebuah atom dengan cara sebagai berikut. Besi memiliki rapat massa sekitar 8 gr/cm3 dan berat atom sekitar 50. Satu mol besi (50 gr) mengandung jumlah atom sebanyak bilangan Avogadro, sekitar 6 X 10 23 buah atom menempati volume sekitar 6 cm3, sehingga 1 atom menempati volume ruang sekitar 10-23 cm3. Jika kita menganggap atom-atom zat padat tersusun rapat sekali seperti bola-bola kerasyang bersentuhan, maka diameter sebuah atom adalah kurang lebih 3



√10−23 𝑐𝑚3 ≅ 2 X 10-8 cm = 0,2 nm).



2.



Semua atom stabil - mereka tidak membelah diri secara spontan menjadi bagian-bagian yang lebih kecil; semua gaya dalam yang mengikat atom haruslah berimbang. Iniberarti, semua gaya tarik dalam atom tentulah saling berlawanan, karena bila tidak demikian, semua atom dalam alam semesta ini akan berantakan.



3.



Semua atom mengandung elektron bermuatan negatif, namun netral. Jika sebuah atom atau sekumpulan atom kita ganggu dengan gaya yang cukup kuat, elektronnya akan terpancarkan. Kenyataan ini kita pelajari dari efek Compton dan efek fotoelektrik.



110



(Dalam Bab 4 akan kita peajari bahwa meskipun inti atom pada proses peluruhan radioaktif memancarkan elektron, elektron tersebut tidak “hadir” dalam inti atom, melainkan terciptakan oleh proses lain. Ketidakhadiran elektron dalam inti atom merupakan akibat hubungan ketidakpastian yang melarang semua elektron yang terpancarkan dari inti atom dengan energi yang diamati dalam laboratorium, hadir dalam inti atom. Sedangkan bagi atom, asas ketidakpastian tidak memberi suatu batasan. Karena itu,elektronnya kita anggap hadir dalam atom). Karena semua bahan bersifat netral secara elektrik, kita dapat menganggap bahwa kenyataan ini juga merupakan salah satu sifat atom. Dan memang, berbagai percobaan dengan berkas atom tunggal mendukung kebenaran anggapan ini. 4.



Atom memancarkan dan menyerap radiasi elektromagnet. Spektrum radiasinya bermacam-macam – cahaya tampak (𝜆 ~ 500 nm), sinar X (𝜆 ~ 1 nm), sinar ultraviolet (𝜆 ~ 10 nm), sinar inframerah (𝜆 ~ 0,1 µm), dan seterusnya. Ternyata, dari pengamatan terhadap radiasi yang dipancarkan dan diserap inilah kita pelajari sebagian besar hal yang kita ketahui tentang atom. Dalam pengukuran pancaran radiasi elektromagnet yang khas, arus elektrik dilewatkan melalui sebuah tabung berisi cuplikan wujud gas suatu unsur tertentu, yang kemudian memancarkan radiasi elektromagnet apabila sebuah atom yang tereksitasi berpindah kembali ke keadaan dasarnya. Berbagai penjang gelombang yang dipancarkan sebuah atom dapat diukur dengan ketelitian tinggi, misalnya dengan menggunakan kisi difraksi dalam hal cahaya tampak. Panjang gelombang radiasi yang diserap dapat diukur dengan melewatkan seberkas cahaya putih melalui suatu cuplikan gas, dan kemudian kita mengamati warna-warna apakah dari cahaya putih yang hilang karena diserap oleh gas. Salah satu hal menarik dari radiasi atom ini adalah bahwa semua atomselalu memancarkan dan menyerap radiasi dengan panjang gelombang yang sama – panjang gelombang yang hadir dalam percobaan pemancaran radiasi tidak lagi muncul dalam percobaan penyerapan radiasi. Dengan demikian, teori struktur atom yang berhasil haruslah mampu menjelaskan spektrum panjang gelombang radiasi yang dipancarkan dan diserap ini.



5.2 MODEL ATOM THOMSON Model struktur atom pertama adalah yang dikemukakan oleh JJ. Thomson, yang telah terkenal karena keberhasilanya mencirikan elektron dan mengukur nisbah muatan terhadap massa (e/m) elektron. Model atom Thomson ini berhasil menerangkan banyak sifat atom yang diketahui seperti: ukuran, massa, jumlah elektron, dan kenetralan muatan 111



elektrik. Dalam model ini, sebuah atom dipandang mengandung Z elektron yang dibenamkan dalam suatu bola bermuatan positif seragam. Muatan positif total bola adalah Ze, massanya pada dasarnya adalah massa atom (massa elektron terlalu ringan sehingga tidak banyak mempengaruhi massa atom), dan bahwa jari-jari R bola ini adalah jari-jari atom pula. (Model ini seringkali dikenal dengan nama model kue “puding prem” (plum pudding), karena elektron-elektron tersebar di seluruh atom seperti halnya kismis yang tersebar dalam kue puding prem atau roti kismis). Gaya pada sebuah elektron yang berjarak r dari pusat sebuah bola bermuatan positif berjari-jari R dapat dihitung menggunakan rumus-rumus dasarelektrostatik. Menurut gambar 6.1, fraksi volume sebuah bola berjari-jari r dari volume keseluruhan bola berjarijari R sama dengan fraksi muatan dalam bola itu dari muatan total Ze. Jadi, 4



Qdalam = Ze 43 3



𝜋𝑟 3



𝜋𝑅 3



𝑟3



= Ze 𝑅3



(6.1)



Menutrut hukum Gauss, medan elektrik pada jarak r dapat dicari dari muatan total yang terkandung di dalam bola berjari-jari r: 1



∫ 𝑬. 𝑑𝑺 = 𝜀 qdalam



(6.2)



0



Karena sifat simetri bola dari persoalannya, medan elektrik E tetap nilainya di seluruh permukaan bola, sehingga integralnya dapat langsung dihitung dengan hasil E.4πr2. Jadi, 1



E = 4𝜋𝜀



𝑞𝑑𝑎𝑙𝑎𝑚 𝑟2



0



Gambar 6.1 Model atom Thomson. Z buah elektron mungil tersebar secara seragam dalam sebuah bola bermuatan positif Ze berjari-jari R. Setiap permukaan bola berjari-jari r mengandung fraksi muatan sebanding dengan r3/R3.



Dengan menggunakan Persamaan (6.1) bagi muatan total yang terkandung di dalambola, kita peroleh 1



E = 4𝜋𝜀



𝑍𝑒 0



𝑅3



r



Karena sebuah elektron dengan muatan e menderita gaya sebesar F = eE, maka 𝑍𝑒 2



F = 4𝜋𝜀



0𝑅



Dengan k = Ze2/4𝜋𝜀0 𝑅 3. 112



3



r = kr



Gaya ini cenderung menarik elektron menuju pusat atom, sehingga hasilnya dapat memberantakkan atom. Oleh karena itu, harus ada gaya lain, yang melawan tarikan elektrik ini agar semua elektron dipertahankan tetap setimbang pada jari-jari r. Gaya tambahan ini dipasok oleh gaya tolak-menolak antara elektron sehingga semua elektron tetap dalam kesetimbangan mantap. Keadaan ini sama seperti yang dialami sebuah benda bermassa m yang tergantung pada sebuah pegas sebuah pegas dengan tetapan pegas k, dalam medan gravitasi bumi. Gaya pegas yang dialami benda, yang besarnya F = kx, berlawanan arah dengan tarikan gravitasi bumi, yang besarnya F = mg. Bendanya setimbang di bawah tarikan kedua gaya yang berlawanan arah itu. Jika bendanya kita pindahkan sedikit jauh dari kedudukan setimbangnya, kemudian dilepaskan, ia akan bergetar (osilasi) dengan frekuensi v = (1/2π)√𝑘/𝑚. Oleh karena itu, kita juga memperkirakan bahwa elektron-elektron dalam atom Thomson akan bergetar sekitar kedudukan setimbangnya dengan frekuensi v = (1/2π ) X √𝑘/𝑚. Dengan k adalah tetapan yang didefinisikan oleh Persamaan (6.4). Karena muatan elektrik yang bergetar memancarkan gelombang elektromagnet dengan frekuensi yang sama dengan frekuensi getarnya, dapatlah kita perkirakan, bahwa berdasarkan model Thomson, semua frekuensi radiasi yang dipancarkan atom akan memperlihatkan frekuensi ciri khas ini. Dalam Soal I pada bagian akhir bab ini anda akan perlihatkan bahwa frekuensi ini ternyata tidak cocok dengan frekuensi radiasi yang dipancarkan oleh atom. Kesulitan lain dari tafsiran ini muncul bilamana kita meninjau penyerapan radiasi oleh atom. Tentunya kita memperkirakan bahwa sebuah atom dalam model atom Thomson memancarkan radiasi pada frekuensi getarnya, dengan akibat amplitudo getarnya menurun, atau menyerap suatu radiasi pada frekuensi yang sama pula, dengan akibat amplitudo getarnya meningkat. Sebagaimana dikemukakan sebelumnya, ternyata seringkali atom-atom tidak memancarkan atau menyerap radiasi pada frekuensi yang sama. Kenyataan ini sulit diterangkan oleh model atom Thomson. Kegagalan mencolok model Thomson muncul dari hamburan partikel (proyektil) bermuatan atom. Tinjaulah gerak sebuah partikel bermuatan positif yang menerobos sebuah atom. Karena adanya gaya elektrik dari atom terhadap partikel tersebut, maka lintasannya mengalami pembelokan yang cukup berarti dari arah gerak semulanya. Gayagaya tersebut adalah (1) gaya tolak yang ditimbulkan muatan positif atom, dan (2) gaya 113



tarik oleh elektron-elektron yang bermuatan negatif. Kita menganggap bahwa massa partikel yang dibelokkan tersebut lebih besar daripada massa elektron, tetapi lebih kecil daripada massa atom. Pada peristiwa interaksi antara partikel dengan sebuah elektron, gaya tarik-menarik antara keduanya tentulah sama besar (menurut hukum ketiga Newton), sehingga yang terutama merasakan akibatnya adalah elektron yang massanya jauh lebih keci; sedangkan efeknya pada proyektil dapatlah diabaikan. (Bayangkan sebuah bola bowling yang menggelinding menerobos sekumpulan bola ping-pong!). Karena itu, hanyalah muatan positif atom yang perlu kita tinjau sebagai penyebab pembelokan lintasan partikel. Dengan alasan yang sama, kita abaikan pula gerak atom, yang lebih besar massanya, sebagai akibat pengaruh proyektil bermuatan yang lewat tersebut. Dengan demikian, percobaan kita adalah hamburan sebuah partikel bermuatan positif oleh bagian atom yang paling padat dan bermuatan positif dalam keadaan diam. Gambar 6.2 memperlihatkan pembelokan lintasan sebuah partikel, yang bergerak dengan laju v (kita menganggap v « sehingga dengan menggunakan mekanika tak relativistik, K = ½mv2) sepanjang seuah garis lurus berjarak b dari pusat atom seandainya tidak dibelokkan. [Jarak b disebut parameter impak (impact parameter)]. Tolakan gaya elektrik menyebabkan arah gerak partikel sedikit membelok, Sehingga setelah melewati atom, partikel bergerak sepanjang suatu lintasan yang agak membelok, sebesar sudut θ, dari arah gerak semulanya. Gambar 6.2 Sebuah partikel alfa bermuatan positif yang menerobos masuk ke dalam model atom Thomson, mengalami pembelokan sebesar sudut θ. Koordinat r dan θ menentukan letak partikel alfa ketika berada di dalam atom.



Kita dapat menghitung sudut belok θ ini dengan meninjau impuls yang diterima partikel, yang memberikannya sebagian momentum dalam arah y ∆py ≅ ∫ Fy dt



(6.5)



Pada sembarang titik sepanjang lintasan proyektil, berlaku Fy = F cos 𝝓 Dengan menganggap proyektilnya bermuatan q = ze, maka gaya F yang dialaminya adalah qE, dengan E diberikan oleh Persamaan (6.3),



114



𝑧𝑍𝑒 2



1



F = 4𝜋𝜀



0



𝑅3



r = zkr



(6.6)



di mana k adalah tetapan yang sama seperti yang didefinisikan oleh Persamaan (6.4). Karena cos 𝝓 ≅ b/r, kita peroleh 𝑏



∆py ≅ ∫ 𝑧𝑘𝑟. 𝑟 . dt = zkb ∫ 𝑑𝑡 = zkbT



(6.7)



T adalah waktu total yang dibutuhkan proyektiluntuk melewati atom, yang sama dengan jarak tempuh total dalam atom dibagi dengan laju rata-rata. Karena pembelokannya kecil, lintasannya dapat dihampiri dengan sebuah garis lurus, seperti diperlihatkan pada Gambar 6.3, dan karena laju rata-ratanya hampir sama dengan v, maka T≅



2√𝑅 2 −𝑏2



(6.8)



𝑣



Gambar 6.3 Geometri hampiran bagi pembelokan sebuah partikel alfa oleh sebuah atom Thomson. Sudut hambur, yang nilai maksimumnya adalah sekitar 0,01o, sengaja dibesarkan melebihi ukuran sebenarnya. dan ∆py ≅



2𝑧𝑘𝑏 𝑣



√𝑅 2 − 𝑏 2



(6.9)



Dengan menganggap px tidak berubah, kita peroleh 𝑝𝑦



tan θ = 𝑝 ≅ 𝑥



∆𝑝𝑦 𝑝



(6.10)



dan jika θ kecil, maka tan θ ≅ θ sehingga θ≅



∆𝑝𝑦 𝑝



=



2𝑧𝑘𝑏 𝑚𝑣 2



√𝑅 2 − 𝑏 2



(6.11)



Bila kita melakukan percobaan semacam ini yang dikenal sebagai percobaan hamburan, kita tidak dapat menembakkan satu proyektil k pada sebuah atom, dan juga tidak dapat mengendalikan atau menentukan parameter impak b. Yang kita lakukan adalah menembakkan seberkas partikel pada selembar tipis bahan tertentu. Berkasnya mungkin dibelokkan seperti yang diperlihatkan pada Gambar 6.4, dan kita dapat menentukan sudut hambur ⊖av atau sudut hambur ⊖maks-nya. (⊖ menyatakan sudut hambur yang diukur; sedangkan θ adalah sudut hambur oleh satu atom).



115



Gambar 6.4 Percobaan hamburan yang khas. Seberkas partikel datang dihamburkan oleh selembar tipis bahan; partikel-partikel yang dihamburkan diamati pada semua nilai ⊖ yang mungkin dalam laboratorium.



Dengan menurunkan Persamaan (6.11) terhadap b, kita dapat memperoleh yakni sudut hambur θmaks bagi satu tumbukan: θmaks =



𝑧𝑘𝑅 2



(6.12)



𝑚𝑣 2



Gambar 6.5 Geometri hamburan bagi satu atom. Partikel-partikel yangg memasuki suatu daerah cincin berjari-jari b dan lebar db dihamburkan ke dalam rentang sudut dθ pada θ.



Untuk mendapatkan sudut hambur rata-rata, atomnya kita tinjau dari sudut pandang partikel, dan membayangkan penampang berbentuk piringan bundar yang dilihat terbagi atas cincin-cincin sepusat. Setiap kali proyektil memasuki daerah sebuah cincin berjari-jari b dengan lebar db, ia dihamburkan ke dalam rentang sudut dθ sekitar θ (lihat Gambar 6.5). Fraksi partikel (dari berkas) datang pada atom yang memasuki cincin tersebut (dan dengan demikian yang terhambur pada sudut θ) adalah tidak lain daripada fraksi luas sasaran yang ditempati cincin tersebut, atau 2𝜋b db/𝜋R2. Sudut hambur rata-rata bagi satu tumbukan diperoleh dengan merata-ratakan terhadap semua nilai b yang mungkin: 𝑅 2𝜋b db



θ avg = ∫0



𝜋𝑅 2



θ



(6.13)



dengan melakukan integrasinya, dan mempergunakan θ dari Persamaan (6.11), akhirnya kita peroleh 𝜋 𝑧𝑘𝑅 2



θ avg =4



116



𝑚𝑣 2



(6.14)



CONTOH 5.1 Dengan menggunakan model Thomson dengan R = 0,1 nm (jari-jari khas atom), hitunglah sudut belok rata-rata per tumbukan apabila berkas partikel alfa berenergi 5 MeV (z = 2) dihamburkan dari atom emas (Z = 79). Pemecahan Besaran zkR2 dapat langsung dihitung, hasilnya: 𝑍𝑒 2



zkR2 = 2 (4𝜋𝜀



0𝑅



3



) R2 = 2,79



1,44 𝑒𝑉 𝑛𝑚 0,1 𝑛𝑚



≅ 2,3 keV



(Dengan mengingat bahwa e2/4𝜋𝜀0 = 1,44 eV.nm). Karena mv2 = 2K = 10 MeV, maka 𝜋 2,3 𝑘𝑒𝑉



θ av ≅ 4 10 𝑀𝑒𝑉 ≅ 2 X 10-4 rad Sudut ini terlalu kecil, sehingga patut dipertanyakan apakah pembelokan sekecil itu dapat diamati dalam laboratorium. Yang tidak kita tinjau di sini adalah kenyataan bahwa ketika menempuh suatu jarak tertentu dalam bahan, proyektil mengalami banyak sekali tumbukan dengan atom-atom bahan, dan setiap tumbukan akan membelokkan proyektil sebesar suatu sudut belok tertentu yang rata-ratanya adalah θav. Beberapa dari antara tumbukan ini memberikan hasil sudut belok total yang lebih besar, sedangkan yang lainnya memberikan hasil sudut belok total yang lebih kecil (lihat Gambar 6.6). Nilai sudut hambur total yang diamati ⊖ tunduk pada hukum-hukum statistik; khususnya bila terdapat N tumbukan, maka ⊖av = √𝑁θav dan bahwa probabilitas hamburan pada 2



sembarang sudut yang lebih besar daripada suatu sudut ⊖ adalah 𝑒 −(⊖/⊖𝑎𝑣) .



Gambar



6.6



Gambaran



mikroskopik



dari



hamburan. Beberapa hamburan tunggal cenderung memperbesar θ. Sedangkan yang lainnya cenderung memperkecil θ.



Pada kasus proyektil menembus selembar emas setebal 1 𝜇m (10-6m), ia akan menumbuk sekitar 104 buah atom (karena masingmasing atom memiliki diameter sekitar 0,1 nm). Karena itu, sudut hambur laboratoris rata-rata besarnya ⊖av sekitar √104 θav, atau sekitar 1o. Angka ini tidak menyimpang jauh dari yang diamati dalam berbagai percobaan seperti itu.



117



Tetapi, jika kita menguji probabilitas hamburan ini untuk sudut yang lebih besar (⊖>90o), kita dapati bahwa ramalannya meleset jauh dari percobaan. Untuk ⊖av ≅ 1o, 2



probabilitas yang kita perkirakan bagi sudut yang lebih besar daripada 90o adalah 𝑒 −90



= 𝑒 −8100 = 10-3500. Percobaan semacam ini dilakukan oleh Hans Geiger dan Ernest Marsden dalam laboratorium Profesor Ernest Rutherford pada 1910. Hasil yang mereka peroleh memperlihatkan bahwa probabilitas sebuah partikel alfa dihamburkan pada sudut-sudut yang lebih besar daripada 90o adalah sekitar 10-4. Penyimpangan yang cukup mencolok anatara hasil yang diperkirakan (yakni 10-3500) dan nilai yang diamati (10-4) dilukiskan oleh Prof. Rutherford dalam kata-kata berikut:



Ini adalah peristiwa sangat tidak masuk akal yang pernah terjadi dalam hidup saya. Ini sama tidak massuk akalnya dengan ibarat anda menembakkan sebuah peluru 15 inci pada selembar kertas tissue dan peluru itu kemudian balik menembaki anda.



Analisis terhadap berbagai hasil percobaan hamburan seperti ini mendorong Rutherford untuk mengusulkan bahwa massa dan muatan positif atom tidaklah tersebar secara merata dalam seluruh volume atom, tetapi terbatas hanya dalam suatu daerah sangat kecil, dengan diameter sekitar 10-14 m, pada pusat atom. Pada bagian berikut akan kita lihat bagaimana model yang diusulkan ini taat asas dengan hasil pengamatan berbagai percobaan hamburan dengan sudut-sudut hambur yang besar.



5.3 INTI ATOM RUTHERFORD Apabila seberkas partikel, seperti berkas partikel alfa menumbuki sebuah sasaran tipis seperti selembar tipis emas, sudut hambur rata-ratanya kecil(dalam orde sekitar 1o). Selanjutnya kita pelajari pula bahwa dalam keadaan seperti itu, probabilitas saling menjumlah dari sebagian besar pembelokan kecil itu untuk memberikan pembelokan besar yang teramati ternyata sangatlah kecil(10-3500) bertentangan dengan hasil pengamatan eksperimen(10-4). Cara paling mungkin bagi sebuah partikel alfa(m=4u) untuk dapat dibelokkan hingga mencapai sudut yang sangat besar adalah bila terjadi tumbukan tunggal dengan suatu objek sangat padat(masif). Rutherford dengan demikian mengusulkan bahwa muatan dan massa atom terpusatkan pada pusatnya, dalam suatu daerah yang disebut inti (nucleus). Gambar 6.7 melukiskan geometri hamburan bagi kasus ini. Proyektil, bermuatan ze, menderita gaya tolak oleh muatan positif inti sebesar: 118



𝐹=



(𝑧𝑒)(𝑍𝑒)



(6.15)



4𝜋𝜀0



(Proyektilnya kita anggap selalu berada diluar inti, sehingga ia rasakan muatan inti Ze secara lengkap). Elektron-elektron atom, dengan massanya yang lebih kecil, tidak banyak mempengaruhi lintasan proyektil, jadi pengaruhnya pada hamburan dapat kita abaikan. Kita juga menganggap massa inti besar sekali dibandingkan terhadap massa proyektil sehingga inti atom tidak bergerak selama berlangsung proses hamburan. Karena tidak ada gerak pental yang diberikan pada inti, energi kinetik awal dan akhir K dari proyektil sama besar.



Sebagaimana diperlihatkan oleh gambar 6.7, bagi setiap parameter impak b, terdapat suatu sudut hambur tertentu 𝜃. Jadi kita perlu mencari hubungan antara b dan 𝜃. Sebagaimana telah diturunkan dalam berbagai buku ajar, proyektil menempuh suatu lintasan berbentuk hiperbola; dalam koordinat polar r dan 𝜙, persamaan hiperbola adalah 1 𝑟



𝑧𝑍𝑒 2



1



= 𝑏 𝑠𝑖𝑛𝜙 + 8𝜋𝜀



0𝑏



2𝐾



(cos 𝜙 − 1)



(6.16) Sebagaimana diperlihatkan oleh gambar 6.8, kedudukan awal partikel adalah pada 𝜙 = 0, 𝑟 → ∞, dan kedudukan akhir adalah pada 𝜙 = 𝜋 − 𝜃, 𝑟 → 8. Dengan menggunakan kedua koordinat kedudukan akhir, persamaan (6.16) tersederhanakan menjadi 𝑧𝑍𝑒 2



𝑏 = 8𝜋𝜀



0



𝑏2𝐾



1



𝑧𝑍 𝑒 2



1



cot 2 𝜃 = 2𝐾 4𝜋𝜀 𝑐𝑜𝑡 2 𝜃



(6.17)



0



Sebuah partikel yang menghampiri inti atom dengan parameter impak b akan dihamburkan pada suatu sudut 𝜃; sedang yang menghampiri inti dengan nilai-nilai b yang



119



lebih kecil, akan dihamburkan dengan sudut yang lebih besar daripada 𝜃, seperti diperlihatkan pada gambar 6.7. Kajian kita terhadap hamburan partikel bermuatan oleh inti atom akan kita bagi dalam tiga bagian: (1) perhitungan fraksi partikel yang dihamburkan pada sudut yang lebih besar daripada 𝜃, (2) rumus Rutherford dan pembuktian kebenarannya lewat percobaan, dan (3) jarak terdekat ke inti atom, yang dapat dicapai oleh partikel bermuatan. 1. Fraksi partikel yang dihamburkan pada sudut yang lebih besar daripada 𝜃. Dari gambar 6.7 langsung kita lihat bahwa setiap partikel dengan parameter impak yang lebih kecil daripada suatu nilai tertentu b akan dihamburkan pada sudut yang lebih besar daripada 𝜃 (dari b) yang bersangkutan. Berapa besarkah peluang itu bagi sebuah partikel bermuatan yang memiliki parameter impak lebih kecil daripada b? Andaikan lembar tipisnya setebal satu atom – suatu lapisan tunggal atom-atom yang tersusun sangat rapat, seperti tampak pada gambar 6.9. Masing-masing atom tampak sebagai sebuah piringan bundar, dengan luas 𝜋𝑅 2. Jika lembar tersebut mengandung N buah atom , maka luas atomnya adalah 𝑁𝜋𝑅 2. Untuk hamburan untuk sudut yang lebih besar daripada 𝜃, parameter impaknya berada antara nol dan b – yang berarti bahwa jarak hampiran proyektil ke inti atom berada dalam daerah piringan bundar seluas 𝜋𝑏 2 . Jika semua proyektil dianggap tersebar merata pada luas lembar tadi , fraksi proyektil yang berada dalam luas tersebut adalah 𝜋𝑏 2 / 𝜋𝑅 2 . Ketebalan lembar hambur yang sebenarnya, dapat mencapai sekitar susunan seribu atau sepuluh ribu buah atom. Andaikanlah t adalah ketebalan lembar hambur dan A adalah luasnya, dan andaikanlah pula bahwa 𝜌 adalah kerapatan dan M adalah massa molekul bahan pembuat lembar itu. Jadi, volume lembar tersebut adalah At, dan massanya 𝜌𝐴𝑡, sehingga jumlah molnya 𝜌𝐴𝑡/𝑀. Jadi jumlah atom atau inti per satuan volume adalah 𝑛 = 𝑁𝐴



𝜌𝐴𝑡 1 𝑀 𝐴𝑡



=



𝑁𝐴 𝜌



(6.18)



𝑀



120



𝑁𝐴 adalah bilangan Avogadro. Bagi sebuah proyektil datang, jumlah inti atom per 𝜌𝑡



satuan luas yang tampak baginya adalah 𝑛𝑡 = 𝑁𝐴 𝑀 ; secara rata-rata , setiap inti 𝜌𝑡



memberi saham luas sebesar (𝑁𝐴 𝑀 )-1 pada medan tampak proyektil. Untuk sudut hambur yang lebih besar daripada 𝜃, proyektil harus berada dalam daerah lingkaran seluas 𝜋𝑏 2 yang berpusat pada sebuah atom. Dengan demikian, fraksi partikel yang dihamburkan pada sudut yang lebih besar daripada 𝜃 adalah tidak lain daripada jumlah partikel yang menghampiri sebuah atom dalam suatu daerah cakupan 𝜋𝑏 2 𝑓𝜃 = 𝑛𝑡𝜋𝑏 2



(6.19)



Dengan anggapan bahwa semua partikel datang tersebar merata pada luas lembar hambur. Contoh 6.2 Selembar emas (𝜌 = 19,3 𝑔/𝑐𝑚3 , 𝑀 = 197 𝑔/𝑚𝑜𝑙)dengan ketebalan 2,0.10-3cm, digunakan untuk menghamburkan partikel-partikel alfa berenergi kinetik 8,0 MeV. (a) berapa fraksi partikel alfa yang dihamburkan kedalam sudut yang lebih besar daripada 900? (b) berapa fraksi partikel alfa yang dihambur dalam sudut antara 900 dan 450? Pemecahan: (a) Untuk kasus ini, jumlah per satuan volume dapat dihitung sebagai berikut: (6,02 x 1023 atom/mol)(19,3 g/𝑐𝑚3 ) = 197𝑔/𝑚𝑜𝑙 = 5,9 𝑥 1028 𝑎𝑡𝑜𝑚/𝑚 Untuk hamburan pada sudut 900 , parameter impak b dapat dihitung dari persamaan (6.17): 𝑏=



2(79) (1,44𝑒𝑉. 𝑛𝑚) cot 450 = 1,4 𝑥 10−14 𝑚 2(8,0 𝑥 106 𝑒𝑉)



Sehingga 𝜋𝑏 2 = 6,4 𝑥10−28 𝑚2 /𝑖𝑛𝑡𝑖, dan dengan demikian kita peroleh: 𝑓>90 = (5,9 𝑥 1028 𝑖𝑛𝑡𝑖/𝑚3 )(2,0 𝑥 10−6 𝑚)(6,4 𝑥10−28 𝑚2 /𝑖𝑛𝑡𝑖) = 7,5 𝑥 10−5 (b) Dengan mengulangi perhitungan diatas untuk 𝜃 = 450 , kita peroleh: 2(79)



𝑏 = 2(8,0 𝑥 106 𝑒𝑉) (1,44𝑒𝑉. 𝑛𝑚) cot 22,50 = 3,4 𝑥 10−14 m



2. Rumus hamburan Rutherford dan bukti percobaanya. Agar kita dapat menghitung probabilitas hamburan sebuah partikel kedalam suatu selang sudut kecil pada 𝜃, kita



121



syaratkan parameter impaknya terletak dalam suatu selang kecil db di b (lihat gambar 6.10).



Dengan demikian, fraksi df adalah 𝑑𝑓 = 𝑛𝑡(2𝜋𝑏 𝑑𝑏) Menurut persamaan (6.19). Dengan mendiferensiasikan persamaan (6.17), kita peroleh pernyataan db dalam 𝑑𝜃 sebagai berikut: 𝑧𝑍 𝑒 2



1



1



𝑑𝑏 = 2𝐾 4𝜋𝜀 (1 − 𝑐𝑠𝑐 2 2 𝜃)(2 𝑑𝜃)



(6.20)



0



Jadi, 𝑧𝑍



𝑒2



1



1



|𝑑𝑓| = 𝜋𝑛𝑡( )2 ( )2 𝑐𝑠𝑐 2 2 𝜃 𝑐𝑜𝑡 2 𝜃 𝑑𝜃 2𝐾 4𝜋𝜀 0



(6.21)



(tanda minus pada persamaan (6.20) tidaklah begitu penting – ia hanyalah memberitahukan kita bahwa 𝜃 bertambah bila b berkurang). Andaikan kita tempatkan sebuah detektor bagi partikel yang terhambur pada sudut 𝜃 sejauh jarak r dari inti atom. Maka probabilitas bagi sebuah partikel untuk dihamburkan ke dalam detektor tersebut bergantung pada df. Namun demikian, df hanyalah memberikan peluang bagi semua proyektil yang dihamburkan pada sudut 𝜃 ke dalam 𝑑𝜃, dan dapat anda lihat bahwa semua proyektil itu akan terdistribusi secara merata sekitar sebuah cincin berjari-jari r sin 𝜃 dengan ketebalan r 𝑑𝜃. Luas cincinnya adalah dA =(2𝜋𝑟𝑠𝑖𝑛 𝜃) r 𝑑𝜃. Untuk menghitung laju arah hambur proyektil kedalam detektor, kita harus mengetahui probabilitas per satuan luas bagi hamburan kedalam daerah cincin tadi. Ini diberikan oleh |𝑑𝑓|𝑑𝐴, yang akan kita sebut N(𝜃). Selanjutnya, dengan melakukan suatu manipulasi perhitungan, akhirnya kita peroleh: 𝑛𝑡



𝑧𝑍



𝑒2



𝑁(𝜃) = 4𝑟 2 (2𝐾)2 (4𝜋𝜀 )2 0



1



(6.22)



1 2



𝑠𝑖𝑛4 𝜃



Ini adalah rumus hamburan Rutherford. Rumus Rutherford ini kemudian diuji kebenarannya dalam laboratorium Rutherford oleh Geiger dan Marsden, melalui serangkaian percobaan yang memerlukan ketelitian dan 122



keterampilan tinggi. Untuk percobaan ini, mereka menggunakan partikel-partikel alfa (z=2) dengan mengamati hamburannya dari berbagai jenis lembar tipis logam. Mengingat pada dewasa itu belum tersedia pencatat elektronik dan alat pemrosesnya, maka Geiger dan Marsden mengamati dan mencatat partikel-partikel alfanya dengan menghitung kerdipan cahaya(scintillatios) yang dihasilkan apabila partikel-partikel alfa tersebut menumbuk sebuah layar sulfida seng. Skema peralatan ini diperlihatkan pada gambar 6.11. hasilnya, keempat ramalan rumus hamburan Rutherford semuanya terbukti keberlakuannya:



(a) N(𝜃) ∝ 𝑡, untuk percobaan ini Geiger dan Marsden menggunakan sumber 8 MeV partikel alfa dari peluruhan radioaktif yang kemudian dihamburkan pada berbagai lembar hambur berketebalan t yang berbeda, dengan sudut hambur 𝜃 dipertahankan tetap pada 250. Hasil-hasil yang mereka peroleh diperlihatkan pada gambar 6.12 yang menampakkan secara jelas ketergantungan N(𝜃) pada t secara linear. Juga terbukti bahwa pada sudut hamburan sedang ini pun, hamburan tunggal lebih berperan daripada hamburan jamak (multiple).



123



(b) N(𝜃) ∝ 𝑍 2 , untuk percobaan ini Geiger dan Marsden menggunakan beraneka jenis bahan penghambur,dengan ketebalan yang hampir sama. Oleh karena itu, ketergantungan linear ini menjadi jauh lebih sulit diuji dibandingkan terhadap kasus (a), mengingat pengujiannya melibatkan ketebalan berbeda untuk bahan yang berbeda. Namun demikian, sebagaimana tampak pada gambar 6.13, hasil-hasil yang diperoleh sesuai dengan kebergantungan linear dari N(𝜃) pada 𝑍 2 . (c) N(𝜃) ∝ 1/𝐾 2 . Untuk menguji ramalan rumus hamburan Rutherford ini, Geiger dan Marsden mempertahankan ketebalan lembar hamburan tetap dan mengubah laju partikelpartikel alfanya. Hal ini mereka capai dengan memperlambat partikel-partikel alfa yang dipancarkan dari sumber radioaktif dengan menggunakan lembar tipis mika. Dari berbagai pengukuran secara terpisah, dapatlah mereka ketahui pengaruh beda ketebalan mika pada kecepatan partikel alfa. Semua hasil percobaan mereka itu diperlihatkan pada gambar 6.14, yang sekali lagi menampakkan kesesuaian yang luar biasa baik dengan hubungan yang diperkirakan.



124



(d) N(𝜃) ∝ 1/𝑠𝑖𝑛4 1/2𝜃. Ketergantungan N pada 𝜃 mungkin adalah ciri yang paling utama dan istimewa dari rumus hamburan Rutherford. Rumus ini juga menghasilkan perubahan terbesar dalam N yang dapat diacapai dalam percobaan. Dalam percobaan sebelumnya perubahan N hanya mencapai sekitar orde sepuluh; sedangkan dalam percobaan ini perubahan N malahan mencapai lima hingga 10 kali mulai dari sudut yang terkecil hingga yang terbesar. Untuk memperoleh hubungan antara N dan 𝜃 yang dilukiskan pada gambar 6.15, Geiger dan Marsden menggunakan selembar emas dengan mengubah 𝜃 dari 5 hingga 1500. Kecocokannya dengan rumus Rutherford sekali lagi sangat baik. Dengan demikian, semua ramalan rumus hamburan Rutherford terbuktikan kebenarannya lewat percobaan, yang sekaligus membuktikan keberadaan “inti atom”.



3. Jarak hampiri terdekat partikel hambur ke inti penghambur. Ketika sebuah partikel bermuatan



positif



menghampiri



sebuah 125



inti



atom,



geraknya



mengalami



perlambatan,karena sebagian energi kinetik awalnya diubah menjadi energi potensial yang berasal dari gaya tolak Coulomb inti atom. Semakin dekat partikelnya menghampiri inti atom, maka semakin besar pula energi potensial yang ia peroleh,kaena: 𝑧𝑍𝑒 2



1



𝑉 = 4𝜋𝜀



0



𝐼



1 1 𝑧𝑍𝑒 2 1 𝐸 = 𝑚𝑣𝑚𝑖𝑛 2 + = 𝑚𝑣 2 2 4𝜋𝜀0 𝑟𝑚𝑖𝑛 2 (6.23a) 𝑉=0 1



𝐾 = 𝑚𝑣 2 2



𝑙 = 𝑚𝑣𝑏 𝑧𝑍𝑒 2



𝑉 = 4𝜋𝜀



vmin v



0



1 𝑟𝑚𝑖𝑛



1



𝐾 = 2 𝑚𝑣𝑚𝑖𝑛 2



rmin



𝑙 = 𝑚𝑣𝑚𝑖𝑛 𝑟𝑚𝑖𝑛



b d



𝑉=



𝑧𝑍𝑒 2 1 4𝜋𝜀0 𝑑 K=0



Momentum sudut juga kekal. Ketika jauh dari inti atom, momentum sudut partikel adalah mvb, dan pada rmin, momentum sudutnya adalah mvminrmin. Karena itu, 𝑚𝑣𝑏 = 𝑚𝑣𝑚𝑖𝑛 𝑟𝑚𝑖𝑛 Atau 𝑣𝑚𝑖𝑛 =



𝑏 𝑟𝑚𝑖𝑛



𝑣



(6.23b) Dengan menggunakan persamaan (6.23a) dan (6.23b) kita peroleh 1 1 𝑟 2𝑣 2 1 𝑧𝑍𝑒 2 2 𝑚𝑣 = 𝑚 ( )+ 2 2 𝑟𝑚𝑖𝑛 2 4𝜋𝜀0 𝑟𝑚𝑖𝑛 (6.24) Persamaan ini dapat dipecahkan untuk memperoleh nilai rmin.



126



Perhatikan bahwa energi kinetik partikel tidaklah nol pada rmin. Kecuali jika b = 0. Pada saat itu jarak terdekat ke inti atom adalah d, jarak hampiran terdekat. Jarak ini dapat kita dapati dengan memecahkan persamaan (6.24) bagi rmin untuk b = 0 yang memberikan 𝑑=



1 𝑧𝑍𝑒 2 4𝜋𝜀0 𝐾



(6.25)



5.4 Spektrum Garis



Jika sebuah gas diletakkan di dalam tabung kemudian arus listrik dialirkan ke dalam tabung, gas akan memancarkan cahaya. Cahaya yang dipancarkan oleh setiap gas berbeda-beda dan merupakan karakteristik gas tersebut. Cahaya dipancarkan dalam bentuk spektrum garis dan bukan spektrum yang kontinu. Kenyataan bahwa gas memancarkan cahaya dalam bentuk spektrum garis diyakini berkaitan erat dengan struktur atom. Dengan demikian, spektrum garis atomik dapat digunakan untuk menguji kebenaran dari sebuah model atom.



Gambar spektrum garis berbagai gas Spektrum garis membentuk suatu deretan warna cahaya dengan panjang gelombang berbeda. Untuk gas hidrogen yang merupakan atom yang paling sederhana, deret panjang gelombang ini ternyata mempunyai pola tertentu yang dapat dinyatakan dalam bentuk persamaan matematis. Seorang guru matematika Swiss bernama Balmer



127



menyatakan deret untuk gas hidrogen sebagai persamaan berikut ini. selanjutnya, deret ini disebut deret Balmer.



  364,5



n2 n2  4



Dimana panjang gelombang dinyatakan dalam satuan nanometer (nm).



Peralatan untuk mengamati spektrum garis Beberapa orang yang lain kemudian menemukan deret-deret yang lain selain deret Balmer sehingga dikenal adanya deret Lyman, deret Paschen, Bracket, dan Pfund. Pola deret-deret ini ternyata serupa dan dapat dirangkum dalam satu persamaan. Persamaan ini disebut deret spektrum hidrogen. 1 1 + 2) 2 m n Dimana R adalah konstanta Rydberg yang nilainya 1,097 × 107 m−1.



 = R(



A. Deret Lyman (m = 1) 1 1 1 = R ( 2 + 2) 𝜆 1 n dengan n = 2, 3, 4, …. B. Deret Balmer (m = 2) 1 1 1 = R ( 2 + 2) 𝜆 2 n dengan n = 3, 4, 5 …. C. Deret Paschen (m = 3) 1 1 1 = R ( 2 + 2) 𝜆 3 n 128



dengan n = 4, 5, 6 …. D. Deret Bracket (m = 4) 1 1 1 = R ( 2 + 2) 𝜆 4 n dengan n = 5, 6, 7, ….



E. Deret Pfund (m = 5) 1 1 1 = R ( 2 + 2) 𝜆 5 n dengan n = 6, 7, 8 …. Dalam model atom Rutherford, elektron berputar mengelilingi inti atom dalam lintasan atau orbit. Elektron yang berputar dalam lintasan seolah-olah bergerak melingkar sehingga mengalami percepatan dalam geraknya. Menurut teori elektromagnetik, elektron yang mengalami percepatan akan memancarkan gelombang elektromagnetik secara kontinu. Ini berarti elektron lama kelamaan akan kehabisan energi dan jatuh ke dalam tarikan inti atom. Ini berarti elektron tidak stabil. Di pihak lain elektron memancarkan energi secara kontinu dalam spektrum kontinu. Ini bertentangan dengan kenyataan bahwa atom memancarkan spektrum garis.Ketidakstabilan elektron dan spektrum kontinu sebagai konsekuensi dari model atom Rutherford tidak sesuai dengan fakta bahwa atom haruslah stabil dan memancarkan spektrum garis. Diperlukan penjelasan lain yang dapat menjelaskan kestabilan atom dan spektrum garis atom hidrogen. Contoh Soal : Batas deret dari deret Paschen (n=3) adalah 820,1 nm. Tentukan ketiga panjang gelombang terpanjang dari deret Paschen tersebut! 𝑛2 1 = 0,8201 2 , 𝑛 = 4, 5, 6 𝑛 − 32 Ketiga panjang gelombang tersebut adalah : 42 = 1875𝑛𝑚 42 − 32



𝑛 = 4;



𝜆 = 820,1



𝑛 = 5;



𝜆 = 820,1



129



52 = 1281𝑛𝑚 5 − 32



𝑛 = 6;



62 𝜆 = 820,1 2 = 1094𝑛𝑚 6 − 32



Ketiga spektrum ini berada dalam rentang inframerah spektrum elektromagnetik



5.5 MODEL ATOM BOHR Setelah Rutherford mengemukakan bahwa massa dan muatan positif atom terhimpun pada suatu daerah kecil dipusatnya, fisikawan Denmark, Neils Bohr pada tahun 1931 mengemukakan bahwa atom ternyata mirip sistem planet mini, dengan elekton-elektron mengedari inti atom seperti halnya planet-planet mengedari matahari. Dengan alasan bahwa tata surya tidak runtuh karena tarikan gravitasi matahari dan antar planet, atom jua tidak runtuh karena tarikan elektrostastis Coulomb antara inti atom dan tiap elektron.



Gambar Model Atom Bohr Jari-jari orbit lingkarannya adalah r, dan elektron (bermassa m) bergerak dengan laju singgung tetap v. Gaya tarik Coulomb berperan memberikan percepatan sentripetal v2/2, jadi : 1 𝑞1 𝑞2 1 𝑒 2 𝑚𝑣 2 𝐹= = = 4𝜋𝜀0 𝑟 2 4𝜋𝜀0 𝑟 2 𝑟 Dengan energi kinetiknya : 1 1 𝑒2 𝐾 = 𝑚𝑣 2 = 2 8𝜋𝜀0 𝑟 Serta energi potensial sistem ini adalah energi potensial Coulomb : 𝑉=−



1 𝑒2 4𝜋𝜀0 𝑟



Dengan demikian, energi total sistem adalah : 1 𝑒2 1 𝑒2 𝐸 =𝐾+𝑉 = − 8𝜋𝜀0 𝑟 4𝜋𝜀0 𝑟 𝐸=−



1 𝑒2 8𝜋𝜀0 𝑟



Fisika klask meramalkan bahwa sebuah muatan elektrin mengalami percepatan, seperti elektron yang mengorbit dalam model ini, harus meradiasikan energi eletrokmagnet 130



secara kontinu. Ketika energi ini dipancarkan, energi totalnya menurun dan elektron berspiral menuju inti atom sehingga atomnya akan runtuh. Untuk mengatasi kesulitan ini, Bohr mengusulkan gagasan keadaan “mantab stasioner” yaitu keadaan gerak tertentu dalam mana elektron tidak meradiasikan energi elektromagnet. Dari sini Bohr menyimpulkan bahwa keadaan ini momentum sudut orbital elektron bernilai kelipatan bulat dari ħ. Vektor momentum sudut dalam fisika klasik didefinisikan sebagai 𝑰 = 𝒓 × 𝒑. Untuk momentum sudut elektron yang beredar mengelilingi inti atom, r tegak lurus p, sehingga kita dapat menyederhanakan menjadi 𝐼 = 𝑟𝑝 = 𝑚𝑣𝑟. Jadi postulat Bohr : 𝑚𝑣𝑟 = 𝑛ℏ Dimana n adalah sebuah bilangan bulat (n = 1, 2, 3, ...). dengan menggunakan pernyataan ini dan hubungan (6.29) bagi energi kinetik, 1 1 𝑛ℏ 2 1 𝑒2 𝑚𝑣 2 = 𝑚 ( ) = 2 2 𝑚𝑟 8𝜋𝜀0 𝑟 (6.33) Kita peroleh dengan deretan nilai jari-jari r yang diperkenankan, yakni : 𝑟𝑛 =



4𝜋𝜀0 2 𝑛 = 𝑎0 𝑛 2 𝑚𝑒 2



(6.34) Dimana didefinisikan jari-jari Bohr a0 𝑎0 =



4𝜋𝜀0 = 0.0529 𝑛𝑚 𝑚𝑒 2



(6.35) Bagi orbit elektron hanya jari-jari orbit tertentu sajalah yang diperkenankan oleh model Bohr. Jari-jaro orbit elektron hanya dapat bernilai a0, 4a0, 9a0, 16a0, dan seterusnya, tidak pernah benilai 3a0 atau 5a0. Dengan menggabungkan pernyataan r yang kita peroleh diatas dengan persamaan (6.31) kita peroleh : 𝑚𝑒 2 1 𝐸𝑛 = − 32𝜋 2 𝜀0 2 ℏ2 𝑛2 (6.36) Jelas n pada energi E mencirikan tingkat energi. Dengan menghitung semua nilai tetapannya, kita peroleh : 𝐸𝑛 =



−13,6 𝑒𝑉 𝑛2



131



(3.37) Semua tingkat energi ini ditunjukan secara sistematis pada gambar 6.21. n=∞



E=0



n=4



E = -0,8 Ev



n=3



E = -1,5 eV



n=2



E = -3,4 eV



n=1



E = -13,6 eV



Energi eksitasi suatu keadaan n adalah energi di atas keadaan dasar, En – E1. Jadi eksitasi pertama (n = 2) memiliki energi eksitasi sebesar -3,4 eV – (-13,6 eV) atau 10,2 eV. Keadaan eksitasi kedua memiliki energi eksitasi 12,1 eV dan seterusnya. Bohr menpostulatkan bahwa meskipun elektron tidak memancarkan radiasi eletromagnet ketika beredar pada suatu tingkat tertentu, ia dapat berpindah dari satu tingkat ke tingkat lain yang lebih rendah. Pada tingkat yang lebih rendah, energi yang dimiliki elektron lebih rendah dari pada di tingkat sebelumnya. Beda energi ini muncul sebagai sebuah kuantum radiasi berenergi hf yang sama besar dengan beda energi antara kedua tingkat tesebut. Artinya, jika elektron melompat dari n = n1 ke n = n2 seperti pada gambar 6.22 maka tepancar sebuah foton dengan energi



h 𝜐 = En1 – En2 atau 𝜐=



𝑚𝑒 4 1 1 ( 2 − 2) 3 2 3 64𝜋 𝜀0 ℏ 𝑛2 𝑛1



(6.39) Jadi panjang gelombang radiasi yang dipancarkan adalah 𝜆=



𝑐 64𝜋 3 𝜀0 2 ℏ3 𝑛1 2 𝑛2 2 = ( 2 ) 𝜐 𝑚𝑒 4 𝑛1 − 𝑛2 2



132



1 𝑛1 2 𝑛2 2 = ( ) 𝑅∞ 𝑛1 2 − 𝑛2 2 (6.40) Tetapan R∞ yang disebut tetapan Rydberg bernilai 1,0973731 x 107 m-1. Contoh 6.6 Carilah panjang gelombang transisi dari n1 = 3 ke n2 = 2 dan dari n1 = 4 ke n2 = 2. Pemecahan Persamaan (6.40) memberikan 1 𝑛1 2 𝑛2 2 𝜆= ( ) 𝑅∞ 𝑛1 2 − 𝑛2 2 𝜆=



1 32 22 ( ) = 656,1 𝑛𝑚 1,0973731 × 107 32 − 22



𝜆=



1 42 22 ( ) = 486,0 𝑛𝑚 1,0973731 × 107 42 − 22



Dan



Rumus Bohr juga menerangkan asas gabung Ritz, yang menyatakan bahwa jumlah dau frekuensi berbeda dalam spektrum pancar memberikan frekuensi ketiga lainnya dalam spektrum tersebut. Marilah kita tinjau transisi dari suatu keadaan n3 ke n2, yang kemudian disusul dengan transisi dari n2 ke n1. Dengan menggunakan persamaan (6.39) bagi kasus ini diproleh 1 1 𝜐𝑛3→𝑛2 = 𝑐𝑅∞ ( 2 − 2 ) 𝑛3 𝑛2 𝜐𝑛2→𝑛1 = 𝑐𝑅∞ (



1 1 − 2) 2 𝑛2 𝑛1



Dengan demikian 1 1 1 1 𝜐𝑛3→𝑛2 + 𝜐𝑛2→𝑛1 = 𝑐𝑅∞ ( 2 − 2 ) + 𝑐𝑅∞ ( 2 − 2 ) 𝑛3 𝑛2 𝑛2 𝑛1 = 𝑐𝑅∞ (



1 1 − ) 𝑛3 2 𝑛1 2



Dengan demikian model atom Bohr taat asas gabung Ritz. Elektron bukannya mengedari inti atom, melainkan bersama-sama dengan inti atom keduanya mengedari pusat masssa mereka berdua. Karena massa inti adalah sekitar 2000 kali lebih besar massa elektron, pusat massa ini letaknya dekat ke inti atom. Dengan demikian energi kinetik harus mengandung suku tambahan yang berasal dari gerka inti 133



atom, dan pengaruhnya adalah menurunkan tetapan Rydberg dari nilai R∞ menjadi R = R8/(1 + m/M). Disini M adalah massa inti atom. Pengaruh ini cenderungpula menaikkan nilai panjang gelombang hasil perhitungan. Kesalahan kedua yang kita lakukan adalah dalam mengubah frekuensi ke dalam panjang gelombang. Frekuensi foton yang dipancarkan sama dengan beda energi antara tingkat-tingkat energi atom, untuk menghitung panjang gelombangnya kita gunakan pernyataan 𝑐 = 𝜆𝜐 yang hanyalah benar dalam ruang hampa. Dengan meninjau ulang penurunan teori Bohr, kita dapati bahwa muatan inti atom hanya muncul pada satu tempat yakni dalam pernyataan bagi gaya elektrostatik antara inti atom dan elektron, persamaan (6.28). jika muatan inti atom adalah Ze, gaya Coulomb yang bekerja pada elektron adalah 1 𝑍𝑒 2 𝐹= 4𝜋𝜀0 𝑟 2 (6.41) Jadi faktor e2 semula diganti dengan Ze2. Dengan melakukan panyisipan ini pada hasil akhir, kita dapati jari-jari edar yang diperkenankan adalah : 4𝜋𝜀0 ℏ2 2 𝑎0 𝑛2 𝑟𝑛 = 𝑛 = 𝑍𝑒 2 𝑚 𝑍 (6.42) Dan energi menjadi 𝐸𝑛 = −



𝑚(𝑍𝑒 2 )2 1 𝑍2 = −(13.6 𝑒𝑉) 32𝜋 2 𝜀0 2 ℏ2 𝑛2 𝑛2



(6.43) Jadi, garis edar pada atom dengan nilai Z yang lebih tinggi, letaknya lebih dekat ke inti atom, dan memiliki energi yang lebih besar (negatif), yang berarti bahwa elektronnya terikat lebih kuat pada inti atomnya. Contoh soal 6.7 Hitunglah kedua panjang gelombang deret Balmer ion berilium teriosnisasi tiga kali (Z = 4) Pemecahan Karena semua radiasi deret Balmer berakhir pada tingkat n = 2, kedau panjang gelombang terpanjang tersebut adalah radiasi yang berkaitan dengan transisi n = 3 ke n = 2 dan n = 4 ke n = 2. Energi dan panjang gelombang radiasi yang bersangkutan adalah



134



1 1 𝐸3 − 𝐸2 = −(13,6 𝑒𝑉)(42 ) ( − ) = 30,2 𝑒𝑉 9 4 ℎ𝑐 1240 𝑒𝑉. 𝑛𝑚 𝜆= = = 41 𝑛𝑚 𝐸 30,2 𝑒𝑉 1 1 − ) = 40,8 𝑒𝑉 16 4 ℎ𝑐 1240 𝑒𝑉. 𝑛𝑚 𝜆= = = 30,4 𝑛𝑚 𝐸 40,8 𝑒𝑉



𝐸4 − 𝐸2 = −(13,6 𝑒𝑉)(42 ) (



Kedua radiasi ini berada dalam daerah ultraviolet. Kedua panjang gelombang ini dekat sekali dengan kedua panjang gelombang terpanjang deret Balmer. Jika kita menghitung panjang gelombang untuk berbagai transisi dari n1 ke n2 = 2, sehingga didapat 𝜆 = (364,5 𝑛𝑚)(



𝑛12 ) 𝑛12 − 4



Hal ini identik dengan persamaan (6.26) bagi deret Balmer. Sehingga dapat dilihat bahwa semua transisi yang dicirikan sebagai deret Balmer adalah yang dari semua tingkat lebih tinggi tingkat n = 2. Pencirian serupa dapat pula dilakukan bagi deret transisi lainnya, seperti diperlihatkan pada gambar 6.23 Rumus Bohr juga menerangkan asas gabung Ritz yang mengakatan bahwa jumlah dua frekuensi berada dalam spektrum pancar memberikan frekuensi ketiga lainnya dalam spektrum tersebut. Ditinjau dari transisi dari suatu keadaan n3 ke keadaan n2, yang kemudian disusul dengan transisi dari n2 ke n1. Dengan menggunakan persamaan (6.39) bagi kasus ini diperoleh 𝑣𝑛3 →𝑛2 = 𝑐𝑅∞ (



1 1 ) 𝑛32 𝑛22



𝑣𝑛2 →𝑛1 = 𝑐𝑅∞ (



1 1 ) 𝑛22 𝑛12



𝑣𝑛3 →𝑛2 + 𝑣𝑛3 →𝑛2 = 𝑐𝑅∞ ( = 𝑐𝑅∞ (



1 1 1 1 2 2 ) + 𝑐𝑅∞ ( 3 2 ) 𝑛3 𝑛2 𝑛2 𝑛2



1 1 ) 𝑛32 𝑛12



Frekuensi sebuah foton yang dipancarkan dalam transisi langsung dari n3 ke n1, jadi 𝑣𝑛3 →𝑛2 + 𝑣𝑛2 →𝑛1 = 𝑣𝑛3 →𝑛1 135



Dengan demikain model bohr taat asas dengan asas gabung Ritz. (Karena frekuensi dari sebuah foton yang dipancarkan berhubungan dengan energinya melalui hubungan E=hv, maka penjumlahan frekuensi di atas sama dengan penjumlahan energi.) Ketika kita menguji bagaimana model atom bohr untuk memahami mengapa spektrum serap tidak mengandung semua garis yang terdapat dalam spektrum pancar. Pada atom hidrogen pada keadaan dasarnya dengan elektron berada pada tingkat n=1. Misalkan atom hidrogen kemudian disinari dengan berkas foton berenergi 13,6 eV, fotonnya akan diserap atom dengan hasil pelepasan satu elektron menjadi bebas. Atom pada keadaan dasar juga dapat menyerap foton berenergi 10,2 eV yang berkaitan dengan energi antara n=1 dan n=2. Pada proses penyerapa ini, atom tereksitasi ke tingkat n=2 dan kemudian meluruh dengan cepat kembali ke tingkat dasar. Jika kita menyinari hidrogen yang berada dalam tabung dengan seberkas foton dengan sebaran energi kontinu, berkas transmisinya akan memperlihatkan intensitas lemah pada beberapa energi tertentu, diantaranya E = 10,2 eV dan 13,6 eV. Karena tingkat n=2 kembali dengan cepat ke tingkat dasar, sehingga elektron tidak sempat menyerap foton lain untuk bertansisi untuk naik ke tingkat n=3. Sehingga dapat disimpulkan bahwa spektrum serap hanya mengandung transisi-transisi pada tingkat n=1. Terdapat dua Kesalahan dalam penghitungan kita. Kesalahan pertama adalah massa berhingga dari inti atom, elektron mengedari inti atom, melainkan elektron dan inti atom bersama mengedari pusat massa dari atom tersebut dan pengaruh nya adalah menurunkan tetapan Rydberg dari nilai R∞ menjadi R = R8 / (1 + m/M). Kesalahan kedua adalah mengubah frekuensi ke dalam panjang gelombang. Frekuensi foton yang dipancarkan sama dengan energi yang antara tingkat-tingkat energi atom. Untuk menghitung panjang gelombangnya kita dapat menggunakan c = λv yang hanya benar pada kondisi ruang hampa. Karena percobaan ini berada pada laboratorium berudara maka pernyataan yang benar adalah λv = vudara ; vudara adalah laju cahaya dalam udara yang bernilai sama dengan c/1,0003. Kesalahan tersebut mengimbangi hingga suatu batasan ketelitian tertentu. Prosedur yang digunakan untuk teori atom bohr hidrogen juga dapat digunakan untuk atom yang berelektron tunggal. Kita dapat menghitung semua tingkat energi ion helium terionisasi sekali (atom helium tanpa satu elektron lainnya), ion lithium terionisasi dua kali dan seterusnya. Dengan meninjau atom bohr kita dapat bahwa muatan inti atom hanya muncul pada satu tempat yakni dalam pernyataan bagi gaya elektrostatik antara inti atom dan elektron. Jika muatan inti atom adalah Ze, maka



136



1 𝑍𝑒 2 𝐹= 4𝜋𝜀0 𝑟 2 Jadi, faktor e2 semula dapat diganti dengan Ze2 dengan melakukan penyisipan pada hasil akhir yang kita dapati bahwa jari-jari edar yang diperkenalkan adalah : 4𝜋𝜀0 ђ2 2 𝑎0 𝑛2 𝑟𝑛 = 𝑛 = 𝑍𝑒 2 𝑚 𝑍 Dan Energinya menjadi 𝑚(𝑍𝑒 2 )2 1 𝑍2 𝐸𝑛 = − = −(13,6 𝑒𝑉) 2 𝑛 32𝜋 2 𝜀02 ђ2 𝑛2 Sehingga garis edar pada atom dengan nilai Z lebih tinggi letaknya lebih dekat ke inti atom dan memiliki energi yang lebih besar (negatif); yang berarti bahwa elektronnya terikat lebih kuat dari inti atomnya



Contoh Hitunglah kedua panjang gelombang terpanjang deret Balmer ion berilium terionisasi tiga kali (Z=4)



Pemecahan Karena semua radiasi deret Balmer berakhir pada tingkat n=2, kedua panjang gelombang tersebut adalah radiasi yang berkaitan dengan transisi n = 3 → n = 2 dan n = 4 → n = 2. Energi dan panjang gelombang radiasi yang bersangkutan adalah 1 1 𝐸3 − 𝐸2 = −(13,6 𝑒𝑉)(42 ) ( − ) = 30,2 𝑒𝑉 9 4 ℎ𝑐 1240 𝑒𝑉. 𝑛𝑚 𝜆= = = 41,0 𝑛𝑚 𝐸 30,2 𝑒𝑉 𝐸4 − 𝐸2 = −(13,6 𝑒𝑉)(42 ) (



𝜆=



1 1 − ) = 40,8 𝑒𝑉 16 4



ℎ𝑐 1240 𝑒𝑉. 𝑛𝑚 = = 30,4 𝑛𝑚 𝐸 40,8 𝑒𝑉



Kedua radiasi ini berada dalam daerah ultraviolet.



137



5.6 Percobaan Frank-Hertz



Percobaan Frank-Hertz Pada Percobaan Frank-Hertz, elektron meninggalkan katoda yang dipanasi dengan sebuah filamen pemanas. Semua elektron tersebut dipercepat menuju sebuah kisi oleh beda potensial V. Yang dapat diatur. Elektron dengan energi V elektron-volt dapat menembusi kisi dan jatuh pada plat anoda, jika V lebih besar dari V0, suatu tegangan perlambat kecil antara kisi dan pelat katoda. Arus elektron yang mencapai pelat anoda diukur dengan menggunakan ammeter A. Jika tabung diisi dengan gas atom hidrogen, jika tegangan dinaikkan dari nol, makin banyak elektron yang mencapai pelat anoda, dan bersamaan dengan itu naik pula arus elektriknya. Elektron-elektron di dalam tabung tentu saja dapat menumbuk atom-atom hidrogen, namun tidak ada energi yang dilepaskan dalam tumbukan ini, jadi tumbukan elastik sempurna. Satu-satunya cara elektron dapat melepaskan energinya dalam suatu tumbukan dengan atom hidrogen adalah jika elektron memiliki energi yang cukup untuk menyebabkan atom hidrogen bertransisi ke suatu keadaan eksitasi. Dengan demikian, apabila energi elektron mencapai dan sidikit melebihi energi 10,2 eV elektron dapat melakukan tumbukan takelastik dengan atom hidrogen dan meninggalkan energi 10,2 eV pada atom hidrogen, sedangkan pada elektron setelah tumbukan bergerak dengan energi yang lebih rendah. Dengan demikian, jika elektron harus melewati kisi dan energinya tidak cukup untuk mengatasi tegangan perlambat rendah, ia tidak dapat mencapai pelat anoda. Dan apabila V = 12,1 V. Pada tegangan ini tumbukan tumbukan takelastik menyebabkan atom hidrogen. Tereksitasi ke tingkat n = 3. Proses ini akan terus berlangsung hingga V = 13,6 V, pada tegangan ini tumbukan akan menyebabkan atomnya terionisasi. Jika V dinaikkan terus, akan segera tampak efek tumbukan jamak



138



(multiple collisions). Artinya apabila V = 20,4 V sebuah elektron dapat melakukan tumbukan takelastik dengan atom dengan akibat mengeksitasi atomnya ke keadaan n = 2. Pada proses ini, elektron kehilangan energi 10,2 eV, sehingga setelah tumbukan ia bergerak dengan energi 10,2 eV, yang cukup untuk mengeksitasi atom hidrogen kedua lewat tumbukan takelastik. Jadi jika penurunan arus diamati terjadi pada tegangan V, penurunan serupa akan teramati pada tegangan-tegangan 2 V, 3 V, ... lebih umum jika penurunan arus yang teramati pada tegangan V1 dan V2. Maka penurunan arus yang sama kaan teramati pula pada tegangan-tegangan V1 + V2, 2V1 + V2, V1 + 2V2, dan seterusnya. Dengan demikian percobaan ini memberikan kita suatu bukti langsung mengenai kehadiran keadaan eksitasi atom. Tetapi tidak mudah untuk melakukan percobaan dengan atom hidrogen karena secara alamiah hidrogen tidak hadir dalam bentuk atom, melainkan dalam bentuk molekul H2 . karena molekul menyerap energi dalam berbagai cara, penafsiran percobaannya akan menjadi kabur. Pada tahun 1914, percobaan serupa dilakukan oleh Frank-Hertz dengan menggunakan tabung berisi uap air raksa. Hasil percobaan mereka memperlihatkan secara jelas bukti kehadiran sebuah keadaan eksitasi pada 4,9 eV. Apabila tegangannnya merupakan kelipatan dari 4,9 eV, maka tampak suatu penurunan dalam arus. Dan bertepatan dengan hal tersebut, sepktrum pancar dari uap air raksa memperlihatkan suatu garis benderang ultraviolet pada panjang gelombang 254 nm, yang berkaitan dengan energi sebesar 4,9 eV dan ini terjadi dari transisi antara keadaan eksitasi dengan energi 4,9 eV yang sama ke tingkat dasarnya. Dengan demikian bukti awal energi diskret dari berbagai keadaan atom tidak hanya mengukuhkan asasasas umum model atom bohr, tetapi juga memperlihatkan secara langsung kuantisasi energi dari berbagai sistem fisis.



Hasil Percobaan Frank-Hertz



139



5.7 Asas Persesuaian Pada model atom bohr memungkinkan untuk menghitung panang gelombang berbagai transisi dalam atom hidrogen yang kesesuaiannya dengan panjang gelombang yang diamati dalam beraneka spektrum pancar dan serap sangatlah mengesankan. Namun bohr harus terpaksa mengajukan dua postulat yang merupakan suatu keloncatan radikal dari fisika klasik. Terutama pada postulat yang mengakatan bahwa sebuah elektron dalam model atom bohr yang mengalami percepatan sewaktu beredar dalam garis edar lingkaran tidak meradiasikan energi elektromagnetik. Ini melanggar hukum fisika klasik yang mengatakan bahwa sebuah partikel bermuatan meradiasikan energi elektromagnet bila mengalami percepatan.teori relativistik memberi pernyataan energi kinetik dalam K = E –E0 , sedangkan fisika klasik memberi bentuk K = ½ mv2 , jika diperhatikan, E-E0 tersederhanakan menjadi ½ mv2 apabila v